\documentclass[10pt,a4paper]{article} %format des sujets \usepackage[french]{babel} \usepackage[T1]{fontenc} \usepackage[utf8x]{inputenc} \usepackage{fancyhdr} \usepackage{amsfonts,amsmath,amssymb,mathrsfs,graphicx,amsthm,dsfont} \usepackage{euscript,mathrsfs} \usepackage{stmaryrd} \usepackage{colortbl} \usepackage{comment} \usepackage{tikz} \usepackage{mathdots} \setlength{\topmargin}{-25mm} \setlength{\textheight}{25.5cm} \setlength{\oddsidemargin}{-10mm} \setlength{\textwidth}{17.5cm} \newcommand{\htrait}{\smallskip\hspace{-0.6cm}\rule{20cm}{0.5pt}\smallskip} % Pour séparer les exos \newcount\exocount % compteur exo \exocount=0 \newcount\espcount % compteur qsp \espcount=0 \newcount\planchecount \planchecount=0 \newcommand{\exos}{\vspace{3mm}\par\advance\planchecount by1 \begin{center} \Large {\bf SUJET S\the\planchecount}\\ \vspace*{1cm} {\large {\bf Exercice principal S\the\planchecount}} \end{center} \vspace*{2cm}} \newcommand{\Esps}{\vspace{3mm} \begin{center} \Large {\bf Exercice sans préparation S\the\planchecount}\\ \end{center} \vspace*{2cm}} \newcommand{\exo}{\vspace{3mm}\par\advance\exocount by1 \noindent{\hspace{-0.4cm}\bf Exercice principal n\textsuperscript{o}\the\exocount\ \ \\ \ \\ }} \newcommand{\Esp}{\vspace{3mm}\par\advance\espcount by1 \noindent{\hspace{-0.4cm}\bf Exercice sans préparation n\textsuperscript{o}\the\espcount\ \ \\ \ \\}} \everymath{\displaystyle} % applique un displaystyle à toutes les commandes de math %ensembles de nombres \def\C{\mathbb{C}} \def\D{\mathbb{D}} \def\K{\mathbb{K}} \def\H{\mathbb{H}} \def\N{\mathbb{N}} \def\Z{\mathbb{Z}} \def\Q{\mathbb{Q}} \def\R{\mathbb{R}} \def\U{\mathbb{U}} \def\E{\mathbb{E}} \def\P{\mathbb{P}} \def\V{\mathbb{V}} \def \A{{\cal A}} \newcommand{\Nstar}{\ensuremath{\mathbb{N}^{*}}} \newcommand{\Zstar}{\ensuremath{\mathbb{Z}^{*}}} \newcommand{\Qstar}{\ensuremath{\mathbb{Q}^{*}}} \newcommand{\Rstar}{\ensuremath{\mathbb{R}^{*}}} \newcommand{\Cstar}{\ensuremath{\mathbb{C}^{*}}} \newcommand{\Rplus}{\ensuremath{\mathbb{R}_{+}}} \newcommand{\Rmoins}{\ensuremath{\mathbb{R}_{-}}} \newcommand{\Rplusstar}{\ensuremath{\mathbb{R}_{+}^*}} \newcommand{\Rmoinsstar}{\ensuremath{\mathbb{R}_{-}^*}} \newcommand{\Rplusetoile}{\ensuremath{\mathbb{R}_{+}^*}} \newcommand{\Rmoinsetoile}{\ensuremath{\mathbb{R}_{-}^*}} % Divers \newcommand{\e}{\mathrm{e}} %e de exponentielle \newcommand{\Id}{\hbox{\rm{Id}\,}} \newcommand{\1}[1]{\mathds{1}_{#1}}% la fonction indicatrice pour les proba \newcommand{\llb}{\llbracket} \newcommand{\rrb}{\rrbracket} %Relation de comparaison \renewcommand{\leq}{\leqslant} \renewcommand{\geq}{\geqslant} \newcommand{\peto}[1]{\rm{o}\left(#1\right)} \newcommand{\gdo}[1]{\rm{O}\left(#1\right)} \newcommand{\eq}[1]{\operatorname*{\sim}_{#1} } %Algèbre \def \M{{\cal M}} \def \L {{\cal L}} \newcommand{\Sp}{\text{Sp}\,} \newcommand{\Ker}{\mbox{ Ker }} \newcommand{\im}{\mbox{ Im }} \newcommand{\Vect}{\ensuremath{\mathop{\rm Vect\,}\nolimits}} \newcommand{\rg}{\ensuremath{\mathop{\rm rg\,}\nolimits}} \newcommand{\tr}{\ensuremath{\mathop{\rm Tr\,}\nolimits}} \newcommand{\abs}[1]{\left| #1 \right|}%valeur absolue \newcommand{\norme}[1]{\left|\left| #1 \right|\right|}%norme \newcommand{\prodscal}[2]{\langle #1 ; \; #2 \rangle} %produit scalaire %Mettre entre .. \newcommand{\pa}[1]{\ensuremath{\left(#1\right)}}%parenthèses \newcommand{\paf}[2]{\ensuremath{\left(\frac{#1}{#2}\right)}} % fractions entre parenthèses \newcommand{\acco}[1]{\ensuremath{\left\{ #1\right\}}}%accolades \newcommand{\crocint}[1]{\left[\begin{array}{c} #1 \end{array}\right]}% grands crochets pour les IPP % Intégrations \newcommand{\dx}{\ensuremath{\,\mathrm{d}x}} \newcommand{\dt}{\ensuremath{\,\mathrm{d}t}} \newcommand{\du}{\ensuremath{\,\mathrm{d}u}} \newcommand{\dv}{\ensuremath{\,\mathrm{d}v}} \newcommand{\dy}{\ensuremath{\,\mathrm{d}y}} \newcommand{\parmi}[2]{\left( \begin{array}{c} #2 \\ #1 \end{array} \right)} %algo de Gauss \newcommand{\opgauss}[1]{\stackrel{\mbox{\scriptsize$\begin{array}{l}#1\end{array}$}}{\longrightarrow}}% Pour décrire les opérations de pivot arg= un tableau du type L_1\devient & L-1+... \newcommand{\devient}{\leftarrow} \newcommand{\echange}{\leftrightarrow} % Matrices par blocs \newcommand{\bigzero}{\mbox{\normalfont\Large\bfseries 0}} \newcommand{\rvline}{\hspace*{-\arraycolsep}\vline\hspace*{-\arraycolsep}} % Numérotation des question \newcommand{\be}{\begin{enumerate}} \newcommand{\ee}{\end{enumerate}} \newcommand{\bi}{\begin{itemize}} \newcommand{\ei}{\end{itemize}} % Fonction \newcommand{\fonc}[4]{ \begin{array}{lcl} {#1} & \rightarrow & {#2} \\ {#3} & \mapsto & {#4} \\ \end{array} } \renewenvironment{comment}{ \htrait\vspace*{4mm}\qquad \textbf{Solution :}\\ }{\par} % Pour masquer ou afficher les solutions \pagestyle{empty} \begin{document} \title{Rapport et sujets, oral HEC, Mathématiques (S) } \author{Juin-juillet 2021} \date{} \maketitle %\cfoot{} Dans leur grande majorit\'{e}, les candidats montrent de belles qualit\'{e}s de logique et de pr\'{e}sentation. Bien s\^{u}r, il y a une large diversit\'{e} entre eux, les notes s'\'{e}talant de 2 \`{a} 20. \\ Les candidats les plus faibles ont montr\'{e} d'importantes lacunes de cours et de grosses faiblesses en calcul.\\ A contrario, les meilleurs candidats \'{e}taient brillants dans leurs connaissances et dans leur finesse de raisonnement. La moyenne est de $11,21$ et l'\'{e}cart-type est de $3,93$. \\ \ Le jury aimerait insister sur certaines fautes récurrentes : \bi \item L'énoncé du théorème central limite est parfois surprenant et n'a souvent rien à voir avec celui du programme. \item Le jury insiste sur le rôle du tableau : il doit être utilisé comme un support et doit permettre à l'étudiant de commenter son travail mais il n'est pas nécessaire d'y voir tous les calculs. Un juste équilibre doit être trouvé, certains candidats veulent recopier l'intégralité de leur brouillon, d'autre ne rien écrire. \item Les étudiants pensent souvent à tort qu'une limite existe toujours et utilisent le symbole $\lim$ a mauvais escient. \item Il est utile de connaitre les lettres grecques. Il est parfois difficile de suivre les étudiants lorsqu'ils confondent deux lettres de leurs énoncés. \item Il est important qu'un étudiant sache dessiner les fonctions usuelles et connaissent l'interprétation géométrique de l'algèbre linéaire et bilinéaire. \item Nous avons fortement apprécié les étudiants qui remarquaient des incohérences dans leurs calculs, par exemple : \bi \item La variable est à valeurs de $[0,1]$ et mon calcul me donne une espérance de $2$. \item La fonction est positive et son intégrale est négative... \ei \ei Nous f\'{e}licitons ceux, nombreux, qui se sont accroch\'{e}s jusqu'au bout, essayant diverses m\'{e}thodes, proposant des id\'{e}es.\\ L'exercice sans préparation a très bien rempli son rôle et à permis de sauver des oraux mal commencés, ou de confirmer une prestation remarquable.\\ \ \\ Voici quelques sujets proposés cette année. Nous publions aussi leurs corrigés, mais insistons sur le fait que ces corrigés sont indicatifs, ont été écrits à l'intention des membres de jury et ne correspondent pas toujours exactement à ce que l'on peut attendre des élèves. \newpage \exos Soient $n\in\Nstar$ et $p\in[0,1]$. Soit $S_n$ une variable aléatoire qui suit la loi binomiale de paramètre $(n,p)$ et $x>0$. On pose $q=1-p$. \be \item {\bf Question de cours :} inégalité de Markov. \item Soit $\lambda>0$. Montrer que: $$\P(S_n-np\geq nx)\leq \frac{\E(\e^{\lambda(S_n-np)})}{\e^{n\lambda x}}$$ \item Montrer que $\E(\e^{\lambda(S_n-np)})=(p\e^{\lambda q}+q\e^{-\lambda p})^n$. \item On admet pour l'instant que, pour tout réel $t$, $\e^t\leq \e^{t^2}+t$. Montrer, en utilisant cette inégalité, que: $$\P(S_n-np\geq nx)\leq \e^{n(\lambda^2-\lambda x)}.$$ puis en déduire que: $$\P(S_n-np\geq nx)\leq \e^{-\frac{nx^2}{4}}.$$ \item Expliquer comment on démontrerait de la même façon que: $$\P(S_n-np\leq -nx)\leq \e^{-\frac{nx^2}{4}}.$$ \item En déduire: $$\P\left(\left|\frac{S_n}{n}-p\right|\geq x\right)\leq 2\e^{-\frac{nx^2}{4}}.$$ Comparer avec l'inégalité de Bienaymé-Tchebychev. \item Dans cette dernière question, on revient sur l'inégalité admise à la question 4. Etudier la fonction $f:t\mapsto e^{t^2-t}+t\e^{-t}$ et en déduire l'inégalité admise à la question 4. \ee %------------------------------------------------------------------------------------------------------------------ \begin{comment} \be \item Question de cours: programme ECS1 2013 p. 24. \item Soit $\lambda>0$. Comme $x\mapsto \e^{\lambda x}$ est une bijection strictement croissante: $$\P(S_n-np\geq nx)=\P(\e^{\lambda(S_n-np)}\geq \e^{n\lambda x})$$ puis par l'inégalité de Markov: $$\boxed{\P(S_n-np\geq nx)\leq \frac{\E(\e^{\lambda(S_n-np)})}{\e^{n\lambda x}}}$$ \item Par le théorème de transfert: \begin{eqnarray*} \E(\e^{\lambda(S_n-np)})&=& \sum_{k=0}^n\e^{\lambda(k-np)}\P(S_n=k)\\ & = & \sum_{k=0}^n\binom{n}{k}p^kq^{n-k}\e^{\lambda(k-np)}\\ &= & \e^{-\lambda np}(p\e^\lambda+q)^n \text{ par la formule du binôme}\\ &=& \boxed{(p\e^{\lambda q}+q\e^{-\lambda p})^n} \end{eqnarray*} comme demandé. \item On applique l'inégalité admise avec $t=\lambda q$ et $t=-\lambda p$. On en déduit: \begin{eqnarray*} p\e^{\lambda q}+q\e^{-\lambda p}&\leq& p(e^{\lambda^2q^2}+\lambda q)+q(e^{\lambda^2p^2}-\lambda p)\\ & \leq & p\e^{\lambda^2q^2}+q\e^{\lambda^2p^2}\\ & \leq & p\e^{\lambda^2}+q\e^{\lambda^2}\text{ car } p^2\leq 1\text{ et }q^2\leq 1\\ & \leq & e^{\lambda^2}\text{ car }p+q=1 \end{eqnarray*} Alors $\E(\e^{\lambda(S_n-np)})\leq e^{n\lambda^2}$ d'où finalement: $$\boxed{\P(S_n-np\geq nx)\leq \e^{n(\lambda^2-\lambda x)}}$$ Cette inégalité, dont le premier membre ne dépend pas de $\lambda$ est vraie pour tout $\lambda$. Or, à $x>0$ fixé, la fonction $\lambda\mapsto \lambda^2-\lambda x$ atteint son minimum pour la valeur $\lambda_0=\frac{x}{2}$ et ce minimum vaut alors $-\frac{x^2}{4}$. On en déduit donc: $$\boxed{\P(S_n-np\geq nx)\leq\e^{-\frac{nx^2}{4}}}$$ \item On démarre de la même façon en écrivant: $$\P(S_n-np\leq -nx)=\P(\lambda(n-S_n)\geq \lambda n x)\leq \frac{\E(\e^{\lambda(np-S_n)})}{\e^{n\lambda x}}$$ Il suffit alors de reprendre les calculs précédents en remplaçant $\lambda$ par $-\lambda$ et on obtient le même majorant à l'arrivée. \item Il suffit d'écrire: \begin{eqnarray*} \P\left(\left|\frac{S_n}{n}-p\right|\geq x\right)&=&\P(|S_n-np|\geq nx)\\ & = &\P(S_n-np\geq x)+\P(S_n-np\leq -nx)\\ &\leq & 2\e^{-\frac{nx^2}{4}} \end{eqnarray*} L'inégalité de Bienaymé-Tchebychev donne simplement: $$\P\left(\left|\frac{S_n}{n}-p\right|\geq x\right)\leq \frac{V\paf{S_n}{n}}{x^2}=\frac{p(1-p)}{nx^2}$$ L'inégalité de Bernstein \fbox{est nettement meilleure} pour les grandes valeurs de $n$ puisque l'exponentielle va plus vite vers zéro. \item On revient sur la preuve de l'inégalité utilisée dans la question 4. Cette preuve a été reportée en fin d'exercice car elle est essentiellement technique. On suit l'indication de l'énoncé et donc, soit $f:t\mapsto \e^{t^2-t}+t\e^{-t}$. La fonction $f$ est dérivable et, pour $t\in\R$: $$f'(t)=(2t-1)\e^{t^2-t}+(1-t)\e^{-t}=[(2t-1)\e^{t^2}+(1-t)]\e^{-t}$$ Le signe de $f'(t)$ est donc le même que celui de $\varphi(t)=(2t-1)\e^{t^2}+(1-t)$. La fonction $\varphi$ ainsi définie est dérivable sur $\R$ et, pour tout $t\in\R$: $$\varphi'(t)=(4t^2-2t+2)\e^{t^2}-1$$ La fonction $t\mapsto 4t^2-2t+2$ admet un minimum en $t_0=\frac{1}{4}$ et ce minimum est égal à $\frac{7}{4}$. On en déduit que: $$\forall t\in\R\quad \varphi'(t)\geq \frac{7}{4}\e^{t^2}-1\geq \frac{7}{4}-1>0$$ La fonction $\varphi$ est donc croissante. Or $\varphi(0)=0$ donc $\varphi$ est négative sur $\Rmoins$ et positive sur $\Rplus$. Il en est donc de même pour $f'$ et la fonction $f$ admet donc un minimum en $t=0$. Or $f(0)=1$. On en déduit donc que: $$\forall t\in\R\quad f(t)\geq 1\text{ soit } \boxed{\forall t\in\R\quad \e^t\leq \e^{t^2}+t}$$ \ee \end{comment} \newpage \Esps Soit $f$ définie sur $\R^+$ par $$\forall x\in \R^{+} \qquad f(x)=\sup_{n\in \N} \frac{x^n}{n!}$$ \be \item Donner une expression de $f(x)$ en fonction de $x$. \item Dessiner le graphe de $f$ sur $[0,4[$. \item $f$ est-elle continue sur $\R$ ? \ee \begin{comment} \be \item Soient $k\in \N$ et $x\in [k,k+1[$ (on a ainsi $k=\lfloor x \rfloor$) . Notons pour tout $n\in \N $ $ u_n(x)=\frac{x^n}{n!}$ $$ u_n(x)\leq u_{n+1}(x) \Leftrightarrow n+1\leq x\Leftrightarrow n\leq x-1\Leftrightarrow n<\lfloor x \rfloor$$ Donc $u_0(x)\leq u_1(x)\leq....\leq u_k(x)>u_{k+1}(x)>u_{k+2}(x)......$ $$f(x)=\frac{x^k}{k!}= \frac{x^{\lfloor x \rfloor }}{(\lfloor x \rfloor )!}$$ \item Sur le dessin, on voit la continuité mais pas la dérivabilité .\\ \includegraphics{Fig.eps} \item $f$ est continue et $C^1$ sur $\R^+$ privé de $\N^*$. Etude en $k$ \\ $$\lim_{\begin{array}{c}x\rightarrow k\\xk\end{array}}f(x)=\frac{k^{k-1}}{(k-1)!}=f(k).$$ $f$ est bien continue sur $\R$. On peut faire aussi chercher les dérivée à droite et à gauche (qui valent respectivement $\frac{k^{k-1}}{(k-1)!}$ et $\frac{k^{k-2}}{(k-2)!}$ si $k\geq 2$, et 0 et 1 si $k=1$) \ee \end{comment} \newpage \exos Soit $f$ une fonction continue sur $[0,1]$, non identiquement nulle, et à valeurs positives. \\ Pour tous polynômes $P$ et $Q$ de $\mathbb{R}[X]$, on pose~: \[ \varphi(P,Q) = \int_{0}^{1} f(t) P(t) Q(t) \text{d} t \] \begin{enumerate} \item {\bf Question de cours :} Définition de l'ordre de multiplicité d'une racine d'un polynôme. \item Justifier que $\varphi$ définit un produit scalaire sur $\mathbb{R}[X]$. \item Montrer que pour tout $n$ entier vérifiant $n \geqslant 1$, il existe une base orthogonale $(P_0,P_1,\ldots,P_n)$ de $\mathbb{R}_n[X]$ telle que~: $$\forall k \in \llbracket 0, n\rrbracket, \ \left\{ \begin{array}{l} \deg(P_k) = k \\ \text{et} \ P_k \ \text{ est unitaire} \end{array} \right. $$ \item Montrer que $P_1$ admet une racine dans $[0,1]$. \item Soit $k \geqslant 1$. On souhaite montrer que $P_k$ admet exactement $k$ racines réelles simples, toutes dans $[0,1]$. \\ Soit $j \in \llbracket 0, k \rrbracket$, et supposons que $P_k$ admette exactement $j$ racines distinctes dans $[0,1]$ qui soient d'ordre de multiplicité impaire, notées (dans le cas où $j \geqslant 1$) $a_1, a_2, \ldots,a_j$. \\ On note alors $Q(X) = \displaystyle \prod\limits_{i=1}^{j} (X-a_i)$ si $j \neq 0$ et $Q(X)=1$ si $j=0$. \begin{enumerate} \item \'Etudier le signe de $t \mapsto P_k(t)Q(t)$ sur $[0,1]$. \item Montrer que $P_k(X)Q(X)=0$ si $j < k$. \item Conclure. \end{enumerate} \item Montrer que pour tout $k \geqslant 1$, le polynôme $(P_k)^2+1$ n'admet que des racines complexes, toutes simples. \end{enumerate} \begin{comment} \begin{enumerate} \item Programme officiel ECS1 page 7. \item \begin{itemize} \item[$\bullet$] L'application $\varphi$ est clairement bien définie, symétrique, et bilinéaire par linéarité de l'intégrale. \item[$\bullet$] Soit $P \in \mathbb{R}[X]$. Alors $\varphi(P,P) = \int_{0}^{1} f(t) \left( P(t) \right)^2 \text{d} t$. \\ Comme $t \mapsto f(t) \left( P(t) \right)^2$ est continue positive, et que les bornes d'intégration sont dans le bon ordre, on a bien $\varphi(P,P) \geqslant 0$ par positivité de l'intégrale. \\ \item[$\bullet$] Soit $P \in \mathbb{R}[X]$ tel que $\varphi(P,P)=0$. \\ La fonction $t \mapsto f(t)\left( P(t) \right)^2$ étant continue positive d'intégrale nulle, elle est alors nulle sur $[0,1]$. \\ On a donc~: $\forall t \in [0,1], f(t) \left( P(t) \right)^2=0$. \\ Comme $f$ n'est pas identiquement nulle~: il existe au moins un réel $c \in [0,1]$ tel que $f(c) \neq 0$. \\ Comme $f$ est continue en $c$, il existe $\varepsilon > 0$ tel que $\forall t \in [0,1] \cap [c-\varepsilon, c+\varepsilon], f(t) \neq 0$. \\ Ainsi, $\forall t \in [0,1] \cap [c-\varepsilon,c+\varepsilon], P(t)=0$. \\ Le polynôme $P$ admet donc une infinité de racines, donc est nul. \end{itemize} \fbox{$\varphi$ est donc bien un produit scalaire.} Notons dans la suite $\| \cdot \|$ la norme associée. \item On procède par récurrence, en appliquant la \fbox{méthode d'orthonormalisation de Schmidt} à partir de la base canonique. \begin{itemize} \item[$\bullet$] Notons $P_0 = \displaystyle \frac{1}{\|1\|}$~: $P_0$ est un polynôme de degré $0$. \item[$\bullet$] Soit $k \geqslant 0$. \\ Supposons qu'on ait défini une base orthonormée $(P_0,\ldots,P_k)$ de $\mathbb{R}_k[X]$ telle que~: $\forall k \in \llbracket 0, n \rrbracket, \ \deg(P_k)~=~k$. \\ Notons $e_{k+1} = X^{k+1} - \sum\limits_{j=0}^{k} \varphi(X^{k+1},P_k) P_k$. \\ Alors $e_{k+1}$ est de degré $k+1$, donc n'appartient pas à $\mathbb{R}_k[X]=Vect(P_0,\ldots,P_k)$. \\ De plus, $\forall i \in \llbracket 0, k \rrbracket, \ \varphi(e_{k+1},P_i) = \varphi(X^{k+1},P_i) - \varphi(X^{k+1},P_i) \varphi(P_i,P_i) = 0$. \\ Ainsi, $e_{k+1}$ est non nul, et orthogonal à $(P_0,\ldots,P_k)$. \\ Ainsi, en posant $P_{k+1} = \displaystyle \frac{e_{k+1}}{\| e_{k+1}\|}$, la famille $(P_0,\ldots,P_k,P_{k+1})$ est une famille orthonormée de $\mathbb{R}_{k+1}[X]$, de cardinal $k+2=\dim(\mathbb{R}_{k+1}[X])$, donc \fbox{une base orthonormée de $\mathbb{R}_{k+1}[X]$.} \end{itemize} \item %Montrer que $P_1$ admet une racine dans $[0,1]$. On sait que $1 \in$Vect($P_0)$, donc $P_1 \perp 1$~: on doit avoir $\varphi(1,P_1)=0$. \[ \forall t \in [0,1], \ \int_{0}^{1} f(t) P_1(t) \text{d} t = 0. \] La fonction $P_1$ est continue sur $[0,1]$, donc est bornée et atteint ses bornes. Comme $\forall t \in [0,1], \ \min\limits_{u \in [0,1]}(P_1(u)) \leqslant P_1(t) \leqslant \max\limits_{u \in[0,1]}(P_1(u))$, en multipliant par $f(t) \geqslant 0$ et en intégrant (positivité de l'intégrale), on obtient~: \[ \min\limits_{u \in [0,1]}(P_1(u)) \int_{0}^{1} f(t) \text{d}t \leqslant 0 \leqslant \max\limits_{u \in[0,1]}(P_1(u)) \int_{0}^{1} f(t) \text{d} t\] Comme $f$ est positive non identiquement nulle, on a $ \int_{0}^{1} f(t) \text{d} t > 0$, donc nécessairement~: $\min\limits_{[0,1]} P_1 \leqslant 0 \leqslant \max\limits_{[0,1]} P_1 $. \\ Par Théorème des Valeurs Intermédiaires, \fbox{$P_1$ s'annule donc au moins une fois sur $[0,1]$} (et c'est la seule racine réelle puisque $\deg(P_1)=1$) \item Soit $k \geqslant 1$. \begin{enumerate} \item Remarquons que pour tout $k \geqslant 1$, $P_k \perp 1$ donc comme dans 4, $P_k$ admettra toujours au moins une racine dans $[0,1]$. \\ Notons $a_1,\ldots,a_j$ les racines d'ordre impair de $P_k$ dans $[0,1]$, de multiplicités respectives $\alpha_1,\ldots,\alpha_j$. \\ Notons $b_1,\ldots,b_m$ les (éventuelles) racines d'ordre pair de $P_k$ dans $[0,1]$, de multiplicités resp. $\beta_1,\ldots,\beta_m$. \\ Remarquons qu'on peut factoriser $P_k$ sous la forme \[ P_k = \prod\limits_{i=1}^{j} (X-a_i)^\alpha_{i} \cdot \prod\limits_{i=1}^{m} (X-b_i)^{\beta_i} \cdot R_k(X), \] où $R_k$ est un polynôme n'admettant pas de racines dans $[0,1]$ donc de signe constant sur $[0,1]$ car continu. Alors, on a~: \[ P_k(X)Q(X) = \prod\limits_{i=1}^{j} (X-a_i)^{\alpha_{i}+1}\cdot\prod\limits_{i=1}^{m} (X-b_i)^{\beta_i} \cdot R_k(X) \] Ainsi, $P_k(t)Q(t)$ est toujours du signe de $R_k(t)$ sur $[0,1]$, donc toujours \fbox{de signe constant.} \item Supposons $j < k$. Alors $Q(X) \in \mathbb{R}_j[X] = $Vect$(P_0,\ldots,P_j)$. Donc $P_k$ est orthogonal à $Q$. \[ \varphi(P_k,Q) = \int_{0}^{1} f(t) P_k(t) Q(t) \text{d} t = 0 \] Or, la fonction $t \mapsto f(t) P_k(t) Q(t)$ est continue, de signe constant, d'intégrale nulle, donc est nulle sur~$[0,1]$. \\ Comme à la question 1, on montre que \fbox{le polynôme $P_k Q$} est nul car admet une infinité de racines. \item Or, $P_k$ est de degré $k \geqslant 1$ et on a toujours $Q \neq 0$, donc c'est absurde d'avoir $P_kQ=0$. \\ Ainsi, on a nécessairement $j=k$. Ainsi, $P_k$ admet exactement $k$ racines réelles dans $[0,1]$, d'ordre de multiplicité impaire, donc toutes ses racines d'ordre de multiplicité $1$ nécessairement, donc \fbox{toutes simples, et toutes dans $[0,1]$.} \end{enumerate} \item %Montrer que pour tout $k \geqslant 1$, le polynôme $(P_k)^2+1$ n'admet que des racines réelles, toutes simples. Soit $k \geqslant 1$. Déjà, pour tout réel $t$, on a $(P_k(t))^2+1 > 0$, donc $P_k^2+1$ ne peut pas avoir de racines réelles. \\ Supposons que $P_k^2+1$ admette une racine au moins double $\alpha \in \mathbb{C} \setminus \mathbb{R}$. On aurait~: \[ 2 P_k(\alpha) P_k'(\alpha)=0 \] Donc $\alpha$ serait racine de $P_k$ ou racine de $P_k'$. Or, $P_k$ admet $k$ racines réelles uniquement, donc $P_k(\alpha) \neq 0$. De plus, en notant toujours $a_1 < \cdots < a_k$ les racines de $P_k$, en appliquant Rolle sur chaque intervalle $[a_i,a_{i+1}]$ ($i \in \llbracket 1, k-1 \rrbracket$), $P_k'$ s'annule déjà au moins $k-1$ fois (une fois sur chacun de ces intervalles), en étant de degré $k-1$. \\ Donc $P_k'$ admet exactement $k-1$ racines, toutes réelles. Ainsi,\fbox{ $ P_k^2+1$ ne peut pas avoir de racine double.} \end{enumerate} \end{comment} \newpage \Esps On dispose d'un d\'e \`a $6$ faces non pip\'e.\\ Proposer une m\'ethode pour effectuer un tirage au sort \'equitable entre $24$ individus (c'est à dire un tirage uniforme entre $1$ et $24$) en au plus 3 lancers.\\ Implémenter la méthode avec Scilab et proposer un programme qui affiche un histogramme permettant de valider la méthode proposée. \begin{comment} On lance le d\'e $6$ fois, on modélise par $X_1,...,X_3$, $3$ variables aléatoires indépendantes suivant des lois uniformes sur $\llb 1, 6 \rrb$. On divise les 24 résultats possibles en quatre "lots" de 6 On utilise les deux derniers lancers pour d\'eterminer quel sous ensemble choisit parmi les quatre, en fonction de la valeur de ces deux derniers lancers par rapport \`a $3$. $X=X_1+12* \1{[X_2\leq 3]}+6* \1{[X_3\leq 3]}$. On peut écrire le programme Scilab suivant: \begin{verbatim} Ntest=50000; test=[]; for i=1:Ntest de=grand(3,1,"uin",1,6); choix=de(1); if de(2)<4 then choix=choix+de(2)*12 end if de(3)<4 then choix=choix+de(3)*6 end test=[test,choix]; end; histplot(0.5:25,test) \end{verbatim} \end{comment} \newpage \exos Soit $n$ un entier supérieur ou égal à 2. On munit $\R^n$ de sa norme euclidienne canonique. Pour tout $x= (x_1,\dots ,x_n)\in \R^n$, on pose $f(x) = \int_0^{+\infty} e^{-t} (1+x_1t + x_2t^2 + \dots + x_nt^n)^2 \dt$. \begin{enumerate} \item \textbf{Question de cours :} Donner une condition suffisante pour qu'une fonction de plusieurs variables admette des extrema globaux sur un ensemble donné. \item Montrer que la fonction $f$ est bien définie. \item On note $M\in {\cal M}_{n+1}(\R)$ la matrice définie par $M=((i+j)!)_{0\leqslant i,j\leqslant n}$ Soit $x = (x_1,\dots, x_n)\in \R^n$, on note $u=\begin{pmatrix} 1 \\ x_1 \\ \vdots \\x_n \end{pmatrix}\in {\cal M}_{n+1,1}(\R)$ Montrer que $f(x) = {}^t u M u$ \item \be\item On admet que pour tout $u\in {\cal M}_{n+1,1}(\R)$, si $u\neq 0$ alors $^tuMu> 0$ Montrer que les valeurs propres de M sont toutes strictement positives. \item En déduire qu'il existe un réel A strictement positif tel que pour tout $x\in\R^n$ : $$f(x)\geq A \norme{x}^2$$ \ee \item En déduire que $f$ admet un minimum global. On notera $a=(a_1,\dots, a_n)$ un point où ce minimum est atteint. \item \begin{enumerate} \item Montrer que, pour tout $k\in \llbracket 1, n\rrbracket$, $k! + (k+1)!a_1 + \dots + (k+n)!a_n = 0$. \item On pose $P(X) = 1 + a_1(X+1) + a_2(X+1)(X+2) + \dots + a_n(X+1)(X+2) \cdot\cdot\cdot(X+n)$.\\ Montrer que $P(X) = \dfrac{(-1)^n}{(n+1)!}(X-1)(X-2)\cdot\cdot \cdot(X-n)$. \item Montrer que $f(a)=P(0)=\frac1n$. \end{enumerate} \end{enumerate} \begin{comment} \begin{enumerate} \item Programme officiel ECS2 page 19. \item Soit $x=(x_1,\dots,x_n)\in \R^n$. Soit la fonction $g : t \mapsto e^{-t} (1+x_1t + x_2t^2 + \dots + x_nt^n)^2$ est continue et positive sur $[0,+\infty[$. De plus, $g(t) \underset{t\rightarrow +\infty}{=} \peto{\dfrac{1}{t^2}}$ par croissances comparées, donc \fbox{l'intégrale $f(x)$ converge} par domination. \item Par linéarité (toutes les intégrales convergent par le même raisonnement qu'à la question précédente), on trouve que : \begin{align*} f(x) &= \int_0^{+\infty} e^{-t} (1+x_1t + x_2t^2 + \dots + x_nt^n)^2 \dt \\ &= \int_0^{+\infty} e^{-t}\dt +2 \sum_{k=1}^n x_k \int_0^{+\infty} t^{k}ke^{-t}\dt + + \sum_{i=1}^n\sum_{j=1}^n x_ix_j \int_0^{+\infty} t^{i+j}e^{-t}\dt \\ &= 1 + 2\sum_{k=1}^n k! x_k + \sum_{i=1}^n\sum_{j=1}^n (i+j)!x_ix_j \\ &= \sum_{i=0}^n\sum_{j=0}^n (i+j)!x_ix_j\text{ en notant }x_0=1\\ &\boxed{= u^TMu} \end{align*} Remarque : $\int_0^{+\infty} t^{i+j}e^{-t}\dt=(i+j)!$ en utilisant la densité d'une loi $\gamma(i+j)$, ou éventuellement par récurrence. \item \be \item Soit $\lambda$ une valeur propre de $M$ et $u$ un vecteur propre associé Comme $u$ est non nul, $u^TMu=\lambda \norme{u}^2>0$. Donc \fbox{Toutes les valeurs propres de $M$ sont strictement positives.} \item La matrice $M$ étant symétrique réelle, elle est diagonalisable d'après le théorème spectral et il existe donc une matrice diagonale $D=\mathop{diag}(\lambda_1,\dots,\lambda_n)$ (on peut supposer $0 < \lambda_0 \leqslant \lambda _1 \leqslant \dots \leqslant \lambda_n$) et une matrice orthogonale $P$ telle que $M = PD{}^t P$. On peut alors écrire avec les notations précédentes, en posant $v = {}^t Pu= (v_0,\dots,v_n)$ : \[ f(x) = {}^t u PD{}^t Pu = {}^t v D v = \sum_{k=0}^n \lambda_kv_k^2 \geqslant \lambda_0 \sum_{k=0}^n v_k^2 = \lambda_0 \|v\|^2 = \lambda_0 \|u\|^2 \boxed{\geqslant \lambda_0\|x\|^2.} \] \ee \item D'après la question précédente, il existe un réel $r >0$ tel que pour tout $x \in \R^n,\ \|x \| > r \Rightarrow f(x) > f(0)$. La fonction $f$ étant polynômiale, elle est continue sur la boule fermée bornée $\mathcal{B}= \{x \in \R^n \mid \|x\| \leqslant r\}$. Elle admet donc un minimum $m$ sur $\mathcal{B}$. Par construction de $r$, on a : $\forall x \in \R^n \setminus \mathcal{B},\ f(x) > f(0) \geqslant m$. \fbox{La fonction $f$ admet donc un minimum global sur $\R^n$.} \item \begin{enumerate} \item Puisque la fonction $f$ est polynômiale, donc $\mathcal{C}^1$, sur l'ouvert $\R^n$, le point $a$ est un point critique, i.e. : \[ \forall k \in \llbracket 1, n\rrbracket,\ \dfrac{\partial f}{\partial x_k}(x) = 0. \] Or, pour tout $k \in \llbracket 1, n\rrbracket,\ \dfrac{\partial f}{\partial x_k}(a) = 2k! + 2\sum_{i=1}^na_k(k+i)!$. \fbox{Pour tout $k \in \llbracket 1, n\rrbracket,$ $k! + \sum_{i=1}^na_k(k+i)!=0$.} \item Pour tout $k\in \llbracket 1,n\rrbracket,\ k!P(k)=k! + \sum_{i=1}^na_k(k+i)!=0$. Le polynôme $P$ est de degré inférieur ou égal à $n$ et admet $n$ racines distinctes. Il existe donc $\lambda \in \R$ tel que $P(X) = \lambda(X-1)\cdot \cdot \cdot (X-n)$. Puisque $P(-1) = 1$, on trouve $\lambda = \dfrac{(-1)^n}{(n+1)!}$. \fbox{$P(X)=\dfrac{(-1)^n}{(n+1)!}(X-1)\cdot \cdot \cdot (X-n)$.} \item On a, par linéarité (toutes les intégrales convergent) : \begin{align*} f(a) &= \int_0^{+\infty} e^{-t} (1+a_1t + a_2t^2 + \dots + a_nt^n)^2 \dt \\ &= \int_0^{+\infty} e^{-t} (1+a_1t + a_2t^2 + \dots + a_nt^n) \dt + \sum_{k=1}^n a_k \int_0^{+\infty} t^k e^{-t} (1+x_1t + x_2t^2 + \dots + x_nt^n) \dt. \end{align*} Or : \[ \forall k\in \llbracket 1, n\rrbracket,\ \int_0^{+\infty} t^k e^{-t} (1+a_1t + a_2t^2 + \dots + a_nt^n) \dt = k! + a_1(k+1)! + a_2(k+2)! + \dots + a_n(k+n)! = 0. \] Ainsi : \[ f(a) = \int_0^{+\infty} e^{-t} (1+a_1t + a_2t^2 + \dots + a_nt^n) \dt = 1 + \sum_{k=1}^n a_k k! = P(0) \boxed{= \dfrac{1}{n}}. \] \end{enumerate} \end{enumerate} \end{comment} \newpage \Esps On tire (avec remise) une boule d'une urne contenant $n$ boules distinctes. On note $V$ la variable al\'eatoire \'egale au num\'ero du tirage o\`u pour la premi\`ere fois, chaque boule a \'et\'e tir\'ee au moins une fois. Calculer l'esp\'erance de $V$ et trouver un \'equivalent quand $n$ tend vers l'infini. Proposer un programme Scilab pour tester ce résultat. \begin{comment} Soit $T_i$ le num\'ero de tirage lorsque $i$ boules diff\'erentes ont \'et\'e tir\'ees au moins une fois. En notant $V_i=T_i-T_{i-1}$ et $V_1=1$, on a $V=V_1+...+V_n$. Or, $V_i$ est \'egal au nombre de tirages pour tirer une des $n-(i-1)$ boules qui n'ont pas encore \'et\'e tir\'ees. $V_i$ suit donc une loi g\'eom\'etrique de param\`etre $$ p_i={{n-(i-1)}\over n} $$ D'o\`u $$ E(V)=E(V_1)+...+E(V_n)={1\over {p_1}}+...{1\over {p_n}}=\boxed{\sum_{i=1}^n{n\over {n-i+1}}} $$ En particulier $\boxed{E(V)\sim n\ln n}$ quand $n$ tend vers l'infini. On peut proposer le programme Scilab suivant: \begin{verbatim} n=30; Ntest=10000; esp=0; E=0 compt=0; for i=1:Ntest A=zeros(1,n); while sum(A)0}S_r$ et que $S_r\cap S_r'=\varnothing$ pour $r\neq r'$ avec $r$ et $r'$ dans $\Rplusstar$. Comme, pour tout $x\in S_r$, $f(x)\geq m(r)$, il suffit de montrer qu'il existe $r_1>0$ tel que $m(r_1)\leq m(r)$ pour tout $r>0$ pour montrer que la fonction $f$ admet un minimum global. Or, pour $r\geq r_0=\frac{1}{2\sqrt{n}}$, $m(r)=r^2-\frac{1}{4}\geq r_0^2-\frac{1}{4}$. On peut par ailleurs remarquer que: $$r_0^2-\frac{1}{4}=(n+1)r_0^2-r_0\sqrt{n}$$ Maintenant la fonction $r\mapsto (n+1)r^2-r\sqrt{n}$ est une fonction polynomiale de degré 2 qui admet un minimum en $$r_1=\frac{1}{2}\frac{\sqrt{n}}{n+1}$$ On peut vérifier que l'on a bien $0n$. Ainsi, pour tout $r\in]0,r_0]$: $m(r_1)\leq m(r)$. En particulier: $m(r_1)0\quad m(r_1)\leq m(r)\text{ où }r_1=\frac{1}{2}\frac{\sqrt{n}}{n+1}$$ Or: $$m(r_1)=(n+1)r_1^2-r_1\sqrt{n}=\frac{1}{4}\frac{n}{n+1}-\frac{1}{2}\frac{n}{n+1}\boxed{=-\frac{1}{4}\frac{n}{n+1}}$$ En particulier $m(r_1)<0=f(0,\ldots,0)$. Ainsi, la fonction $f$ admet un minimum global sur $\R^n$ égal à $\boxed{-\frac{1}{4}\frac{n}{n+1}}$. On retrouve bien la valeur obtenue à la question 5. \item On note $g(x)=\sum_{k=1}^nx_k^2$. La fonction $g$ ainsi définie est de classe ${\cal C}^1$ sur $\R^n$. Les points $x=(x_1,\ldots,x_n)\in\R^n$ où les extrema de $f$ sous la contrainte ${\cal C}_r$ sont atteints sont nécessairement ceux pour lesquels $g(x)=r^2$ et pour lesquels il existe $\lambda\in\R$ tel que: $$\forall i\in\llb1,n\rrb\quad \partial_if(x)=\lambda\partial_ig(x)$$ soit: $$\forall i\in\llb1,n\rrb\quad2(2-\lambda)x_i+2\sum_{k\neq i}x_k=1$$ La matrice augmentée de ce système est: $$\tilde{B}_r=\begin{pmatrix} 2(2-\lambda) & 2 & 2 & \ldots & 2 & 1\\ 2& 2(2-\lambda) & 2 & \ldots & 2 & 1\\ 2 & 2 & 2(2-\lambda) & & 2 & 1\\ \vdots & \vdots& & \ddots & \vdots & \vdots\\ 2 & 2 & \ldots &2 & 2(2-\lambda) & 1 \end{pmatrix}$$ On suppose d'abord $\lambda\neq 1$. Sous cette hypothèse, on peut résoudre le système par les mêmes opérations élémentaires sur les lignes et colonnes que pour le système de la question 3.(a) et on obtient de même que $x_1=\ldots=x_n$. On a alors $2(n+1-\lambda)x_1=1$ et donc nécessairement, pour qu'il existe une solution, $\lambda\neq n+1$. Alors: $$x_1=\ldots=x_n=\frac{1}{2(n+1-\lambda)}$$ La contrainte $g(x)=r^2$ donne alors la condition: $$\frac{n}{4(n+1-\lambda)^2}=r^2$$ soit: $$\lambda=\lambda_1=n+1+\frac{\sqrt{n}}{2r}\text{ ou }\lambda=\lambda_2=n+1-\frac{\sqrt{n}}{2r}$$ On remarque que $\lambda_1\neq n+1$ et $\lambda_2\neq n+1$. De même $\lambda_1\neq 1$. Par contre: $$\lambda_2=1\Longleftrightarrow r=\frac{1}{2\sqrt{n}}$$ Pour la valeur $r=r_0=\frac{1}{2\sqrt{n}}$, la valeur $\lambda_2$ est donc à éliminer. De plus, pour $\lambda=\lambda_1$, alors: $$x_1=\ldots=x_n=-\frac{r}{\sqrt{n}}\text{ et }f(x_1,\ldots,x_n)=(n+1)r^2+r\sqrt{n}$$ tandis que pour $\lambda=\lambda_2$, alors: $$x_1=\ldots=x_n=\frac{r}{\sqrt{n}}\text{ et }f(x_1,\ldots,x_n)=(n+1)r^2-r\sqrt{n}$$ Regardons maintenant ce qui se passe dans le cas où $\lambda=1$. Le système ne contient alors qu'une seule équation: $\sum_{k=1}^nx_k=\frac{1}{2}$. A cela s'ajoute la contrainte $\sum_{k=1}^nx_k^2=r^2$. S'il existe un $x=(x_1,\ldots,x_n)$ qui vérifie ces deux équations, on a donc nécessairement: $$f(x_1,\ldots,x_n)=r^2+\paf{1}{2}^2-\frac{1}{2}=r^2-\frac{1}{4}$$ A $r$ fixé, on peut remarquer que: $$r^2-\frac{1}{4}\leq (n+1)r^2-r\sqrt{n}\leq (n+1)r^2+r\sqrt{n}$$ avec égalité dans la première inégalité si et seulement si $r=r_0=\frac{1}{2\sqrt{n}}$. Il est donc essentiel de savoir s'il existe un $x=(x_1,\ldots,x_n)$ vérifiant les deux conditions: $$\sum_{k=1}^nx_k=\frac{1}{2}\quad\text{et}\quad\sum_{k=1}^nx_k^2=r^2$$ Or, par l'inégalité de Cauchy-Schwartz: $$\sum_{k=1}^nx_k\leq \sqrt{\sum_{k=1}^nx_k^2}\sqrt{\sum_{k=1}^n1}$$ Ainsi, si un $x$ vérifie les deux contraintes demandées, nécessairement: $$\frac{1}{2}\leq r\sqrt{n}\quad\text{soit}\quad r\geq \frac{1}{2\sqrt{n}}$$ Pour $r=r_0=\frac{1}{2\sqrt{n}}$, on remarque que $x=\pa{\frac{1}{2n},\ldots,\frac{1}{2n}}$ vérifie les deux conditions et par homothétie on pourra trouver un tel $x$ pour $r>r_0$. Comme déjà remarqué, à $r$ fixé: $$r^2-\frac{1}{4}\leq (n+1)r^2-r\sqrt{n}\leq (n+1)r^2+r\sqrt{n}$$ Comme l'on sait déjà que $m(r)$ et $M(r)$ existent, on peut affirmer à partir des calculs précédents que: $$\boxed{m(r)=\begin{cases} (n+1)r^2-r\sqrt{n}&{\rm si\ }0p if (position=0) or (position=8) then sens=(-1)*sens end; position=position+sens a=rand() end x=position; endfunction \end{verbatim} \end{comment} \newpage \exos Soit $a$ un réel de $]0,1[$ qu'on suppose inconnu. Soit $X$ une variable aléatoire réelle telle que~: \begin{itemize} \item[$\bullet$] $\mathbb{P}([X \leqslant a]) = \displaystyle \frac{1}{2}$. \item[$\bullet$] la loi conditionnelle de $X$ sachant $[X \leqslant a]$ est la loi uniforme sur $[0,a]$. \item[$\bullet$] la loi conditionnelle de $X$ sachant $[X > a]$ est la loi uniforme sur $[a,1]$. \end{itemize} \begin{enumerate} \item {\bf Question de cours :} \'Enoncer le théorème central limite. \item Montrer que la variable aléatoire $X$ est à densité, dont une densité $f_X$ est donnée par~: \[ \forall x \in \mathbb{R}, \ f_X(x) = \left\{ \begin{array}{cl} \displaystyle \frac{1}{2a} & \text{ si } 0 \leqslant x \leqslant a \medskip \\ \displaystyle \frac{1}{2(1-a)} & \text{ si } a < x \leqslant 1 \\ 0 & \text{ sinon } \end{array} \right. \] \item Montrer que $X$ admet une espérance et une variance et les calculer en fonction de $a$. \item \'Etablir que l'on a $\displaystyle \frac{1}{12} \leqslant \mathbb{V}[X] \leqslant \frac{1}{8}$. \item On considère $(X_i)_{i \geqslant 1}$ une suite de variables aléatoires mutuellement indépendantes, toutes de même loi que~$X$. \\ On note pour tout $n \geqslant 1$, $\overline{X_n} = \displaystyle \frac{1}{n} \sum\limits_{k=1}^{n} X_k$. \begin{enumerate} \item Déterminer deux réels $\beta$ et $\gamma$ ($\beta \neq 0$) tels que $T_n = \beta \overline{X_n} + \gamma$ soit un estimateur sans biais de $a$. \item Montrer que $T_n$ est un estimateur convergent de $a$. \item Soit $\alpha \in ]0,1[$. Montrer que $\displaystyle \left[ T_n - \frac{1}{\sqrt{2n \alpha}}, T_n + \displaystyle \frac{1}{\sqrt{2n\alpha}} \right]$ est un intervalle de confiance de $a$ au niveau de confiance $1-\alpha$. \item On pose pour tout $\alpha \in ]0,1[$, $u_\alpha = \Phi^{-1}\left( 1 - \frac{\alpha}{2} \right)$ où $\Phi$ désigne la fonction de répartition de la loi normale centrée réduite. Monter que l'intervalle $\displaystyle \left[ T_n - \frac{1}{\sqrt{2n}} u_\alpha , T_n + \frac{1}{\sqrt{2n}} u_\alpha \right]$ est un intervalle de confiance asymptotique de $a$ au niveau de confiance $1-\alpha$. \end{enumerate} \end{enumerate} \begin{comment} \begin{enumerate} \item Programme officiel ECS2 page 29. \item D'après la définition de $X$, on voit que $X(\Omega) \subset [0,1]$ presque-sûrement. \begin{itemize} \item[$\bullet$] Pour $x < 0$, $F_X(x) = 0$. \item[$\bullet$] Pour $x \geqslant 1$, $F_X(x)=1$. \item[$\bullet$] Soit $x \in [0,a]$. Alors~: \[ F_X(x) = \mathbb{P}(X \leqslant x) = \mathbb{P}([X \leqslant a] \cap [X \leqslant x]) = \mathbb{P}(X \leqslant a)\mathbb{P}_{[X \leqslant a]}(X \leqslant x) = \frac{1}{2} \cdot \frac{x}{a} = \frac{x}{2a} \] \item[$\bullet$] Soit $x \in [a,1]$. Alors~: \begin{align*} F_X(x) & = \mathbb{P}(X \leqslant x) = \mathbb{P}([X \leqslant a] \cup [a < X \leqslant x]) = \mathbb{P}(X \leqslant a) + \mathbb{P}( X > a] \cap [a < X \leqslant x]) \\ & = \frac{1}{2} +\mathbb{P}(X > a) \mathbb{P}_{[X > a]}(a < X \leqslant x) = \frac{1}{2} + \frac{1}{2} \cdot \frac{x-a}{1-a} = \frac{x+1-2a}{2(1-a)} \end{align*} \end{itemize} Ainsi, la fonction de répartition de $X$ est donnée par~: $ \forall x \in \mathbb{R}, \ F_X(x) = \left\{ \begin{array}{cl} 0 & \text{ si } x < 0 \\ \displaystyle \frac{x}{2a} & \text{ si } 0 \leqslant x \leqslant a \medskip \\ \displaystyle \frac{x+1-2a}{2(1-a)} & \text{ si } a \leqslant x \leqslant 1 \\ 1 & \text{ si } x > 1 \end{array} \right.$. \\ La fonction de répartition $F_X$ est clairement continue sur $\mathbb{R}$, de classe $\mathcal{C}^1$ au moins sur $\mathbb{R} \setminus \{0,a,1\}$ ce qui suffit, donc $X$ est bien une variable aléatoire à densité. On peut donner comme densité par exemple la fonction $f_X$ donnée par~: \[ \forall x \in \mathbb{R},\boxed{ \ f_X(x) = \left\{ \begin{array}{cl} \displaystyle \frac{1}{2a} & \text{ si } 0 \leqslant x \leqslant a \medskip \\ \displaystyle \frac{1}{2(1-a)} & \text{ si } a < x \leqslant 1 \\ 0 & \text{ sinon } \end{array} \right.} \] \item La variable aléatoire $X$ est bornée puisque $X(\Omega)=[0,1]$, donc $X$ admet bien une espérance et une variance. \[ \mathbb{E}[X] = \int_{0}^{1} t f_X(t) dt = \int_{0}^{a} \frac{t}{2a} dt + \int_{a}^{1} \frac{t}{2(1-a)} dt = \frac{1}{2a} \cdot \frac{a^2}{2} + \frac{1}{2(1-a)} \cdot \frac{1-a^2}{2} = \frac{a}{4} + \frac{1+a}{4} \boxed{= \frac{1+2a}{4}} \] Par théorème de transfert~: \[ \mathbb{E}[X^2] = \int_{0}^{1} t^2 f_X(t) dt = \int_{0}^{a} \frac{t^2}{2a} dt + \int_{a}^{1} \frac{t^2}{2(1-a)} dt = \frac{1}{2a} \cdot \frac{a^3}{3} + \frac{1}{2(1-a)} \cdot \frac{1-a^3}{3} = \frac{a^2}{6} + \frac{1+a+a^2}{6} \boxed{= \frac{1+a+2a^2}{6}} \] Donc~: \[ \hspace{-1cm} \mathbb{V}[X] = \mathbb{E}[X^2] - (\mathbb{E}[X] )^2 = \frac{1+a+2a^2}{6} - \frac{(1+2a)^2}{16} = \frac{8(1+a+2a^2) - 3(1+4a+4a^2)}{48} = \frac{5-4a+4a^2}{48} = \boxed{\frac{(2a-1)^2+4}{48} }\] \item D'après ce qui précède, on a~: \[ \mathbb{V}[X] = \frac{(2a-1)^2 +4}{48} \geqslant \frac{4}{48} = \frac{1}{12} \] et \[ \mathbb{V}[X]= \frac{(2a-1)^2+4}{48} < \frac{1+4}{48} \leqslant \frac{6}{48} = \frac{1}{8} \] \item \begin{enumerate} \item Par linéarité de l'espérance, \[ \mathbb{E}\left[ \overline{X_n} \right] = \frac{1}{n} \sum\limits_{k=1}^{n} \mathbb{E}[X_k] = \mathbb{E}[X_1] = \frac{1+2a}{4} \] On voit donc qu'en particulier $\mathbb{E}\left[ \frac{4 \overline{X_n}-1}{2} \right] = a$. \\ Il suffit donc de poser \fbox{$T_n = 2 \overline{X_n} - \displaystyle \frac{1}{2}$ pour obtenir un estimateur sans biais de $a$.} \item D'après la loi faible des grands nombres, $\overline{X_n}$ converge en probabilité vers $\frac{1+2a}{4}$\\ $x\mapsto \frac{4 x-1}{2} $ est continue donc \fbox{$T_n$ converge en probabilité vers $a$.}\\ \item \begin{align*} \mathbb{P}\left( T_n - \frac{1}{\sqrt{2n \alpha}} \leqslant a \leqslant T_n + \displaystyle \frac{1}{\sqrt{2n\alpha}} \right) & = \mathbb{P}\left( - \frac{1}{\sqrt{2n\alpha}} \leqslant T_n-a \leqslant \frac{1}{\sqrt{2n\alpha}} \right) \\ & = \mathbb{P}\left( \left| T_n - \mathbb{E}[T_n] \right| \leqslant \frac{1}{\sqrt{2n\alpha}} \right) \\ & = 1 - \mathbb{P}\left( \left| T_n - \mathbb{E}[T_n] \right| > \frac{1}{\sqrt{2n\alpha}} \right) \\ & \geqslant 1 - \frac{\mathbb{V}[T_n]}{ \left( \frac{1}{\sqrt{2n\alpha}} \right)^2} \\ & = 1 - 2n\alpha \mathbb{V}[T_n] \\ & = 1 - 2n \alpha \left( \frac{4}{n} \mathbb{V}[X_1] \right){\mbox (les }X_i\mbox{ sont indépendants)}\\ & = 1 - 8 \mathbb{V}[X_1] \alpha \\ & \geqslant 1-\alpha \end{align*} \fbox{$\displaystyle \left[ T_n - \frac{1}{\sqrt{2n \alpha}}, T_n + \displaystyle \frac{1}{\sqrt{2n\alpha}} \right]$ est un intervalle de confiance de $a$ au niveau de confiance $1-\alpha$.} \item En utilisant le théorème central limite $ \sqrt{n} \frac{\overline{X_n}-\frac{2a+1}{2}}{\sigma(X)}$ converge en loi vers $N\hookrightarrow N(0,1)$. Donc $$ \lim_{n\rightarrow +\infty } \P \left ( \sqrt{n} \frac{\overline{X_n}-\frac{2a+1}{4}}{\sigma(X)} \in \left [-u_\alpha, u_\alpha \right ] \right )= 1-\alpha$$ Or $\overline{X_n}=\frac{2T_n +1}{4}$ donc $$ \lim_{n\rightarrow +\infty } \P \left (\sqrt{n} \frac{T_n-a}{2\sigma(X)} \in \left [-u_\alpha, u_\alpha \right ] \right )= 1-\alpha$$ $$ \lim_{n\rightarrow +\infty } \P \left ( T_n-a \in \left [-u_\alpha \sqrt{\frac{4 \V(X)}{n}}, u_\alpha\sqrt{\frac{4\V(X)}{n}} \right ] \right )= 1-\alpha$$ Or $ \mathbb{V}[X] \leqslant \frac{1}{8}$ donc $$\P \left (T_n-a \in \left [-u_\alpha \sqrt{\frac{4\V(X)}{n}}, u_\alpha \sqrt{\frac{4\V(X)}{n}} \right ]\right ) \leq \P \left (T_n-a \in \left [-u_\alpha \sqrt{\frac{1}{2n}}, u_\alpha \sqrt{\frac{1}{2n}} \right ] \right )$$ \fbox{$ \left[ T_n - \frac{1}{\sqrt{2n}} u_\alpha , T_n + \frac{1}{\sqrt{2n}} u_\alpha \right]$ est un intervalle de confiance asymptotique de $a$ de niveau $1-\alpha$.} \end{enumerate} \end{enumerate} \end{comment} \newpage \Esps Soit $A\in \M_3(\R)$ une matrice non nulle telle que $A^2=0$.\\ Montrer que la matrice $A$ est semblable à la matrice $B = \begin{pmatrix} 0 & 0 & 1 \\ 0 & 0 & 0 \\ 0 & 0 & 0 \end{pmatrix}$ puis déterminer la dimension de l'espace vectoriel $\mathcal{C_A} = \{M \in \M_3(\R) \ , \ AM = MA\}$. \begin{comment} Notons $f$ l'endomorphisme de $\R^3$ canoniquement associé à $A$. On vérifie sans peine que $\mathcal{C}_A$ est un sous-espace vectoriel de $\M_3(\R)$ (en tant que noyau de l'endomorphisme $M \mapsto AM - MA$).\\ Puisque $A^2 = 0$, $f^2=0$, et ainsi $\im f \subset \Ker f$. Puisque $f\neq 0$ (car $A \neq 0$), le théorème du rang assure nécessairement que $\rg f = 1$ et $\dim \Ker f = 2$.\\ Soit $(e_1)$ une base de $\im f$. En ajoutant un vecteur $e_2 \in \Ker f \setminus \im f$, on obtient une famille libre donc une base de $\Ker f$. Soit $e_3$ un antécédent de $e_1$ par $f$. Puisque $e_1 \neq 0$, $e_3 \notin \Ker f$. La famille $\mathcal{B}= (e_1,e_2,e_3)$ est alors une famille libre de cardinal $3=\dim \R^3$ donc une base de $\R^3$. Dans cette base la matrice de $f$ est : \[ B = \begin{pmatrix} 0 & 0 & 1 \\ 0 & 0 & 0 \\ 0 & 0 & 0 \end{pmatrix} = P^{-1}AP \] en notant $P$ la matrice de passage de la base canonique à la base $\mathcal{B}$. \fbox{A et B sont semblables.} Remarquons que : \[ M \in \mathcal{C}_A \Leftrightarrow AM = MA \Leftrightarrow PBP^{-1}M = MPBP^{-1} \Leftrightarrow B\left(P^{-1}MP\right) = \left(P^{-1}MP\right)B \] Les matrices $N\in \M_3(\R)$ vérifiant $BN=NB$ sont les matrices de la forme \[ N = \begin{pmatrix} a & b & c \\ 0 & d & e \\ 0 & 0 & a \end{pmatrix} \] On en déduit une famille génératrice de $\mathcal{C}_A$ : \[ \mathcal{C}_A = \left\{ P\begin{pmatrix} a & b & c \\ 0 & d & e \\ 0 & 0 & a \end{pmatrix}P^{-1}, \ (a,b,c,d,e)\in \R^5\right\} = \Vect(M_1,M_2,M_3,M_4,M_5) \] où $M_1 = P\begin{pmatrix} 1 & 0 & 0 \\ 0 & 0 & 0 \\ 0 & 0 & 1 \end{pmatrix}P^{-1}$, $M_2 = P\begin{pmatrix} 0 & 1 & 0 \\ 0 & 0 & 0 \\ 0 & 0 & 0 \end{pmatrix}P^{-1}$, $M_3 = P\begin{pmatrix} 0 & 0 & 1 \\ 0 & 0 & 0 \\ 0 & 0 & 0 \end{pmatrix}P^{-1}$, $M_4 = P\begin{pmatrix} 0 & 0 & 0 \\ 0 & 1 & 0 \\ 0 & 0 & 0 \end{pmatrix}P^{-1}$ et $M_5 = P\begin{pmatrix} 0 & 0 & 0 \\ 0 & 0 & 1 \\ 0 & 0 & 0 \end{pmatrix}P^{-1}$. On vérifie sans problème que $(M_1,\dots,M_5)$ forme une famille libre, i.e. \fbox{$\dim \mathcal{C}_A = 5$.} \end{comment} \newpage \exos \be\item {\bf Question de cours } Rappeler la définition d'un produit scalaire sur un espace vectoriel réel $E$. \item Soient $l$ et $k$ deux entiers de $\llb1,n\rrb$. Soit $E_{l,k}$ la matrice de la base canonique de $M_n(\R)$ dont tous les coefficients sont nuls, sauf celui situé sur la $l$-ième ligne et la $k$-ième colonne qui vaut alors 1. Soit $A\in M_n(\R)$. Exprimer ${\rm Tr}(E_{l,k}A)$ en fonction d'un coefficient de la matrice A. \item Soit $n$ un entier supérieur ou égal à 2. Soit $F$ l'application bilinéaire symétrique définie par $$F:\fonc{M_n(\R)\times M_n(\R)}{\R}{(A,B)}{{\rm Tr}(A){\rm Tr}(B)-{\rm Tr}(AB)}.$$ \be \item Montrer que si, pour tout $X$ de $ M_n(\mathbb{R}) $ on a $F(A , X) = 0$, alors $A = 0$.\\ \item Soit $u$ une application de $M_n(\mathbb{R})$ vers $M_n(\mathbb{R})$ telle que : $$\forall(X,Y)\in (M_n(\mathbb{R}))^2,\quad F(u(X),Y) = F(X, u(Y)).$$ Montrer que $u$ est linéaire. \item L'application $F$ est-elle un produit scalaire sur $M_n(\mathbb{R})$ ? \ee \item Soit $M$ une matrice de $GL_n(\mathbb{R})$. Soit $f$ l'endomorphisme de $ M_n(\mathbb{R} ) $ défini par : $$f:\fonc{M_n(\R)}{M_n(\R)}{X}{ M X M^{-1}}.$$ On pose l'équation suivante pour $h$ un endomorphisme de $M_n(\R)$ : $$\forall(X,Y)\in (M_n(\mathbb{R}))^2,\quad F(f(X), Y) = F(X, h(Y))\quad (*)$$ \be\item Montrer que $f$ est bijective et expliciter $f^{-1}.$ \item Montrer que $f^{-1}$ est l'unique endomorphisme de $M_n(\R)$ vérifiant l'équation (*).\\ \item Montrer qu'une condition suffisante pour que $ f$ soit égal à $f^{-1}$ est que la matrice $M$ soit diagonalisable et que $\Sp(M) \subset \{ 1 ; - 1\} $. Cette condition est-elle nécessaire ?\\ \ee \item Soit l'endomorphisme $g$ de $M_2(\mathbb{R}) $ défini par: $$g:\fonc{ M_n(\mathbb{R})}{M_2(\mathbb{R})}{ \begin{pmatrix}a & b \\ c & d \\\end{pmatrix}}{\begin{pmatrix}a+c & b+d-a-c \\ c & d-c\end{pmatrix}}$$ \be\item Déterminer $M\in M_2(\R)$ telle que $\forall A\in M_2(\R)$, $g(A)M=MA$. \item Déterminer tous les endomorphismes $k$ de $M_2(\mathbb{R}) $ tel que, pour tout $X$ et $Y$ de $M_2(\mathbb{R}) $ on ait : $$Tr(g(X)) Tr(Y) - Tr(X) Tr(k(Y)) = - Tr(k(Y)X) + Tr(Yg(X)).$$ \ee\ee \begin{comment} \be\item Programme officiel ECS2 page 7. \item On note $(a_{i,j})_{1\leq i,j\leq n}$ et $(x_{i,j})_{1\leq i,j\leq n}$ les coefficients respectifs des matrices $A$ et $E_{l,k}$ Avec la formule du produit matriciel : Tr($A.E_{l,k})=\sum_{i=1}^{n} ( \sum_{j=1}^{n} a_{i, j} x_{j,i})$ Mais $x_{j,i}=0$ seulement si $j=l$ et $i=k$, donc \fbox{Tr($A.E_{l,k})=a_{k,l}$} \item \be\item Soit A telle que pour tout $X\in M_n(\R)$, $F(A,X)=0$ En prenant $X=E_{l,k}$ on obtient Si $l=k$ : ${\rm Tr}(A)\times 1-a_{l,l}=0$ et si $l\neq k$, $0-a_{k,l}=0$. Les $a_{l,l}$ sont tous égaux, on doit avoir ${\rm Tr}(A)=n{\rm Tr}(A)$ ce qui donne ${\rm Tr}(A)=0$. \fbox{Ainsi A=0.} \item Soient $\alpha$ et $\beta$ réels et des matrices $X$, $Y$ et $T$ de $ M_n(\mathbb{R}) $ on a: \begin{eqnarray*} F( u(\alpha X + \beta Y) - \alpha u(X) -\beta u( Y) , T)& =& F( u(\alpha X + \beta Y), T) - F(u(\alpha X ) , T) - F( u(\beta Y) , T)\\ & =& F( \alpha X + \beta Y , u(T) - F(\alpha X , u(T))) - F(\beta Y , u(T) )\\ &=& F( \alpha X + \beta Y - \alpha X - \beta Y , u(T)) \\ &=& F( 0 , u(T)) = 0 \end{eqnarray*} Comme cette propriété est vraie pour tout $T$ , d'après a) on a $u(\alpha X + \beta Y) - \alpha u(X) -\beta u( Y) = 0$ \fbox{l'application $u$ est linéaire.}\\ \item Non : en effet si on note \fbox{$C =\begin{pmatrix} 0 & 1 & 0 &\cdots & 0 \\ 1 & 0 & 0 &\cdots&0\\ 0 & 0 & 0& \cdots &0\\ \vdots&\vdots&\vdots&&\vdots\\ 0 & 0 & 0& \cdots &0\end{pmatrix}$, $C^2= \begin{pmatrix} 1 & 0 & 0 &\cdots & 0 \\ 0& 1 & 0 &\cdots&0\\ 0 & 0 & 0& \cdots &0\\ \vdots&\vdots&\vdots&&\vdots\\ 0 & 0 & 0& \cdots &0\end{pmatrix}$} Et $F(C,C)={\rm Tr}^2(C)-{\rm Tr}(C^2)=-1<0$ \fbox{F n'est pas un produit scalaire.} \ee \item \be\item Soit $Y\in M_n(\R)$, $f(X)=Y$ ssi $MXM^{-1}=Y$ ssi $X=M^{-1}YM$. \fbox{Pour tout $Y\in M_n(\R)$, $f^{-1}(Y) = M^{-1} Y M$} \item \begin{itemize} \item[*]Montrons que $f^{-1}$ vérifie (*) ie ${\rm Tr}(MXM^{-1}Y)-{\rm Tr}(f(X)){\rm Tr}(Y)= {\rm Tr}(XM^{-1}YM)-{\rm Tr}(X){\rm Tr}(f^{-1}Y).$ On a déjà ${\rm Tr}(X)={\rm Tr}(f(X))$ et ${\rm Tr}(f^{-1}(Y))={\rm Tr}(Y)$. De plus ${\rm Tr}(M.XM^{-1}Y)={\rm Tr}(XM^{-1}Y.M)$, on a bien l'identité voulue. Ainsi $\forall(X,Y)\in (M_n(\mathbb{R})^2,\quad F(f(X), Y) = F(X, f^{-1}(Y))$ et $f^{-1}$ vérifie (*) \item[*] Supposons qu'il y ait un autre endomorphisme $h$ de $ M_n(\mathbb{R}) $ ; alors, pour tout $X$ tout $Y$ de $ M_n(\mathbb{R}) $ on aurait $F(X, h(Y)) = F(X, f^{-1}(Y))$ d'où pour tout $X$ de $ M_n(\mathbb{R} ) $ on aurait $F(X, h(Y) - f^{-1}(Y)) = 0$ d'où d'après 1) a) pour tout $Y$ de $ M_n(\mathbb{R}) ) $ on a: $ h(Y) - f^{-1}(Y) = 0$ d'où $h = f^{-1}$. \fbox{$f^{-1}$ est l'unique endomorphisme de $M_n(\R)$ vérifiant la relation demandée} \ei \item Si la matrice $M$ est semblable à une matrice diagonale $D$ ne contenant que des $1$ ou des $-1$ sur la diagonale on a $D^2 =I$ et par suite $ M^2 = I$ donc $M = M^{-1}$ et $h=f$ \fbox{La condition est bien suffisante}\\ Cette condition \fbox{n'est pas nécessaire} car si on choisit $M = 2 I$ on a $h = f$. \ee \item \be \item On détermine cette matrice $M=\begin{pmatrix}\alpha&\beta\\\gamma&\delta\end{pmatrix}$ en écrivant l'égalité $Mg(A)=Ag(M)$ pour des matrices de la base canonique. Pour $E_{1,1}$ ; $\begin{pmatrix}\alpha&0\\\gamma&0\end{pmatrix}= \begin{pmatrix}\alpha-\gamma&\beta-\delta\\0&0\end{pmatrix}.\quad$ Pour $E_{1,2}$ $\begin{pmatrix}0&\alpha\\0&\gamma\end{pmatrix}= \begin{pmatrix}\gamma&\delta\\0&0\end{pmatrix}$ On trouve ainsi $\alpha=\beta=\delta$ et $\gamma=0$ \fbox{ On pose alors $M=\begin{pmatrix}1&1\\0&1\end{pmatrix}$} et l'on trouve bien : $M \left( \begin{array}{cc} a & b \\ c & d \\ \end{array} \right) = \begin{pmatrix}a+c&b+d\\c&d\end{pmatrix}=\left( \begin{array}{cc} a+c & b+d-a-c \\ c & d-c \\ \end{array} \right) M$. (en fait tous les multiples non nuls de M conviennent) \item La matrice M trouvée est inversible, donc l'endomorphisme $g$ est de la forme $\displaystyle g(X) = MXM^{-1}$ avec $\displaystyle M = \left( \begin{array}{cc} 1 & 1 \\ 0 & 1 \\ \end{array} \right)$ Alors d'après 4. on sait que l'endomorphisme $k$ existe et est unique ; il est défini par: $\displaystyle k(X) = M^{-1}XM$ c'est à dire puisque $\displaystyle M^{-1} = \left( \begin{array}{cc} 1 & -1 \\ 0 & 1 \\ \end{array} \right)$ on a: \fbox{$\displaystyle k(\left( \begin{array}{cc} a & b \\ c & d \\ \end{array} \right)) = \left( \begin{array}{cc} a-c & a-c+b-d \\ c & c+d \\ \end{array} \right)$.} \ee\ee \end{comment} \newpage \Esps On dit qu'une variable aléatoire réelle $X$ définie sur un espace probabilisé $(\Omega,\cal A,\P)$ suit la distribution de {\sc Lévy}--{\sc Pareto} si~: \begin{itemize} \item Il existe $\varepsilon>0$ tel que \begin{itemize} \item $\P(X>\varepsilon)=1$ ; \item pour tout $\eta>\varepsilon$, $\P(X>\eta)>0$. \end{itemize} \item pour tous $\eta_1$, $\eta_2>\varepsilon$, la loi de $\frac{X}{\eta_1}$ conditionnellement à l'événement $\acco{X>\eta_1}$ est la même que celle de $\frac{X}{\eta_2}$ conditionnellement à l'événement $\acco{X>\eta_2}$, ce qui équivaut à dire que $$ \forall x\in\R,\, \P\left(\frac{X}{\eta_1}>x | X > \eta_1\right) = \P\left(\frac{X}{\eta_2}>x| X > \eta_2\right) \mathpunct. $$ \end{itemize} Déterminer la forme de la densité d'une telle variable aléatoire. \begin{comment} % sa correction Il suffit de voir que l'hypothèse implique que $Z=\ln\frac{X}{\varepsilon}$ possède la propriété d'absence de mémoire (ECS1 p.24) et donc que $Z\hookrightarrow {\mathcal E}(\lambda)$ pour un certain $\lambda>0$. On détermine la densité cherchée en déterminant la fonction de répartition de $x$ (et en la dérivant à la fin)~: Soit $x>\varepsilon$ (et donc $\ln\frac{x}{\varepsilon}>0$), on a \begin{eqnarray*} \P(X \leq x) &=& \P\left(\ln\frac{X}{\varepsilon}\leq \ln\frac{x}{\varepsilon}\right)\\ &=& 1-\exp(-\lambda.\ln\frac{x}{\varepsilon}) = 1-\left(\frac{x}{\varepsilon}\right)^{-\lambda} \end{eqnarray*} On obtient par dérivation (la f.r. obtenue, valant $0$ sur $]-\infty,\varepsilon[$ est bien continue sur tout $\R$, de classe ${\mathcal C}^1$ sur $\R\setminus\acco{\varepsilon}$) la densité de $X$~: $$ \boxed{\delta_X(x) = \lambda.\frac{\varepsilon^{\lambda}}{x^{\lambda+1}}\1{]\varepsilon,+\infty[}(x)} \mathpunct. $$ De là, on peut enchainer sur des questions d'existence d'espérance et de variance, de lois du min, du max, etc... \end{comment} \newpage \exos On se place dans un espace euclidien $E$ de dimension $n\geq 1$. \be \item {\bf Question de cours :} réduction des matrices symétriques réelles. \item Soit $(e_1,\ldots,e_n)$ une base orthonormée de $E$. Montrer que: $$\forall x\in E\quad \norme{x}^2=\sum_{k=1}^n\prodscal{x}{e_k}^2.$$ \item On s'intéresse maintenant à la réciproque du résultat précédent. Soit donc une famille de vecteurs $(e_1,\ldots,e_m)$ de $E$ telle que: $$\forall x\in E\quad \norme{x}^2=\sum_{k=1}^m\prodscal{x}{e_k}^2.$$ On suppose en outre que les vecteurs $e_1,\ldots,e_m$ sont tous de norme 1. \be \item Montrer que la famille $(e_1,\ldots,e_m)$ est une famille orthonormée de $E$. \item Déterminer $\pa{{\rm Vect}(e_1,\ldots,e_m)}^\perp$ et conclure. \ee \item On considère désormais une famille de $n$ vecteurs $(e_1,\ldots,e_n)$ de $E$ (où $n=\dim E$) telle que: $$\forall x\in E\quad \norme{x}^2=\sum_{k=1}^n\prodscal{x}{e_k}^2.$$ On ne suppose plus que les vecteurs $e_1,\ldots,e_n$ sont de norme 1. \be \item Pourquoi la famille $(e_1,\ldots,e_n)$ est-elle encore une base de $E$ avec ces hypothèses ? \item Montrer que, pour tous $x$ et $y$ dans $E$: $$\prodscal{x}{y}=\sum_{k=1}^n\prodscal{x}{e_k}\prodscal{y}{e_k}.$$ \item Soit $G$ la matrice de $\M_n(\R)$ définie par: $$G=\begin{pmatrix} \prodscal{e_1}{e_1} & \prodscal{e_1}{e_2}&\ldots&\prodscal{e_1}{e_n} \\ \prodscal{e_2}{e_1} & \prodscal{e_2}{e_2}&\ldots&\prodscal{e_2}{e_n} \\ \vdots& \vdots&\ddots & \vdots\\ \prodscal{e_n}{e_1} & \prodscal{e_n}{e_2}&\ldots&\prodscal{e_n}{e_n} \\ \end{pmatrix}.$$ Autrement dit: $G=(\prodscal{e_i}{e_j})_{1\leq i,j\leq n}$. Pourquoi la matrice $G$ est-elle diagonalisable dans $\R$ ? \item Prouver que $G^2=G$. En déduire que $G=I_n$ et conclure. \ee \ee %------------------------------------------------------------------------------------------------------------------ \begin{comment} \be \item Question de cours: programme ECS2 2013 p. 18. \item C'est encore un résultat du cours. Si la base $(e_1,\ldots,e_n)$ est orthonormée, le vecteur $x$ se décompose sous la forme: $$x=\sum_{k=1}^n\prodscal{x}{e_k}e_k$$ d'où par bilinéarité du produit scalaire: $$\norme{x}^2=\prodscal{x}{x}=\sum_{i=1}^n\sum_{j=1}^n\prodscal{x}{e_i}\prodscal{x}{e_j}\prodscal{e_i}{e_j}\boxed{=\sum_{k=1}^n\prodscal{x}{e_k}^2}$$ car la famille $(e_1,\ldots,e_n)$ est orthonormée. \item \be \item On applique l"hypothèse avec $x=e_i$ pour $1\leq i\leq m$. On obtient ainsi: $$1=\norme{e_i}^2=\sum_{k=1}^m\prodscal{e_i}{e_k}^2=1+\sum_{k\neq i}\prodscal{e_i}{e_k}^2$$ Ainsi: $\sum_{k\neq i}\prodscal{e_i}{e_k}^2=0$. Mais c'est une somme de termes positifs donc chaque terme est nul: $$\forall (i,k)\in\llb 1,m\rrb\quad i\neq k\Rightarrow \prodscal{e_i}{e_k}=0$$ Les vecteurs étant unitaires, \fbox{la famille est donc bien orthonormée.} \item Soit $x\in\pa{{\rm Vect}(e_1,\ldots,e_m)}^\perp$. Alors, pour tout $k\in\llb 1, m\rrb$, $\prodscal{x}{e_k}=0$ d'où : $$\norme{x}^2=\sum_{k=1}^m\prodscal{x}{e_k}^2=0$$ Donc $x=0$. Ainsi : $$\pa{{\rm Vect}(e_1,\ldots,e_m)}^\perp=\acco{0}$$ Mais alors ${\rm Vect}(e_1,\ldots,e_m)=E$ et donc la famille $(e_1,\ldots,e_m)$ est génératrice. Elle est libre car orthonormée et c'est donc bien \fbox{une base orthonormée de $E$.} En particulier, cela prouve que $m=n$. \ee \item \be \item L'argument utilisé à la question 3.(b) pour montrer que la famille $(e_1,\ldots,e_n)$ est génératrice reste valable. De plus, comme on sait désormais que le cardinal de la famille est égal à la dimension de $E$, on peut affirmer que \fbox{la famille $(e_1,\dots,e_n)$ est une base de $E$. } \item D'une part: $$\norme{x+y}^2=\norme{x}^2+2\prodscal{x}{y}+\norme{y}^2$$ et d'autre part, en utilisant l'hypothèse sur la famille $(e_1,\ldots,e_n)$ on peut également calculer: \begin{eqnarray*} \norme{x+y}^2& = & \sum_{k=1}^n\prodscal{x+y}{e_k}^2\\ &=& \sum_{k=1}^n\pa{\prodscal{x}{e_k}+\prodscal{y}{e_k}}^2\\ & = & \sum_{k=1}^n\prodscal{x}{e_k}^2+2\sum_{k=1}^n\prodscal{x}{e_k}\prodscal{y}{e_k}+ \sum_{k=1}^n\prodscal{y}{e_k}^2\\ & = & \norme{x}^2+2\sum_{k=1}^n\prodscal{x}{e_k}\prodscal{y}{e_k}+\norme{y}^2 \end{eqnarray*} En comparant les deux expressions on en déduit: $$\boxed{\prodscal{x}{y}=\sum_{k=1}^n\prodscal{x}{e_k}\prodscal{y}{e_k}}$$ \item La matrice $G$ est diagonalisable dans $\R$ car c'est une matrice \fbox{symétrique réelle. } \item On note $A=G^2=(a_{i,j})_{1\leq i,j\leq n}$. Alors par définition du produit matriciel, pour $(i,j)\in \llb 1,n\rrb$: $$a_{i,j}=\sum_{k=1}^n\prodscal{e_i}{e_k}\prodscal{e_k}{e_j}=\sum_{k=1}^n\prodscal{e_i}{e_k}\prodscal{e_j}{e_k}=\prodscal{e_i}{e_j}$$ par la question 4.(b). Mais cela signifie exactement que \fbox{$G^2=G$.} La matrice $G$ définit donc un projecteur. Montrons que $G$ est inversible. Il suffit pour cela de montrer que ses colonnes forment une famille libre. On note $C_1,\ldots,C_n$ les colonnes de $G$. Soient $\lambda_1,\ldots,\lambda_n$ des scalaires tels que $\sum_{j=1}^n\lambda_jC_j=0$. Alors, pour tout $i\in\llb 1, n\rrb$: $$\sum_{j=1}^n\lambda_j\prodscal{e_j}{e_i}=0$$ soit encore par bilinéarité du produit scalaire: $$\prodscal{\sum_{j=1}^n\lambda_je_j}{e_i}=0$$ Mais alors cela signifie que $\sum_{j=1}^n\lambda_je_j\in\pa{{\rm Vect}(e_1,\ldots,e_n)}^\perp=\acco{0}$ et donc $\sum_{j=1}^n\lambda_je_j=0$. Mais alors, par liberté de la famille $(e_1,\ldots,e_n)$, on obtient $\lambda_1=\ldots=\lambda_n=0$. Cela prouve que la matrice $G$ est inversible. En multipliant l'égalité $G^2=G$ par $G^{-1}$ on obtient alors $G=I_n$. Mais cela signifie exactement que $\prodscal{e_i}{e_j}=0$ si $i\neq j$ et que $\prodscal{e_i}{e_i}=1$: \fbox{la base $(e_1,\ldots,e_n)$ est orthonormée.} \ee \ee \end{comment} \newpage \Esps \begin{verbatim} function y=g(x) X=grand(1,10000,"unf",0,1/2); Y=grand(1,10000,"unf",0,1/3); S=0; for k=1:10000 if X(k)+Y(k)<=x then S=S+1; end end y=S/10000; endfunction function graph2() X=linspace(-0.1,1,101); fplot2d(X,g) endfunction \end{verbatim} \be \item Commentez les fonctions Scilab. \item Dessiner le graphe de sortie la fonction {\bf graph2} \ee \begin{comment} \begin{enumerate} \item $g$ représente la fonction de répartition de $X+Y$ où $X\hookrightarrow U([0,\frac{1}{2}])$ et $Y\hookrightarrow U([0,\frac{1}{3}])$. \item $X$ et $Y$ sont indépendantes. Posons $Z=X+Y$, le produit de convolution donne une densité de $Z$ $$h(z)=6\int_0^{+\infty} \mathds{1}_{0\leq t\leq 0.5} \mathds{1}_{0\leq z-t\leq \frac{1}{3}} \dt=6\int_{0}^{\frac12} \mathds{1}_{z-\frac{1}{3}\leq t\leq z}\dt $$ $z-\frac13\geq 0$ ssi $z\geq \frac13$ et $z-\frac13\geq \frac12$ ssi $z\geq \frac56$ si $z<0$ ou $z>\frac56$, $h(z)=0$,\\ si $z\leq \frac{1}{3}$, $h(z)=6\int_0^z\dt{=6z}$,\\ si $ \frac{1}{3} \frac56$, $H(z)=1$.\\ \end{enumerate} \includegraphics{Fig22.png} \end{comment} \newpage \exos Pour tout réel $t$ de $[0,1]$, on définit la fonction $f_t$ sur $]0,+\infty[$ par~: \[ \forall x \in ]0,+\infty[, \quad f_t(x) = \ln(x)-\ln(x+t) + \frac{1}{x}. \] \begin{enumerate} \item {\bf Question de cours :} Convergence en loi d'une suite de variables aléatoires à densité. \item \begin{enumerate} \item Soit $t \in [0,1]$. \\ Montrer que l'équation $f_t(x)=1$, d'inconnue $x \in ]0,+\infty[$, admet une unique solution dans $]0,+\infty[$ que l'on note $\varphi(t)$. \item Calculer $\varphi(0)$, et montrer que $\frac13<\varphi(1)<\frac12$. \item Montrer que $\varphi$ est strictement décroissante sur $[0,1]$. \end{enumerate} \item Pour tout réel $t$ de $[0,1]$, on note $g_t$ la fonction définie sur $\mathbb{R}$ par~: \[ \forall x \in \mathbb{R}, \quad g_t(x) = \left\{ \begin{array}{ll} 0 & \text{ si } x < \varphi(t) \\ \displaystyle \frac{t+x-tx}{x^2(x+t)} & \text{ si } x \geqslant \varphi(t) \end{array} \right. \] Montrer que $g_t$ est une densité de probabilité. \item Soit $t$ un réel de $[0,1]$. On considère dans la suite une variable aléatoire $X_t$ admettant $g_t$ pour densité. \\ Les questions (a), (b) et (c) sont indépendantes. \begin{enumerate} \item On admettra provisoirement que $\varphi$ est continue en 0. On note pour tout $n \geqslant 1$, $Z_n = X_{1/n}$. Montrer que la suite de variables aléatoires $(Z_n)_{n \geqslant 1}$ converge en loi vers une variable aléatoire dont on précisera la loi. \item Montrer que $X_t$ admet une espérance si et seulement si $t=1$, et vérifier que~: $\mathbb{E}[X_1] = \displaystyle \frac{1}{\varphi(1)} - 1$. \item On note $Y_t = f_t(X_t)$. Montrer que $Y_t$ suit une loi uniforme sur un intervalle $[a,b[$, où $a$ et $b$ sont deux réels à préciser. \end{enumerate} \item Montrer que $\forall t\in[0;1[$, $\varphi(t)\geq 1-t$. En déduire que $\varphi$ est continue en 0. \end{enumerate} \begin{comment} \begin{enumerate} \item Programme officiel ECS2 page 22 \item \begin{enumerate} \item Soit $t \in [0,1]$. La fonction $f_t$ est dérivable sur $]0,+\infty[$. De plus, \[ \forall x > 0, \ f_t'(x) = \frac{1}{x} - \frac{1}{x+t} - \frac{1}{x^2} = \frac{x(x+t)-x^2-x-t}{x^2(x+t)} = \frac{x(t-1)-t}{x^2(x+t)} \] Comme $t-1 \leqslant 0$ et $-t \leqslant 0$, on a $\forall x > 0, f_t'(x) \leqslant 0$ et $f_t'$ ne s'annule jamais. (si $x(t-1)+ (-t)=0$, on a nécessairement (tout est négatif) $x(t-1)=-t=0$, donc $t=1$ et $t=0$, absurde). \\ Ainsi $f_t$ est strictement décroissante sur $]0,+\infty[$, et continue. Donc $f_t$ réalise une bijection de $]0,+\infty[$ vers $f_t(]0,+\infty[) = ]\lim\limits_{+\infty} f_t, \lim\limits_{0^+} f_t[$. Or, en $0^+$, $f_t(x) = \frac{x\ln(x)+1}{x} - \ln(x+t) \underset{x \rightarrow0^+}{\longrightarrow} +\infty$ car $x\ln(x) \underset{x \rightarrow 0^+}{\longrightarrow} 0$ par croissance comparée. \\ En $+\infty$, $f_t(x) = \displaystyle \ln\left( \frac{x}{x+t} \right) + \frac{1}{x} = \ln\left( \frac{1}{1+\frac{t}{x}} \right) + \frac{1}{x} \underset{x \rightarrow+\infty}{\longrightarrow} 0$. Finalement~: $1 \in ]0,+\infty[=f_t(]0,+\infty[)$, donc \fbox{il existe un unique réel $\varphi(t) \in ]0,+\infty[$ tel que $f_t(\varphi(t))=1$.} \item $f_0(x) = \displaystyle \frac{1}{x}$, donc $f_0(x) = 1 \Longleftrightarrow x=1$~: on a donc $\varphi(0)=1$. $f_1(x) = \displaystyle \frac{1}{x} + \ln\left( \frac{x}{x+1} \right)$. \\ On a $f_1\left( \frac{1}{2} \right) = 2 + \ln\left( \frac{1/2}{3/2} \right) = 2 - \ln(3) < 1$ (car $\ln(3) > \ln(e) > 1$). \\ Par ailleurs $f_1\left(\frac{1}{3} \right) = 3 + \ln\left( \frac{1/3}{4/3} \right) = 3 - \ln(4) = 3 - 2 \ln(2) > 1$ (car $\ln(2) < \ln(e) < 1$). \\ Ainsi~: \[ f_1 \left( \frac{1}{2} \right) < f_1(\varphi(1)) < f_1\left( \frac{1}{3} \right) \overset{f_1 \ \searrow}{\Longrightarrow} \boxed{\frac{1}{3} < \varphi(1) < \frac{1}{2}} \] \item Fixons $0 \leqslant a < b \leqslant 1$. \\ Alors~: \[ f_a(\varphi(b)) = \ln(\varphi(b)) - \ln(\varphi(b)+a) + \frac{1}{\varphi(b)} = \left( 1 + \ln(\varphi(b)+b) \right) - \ln(\varphi(b) + a) = 1 + \ln\left( \frac{\varphi(b)+b}{\varphi(b)+a} \right) > 1 \] Ainsi, $f_a(\varphi(b)) > f_a(\varphi(a))$, et $f_a$ est strictement décroissante, donc nécessairement $\varphi(b) < \varphi(a)$. \\ La fonction \fbox{ $\varphi$ est donc strictement décroissante sur $[0,1]$.} \end{enumerate} \item %Pour tout réel $t$ de $]-\infty,1]$, on note $g_t$ la fonction définie sur $\mathbb{R}$ par~: %\[ \forall x \in \mathbb{R}, \quad g_t(x) = \left\{ \begin{array}{ll} 0 & \text{ si } x < \varphi(t) \\ \displaystyle \frac{t+x-tx}{x^2(x+t)} & \text{ si } x \geqslant \varphi(t) \end{array} \right. \] %Montrer que $g_t$ est une densité de probabilité. La fonction $g_t$ est clairement continue au moins sur $\mathbb{R} \setminus \{\varphi(t) \}$. \\ De plus, comme $t \in [0,1]$, on a $\forall x \geqslant \varphi(t), \ t+x-tx = x(1-t) + t \geqslant 0$. \\ La fonction $g_t$ est donc bien positive. \\ De plus, \[ \forall A > 0, \ \int_{\varphi(t)}^{A} g_t(x) \text{d} x = \int_{\varphi(t)}^{A} (-f_t'(x)) \text{d}x = \bigg[ - f_t(x) \bigg]_{\varphi(t)}^{A} = 1 - f_t(A) \underset{A \rightarrow+\infty}{\longrightarrow} 1 \] Ainsi, l'intégrale $\displaystyle \int_{-\infty}^{+\infty} g_t(x) \text{d}x= \int_{\varphi(t)}^{+\infty} g_t(x) \text{d}x$ converge et vaut $1$. \\ \fbox{La fonction $g_t$ est donc bien une densité de probabilité.} \item %Soit $X_t$ une variable aléatoire admettant $g_t$ pour densité. \\ %Montrer que $X_t$ admet une espérance si et seulement si $t=1$, et calculer $\mathbb{E}[X_1]$. \begin{enumerate} \item On note pour tout $n \geqslant 1$, $Z_n = X_{1/n}$. Pour tout $n \geqslant 1$, on a~: \[ \forall x \in \mathbb{R}, \ F_{Z_n}(x) = \left\{ \begin{array}{cl} 0 & \text{ si } x < \varphi\left( \frac{1}{n} \right) \\ 1-f_{1/n}(x) & \text{ si } x \geqslant \varphi\left( \frac{1}{n} \right) \end{array} \right. = \left\{ \begin{array}{cl} 0 & \text{ si } x < \varphi\left( \frac{1}{n} \right) \\ 1-\frac{1}{x} - \ln(x) + \ln\left( x + \frac{1}{n} \right) & \text{ si } x \geqslant \varphi\left( \frac{1}{n} \right) \end{array} \right. \] Comme $\varphi$ est décroissante et $\varphi(0)=1$, pour tout $n\in\N^*$, $\varphi\paf{1}{n}<1$ donc : pour $x < 1$, alors comme $\lim_{n\rightarrow +\infty}\varphi\paf1n=1$, pour $n$ assez grand, $x<\varphi\paf1n$ et donc $F_{Z_n}(x)=0 \underset{n \rightarrow+\infty}{\longrightarrow} 0$. \\ Pour $x \geq 1$, on a pour tout $n\in\N^*$, $x\geq 1\geq \varphi\paf1n$ et $F_{Z_n}(x) = 1-\frac{1}{x} - \ln(x) + \ln\left( x + \frac{1}{n} \right) \underset{n \rightarrow+\infty}{\longrightarrow} 1 - \frac{1}{x}$. La fonction $x \mapsto \left\{ \begin{array}{ll} 0 & \text{ si } x \leq 1 \\ 1 - \frac{1}{x} & \text{ si } x > 1 \end{array} \right.$ est continue, $\mathcal{C}^1$ sur $\mathbb{R} \setminus \{1\}$, croissante, de limites $0$ en $-\infty$ et $1$ en~$+\infty$~: c'est donc la fonction de répartition d'une variable aléatoire $Z$ et \fbox{$(Z_n)$ converge en loi vers $Z$.} \item \[ \mathbb{E}[X_t] <+\infty \Longleftrightarrow \int_{}^{+\infty} x g_t(x) \text{d} x \ \text{converge (absolument)} \] Or, \[ x g_t(x) = \frac{x((1-t) x + t) }{x^2(x+t)} \underset{x \rightarrow+\infty}{\sim} \left\{ \begin{array}{ll} \frac{(1-t)x^2}{x^3} = \frac{1-t}{x} & \text{ si } t \neq 1 \\ \frac{tx}{x^3} = \frac{t}{x^2} & \text{ si } t =1 \end{array} \right. \] Par critère d'équivalence de fonctions positives, par comparaison avec les intégrales de Riemann, on a donc bien que~: \[ \boxed{\mathbb{E}[X_t] \ \text{existe} \ \Longleftrightarrow t=1} \] Dans le cas où $t=1$, on a~: \[ \mathbb{E}[X_1] = \int_{\varphi(1)}^{+\infty} \frac{1}{x(x+1)} dx = \lim\limits_{A \rightarrow+\infty} \left( \bigg[ \ln(x) - \ln(x+1) \bigg]_{\varphi(1)}^{A} \right) = 0 - (\ln(\varphi(1))-\ln(\varphi(1)+1) \boxed{= \frac{1}{\varphi(1)} - 1} \] \item %On note $Y_t = f_t(X_t)$. Vérifier que $Y_t$ suit une loi uniforme sur un intervalle $[a,b]$, où $a$ et $b$ sont deux réels à préciser. Vu l'expression de $g_t$, on voit déjà que $X_t(\Omega)=[\varphi(t),+\infty[$. \\ La fonction de répartition de $X_t$ est alors donnée par~: \[ \forall x \in \mathbb{R}, \ F_{X_t}(x) = \left\{ \begin{array}{cl} 0 & \text{ si } x < \varphi(t) \\ 1-f_t(x) & \text{ si } x \geqslant \varphi(t) \end{array} \right. \] Comme $f_t([\varphi(t),+\infty[) = ]0,1]$, on a donc $Y_t(\Omega)=]0,1]$. \\ De plus, \[ \forall x \in ]0,1], \ F_{Y_t}(x) = \mathbb{P}(Y_t \leqslant x) = \mathbb{P}(f_t(X_t) \leqslant x) = \mathbb{P}(X_t \geqslant f_t^{-1}(x)) = 1 - F_{X_t}(f_t^{-1}(x)) = 1 - (1 - x) = x \] (la variable $X_t$ étant à densité, $\P((X_t=f_t^{-1}(x))=0$) \fbox{Ainsi, $Y_t$ suit bien une loi uniforme sur $]0,1]$.} \end{enumerate} \item Soit $t\in[0,1[$. $\varphi(t)\geq 1-t$ ssi $f_t(\varphi(t))\leq f_t(1-t)$ ssi $1\leq \ln(1-t)+\frac{1}{1-t}$ On définit sur $[0;1[$ $x\mapsto \ln(1-x)+\frac{1}{1-x}$, $g'(x)=\frac{-1}{1-x}+\frac{1}{(1-x)^2}=\frac{x}{(1+x)^2}.$ $g$ est bien croissante sur $[0;1[$ et $g(0)=1$, donc en particulier $g(t)\geq 1$ et $1\geq \varphi(t)\geq 1-t$. Par encadrement $\lim_{t\rightarrow 0}\varphi(t)=1=\varphi(0)$. \fbox{$\varphi$ est continue en 0.} \end{enumerate} \end{comment} \newpage \Esps Soit $a$ un réel strictement positif. \be\item Étudier la nature de la série $\sum_{n\in \N} \left(\arctan (n+a) - \arctan n\right)$. \item Proposer un programme SCILAB permettant d'en donner une valeur approchée à 0.001 près quand $a=\frac12$ On rappelle que {\tt atan(x)} renvoie la valeur de $\arctan(x)$ \ee \begin{comment} \be\item Soit $n\in \N$. La fonction $\arctan$ est dérivable sur $[n,n+a]$, donc, en vertu du théorème des accroissements finis, il existe un réel $u_n \in ]n,n+a[$ tel que $\dfrac{ \arctan (n+a) - \arctan n}{a} = \dfrac{1}{u_n^2+1}$. On en déduit que \[ \arctan (n+a) - \arctan n = \dfrac{a}{u_n^2+1} \underset{n \rightarrow +\infty}{\sim} \dfrac{a}{n^2}. \] Puisque la série $\sum_{n\geqslant 1} \dfrac{a}{n^2}$ converge, la série $\sum_{n\geqslant 1} \arctan(n+a) - \arctan n$ converge. \item Il s'agit donc de calculer $\sum_{n=0}^{N}\pa{\arctan\pa{n+\frac12}-\arctan(n)}$ pour un N tel que $\abs{\sum_{n=N+1}^{+\infty}\pa{\arctan\pa{n+\frac12}-\arctan(n)}}<0.001$ \begin{eqnarray*} \abs{\sum_{n=N+1}^{+\infty}\pa{\arctan\pa{n+\frac12}-\arctan(n)}}&=&\sum_{n=N+1}^{+\infty}\pa{\arctan\pa{n+\frac12}-\arctan(n)}\\ &=&\sum_{n=N+1}^{+\infty}\frac{1}{2(u_n^2+1)}\leq \sum_{n=N+1}^{+\infty}\frac{1}{2n^2}\end{eqnarray*} Par une comparaison série intégrale $\sum_{n=N+1}^{+\infty}\frac{1}{2n^2}\leq \frac{1}{2N}$. Il suffit donc de calculer $S_{500}$. \begin{verbatim} S=0; for n=0:500 S=S+atan(n+1/2)-atan(n); end; disp(S) \end{verbatim} \ee \end{comment} \newpage \exos Soit $E$ un espace vectoriel muni d'un produit scalaire $\langle \cdot, \cdot \rangle$ et soit $\mathcal{F}= (v_1,\dots,v_n)$ une famille de vecteurs unitaires de $E$. \begin{enumerate} \item \textbf{Question de cours :} énoncer le théorème de Pythagore. \item Soit $(x,y) \in E^2$. Exprimer $\langle x , y \rangle$ à l'aide de $\|x+y\|^2$ et de $\|x-y\|^2$. \item Soient $X_1,\dots,X_n$ des variables aléatoires indépendantes et de même loi donnée par : $$\P(X_1= 1) = \P(X_1=-1) = \dfrac{1}{2}$$ On pose $U = X_1v_1 + \dots + X_nv_n$. Calculer l'espérance de $\|U\|^2$. \item En déduire qu'il existe $(\varepsilon_1,\dots,\varepsilon_n) \in \{-1;1\}^n$ tel que $\|\varepsilon_1 v_1 + \dots + \varepsilon_n v_n\| \leqslant \sqrt{n}$. \item Montrer que la famille $\mathcal{F}$ est orthogonale si, et seulement si : \[ \forall (\varepsilon_1,\dots,\varepsilon_n) \in \{-1;1\}^n,\ \|\varepsilon_1 v_1 + \dots + \varepsilon_n v_n\| = \sqrt{n}. \] \item Montrer que si $\mathcal{F}$ n'est pas orthogonale, il existe $(\varepsilon_1,\dots,\varepsilon_n) \in \{-1;1\}^n$ tel que $ \|\varepsilon_1 v_1 + \dots + \varepsilon_n v_n\| > \sqrt{n}$. \end{enumerate} \begin{comment} \begin{enumerate} \item Programme officiel ECS2 page 7. \item Soit $(x,y) \in E^2$. Alors : $\|x+y\|^2 - \|x-y\|^2 = \|x\|^2 + 2 \langle x , y \rangle + \|y\|^2 - \left(\|x\|^2 - 2 \langle x , y \rangle + \|y\|^2\right) = 4 \langle x , y \rangle$. $$\boxed{\forall (x,y) \in E^2,\ \langle x , y \rangle = \dfrac{1}{4}\left(\|x+y\|^2 - \|x-y\|^2\right).}$$ \item Pour tout $\omega \in \Omega$, on a : \[ \|U(\omega)\|^2 = \sum_{i=1}^n \sum_{j=1}^n X_i(\omega)X_j(\omega) \langle v_i , v_j\rangle. \] Par linéarité de l'espérance puis indépendance des $X_1,\dots,X_n$, on trouve : \[ \E\left(\|U\|^2\right) = \sum_{i=1}^n \sum_{j=1}^n \E(X_iX_j) \langle v_i , v_j\rangle = \sum_{i=1}^n \E(X_i^2) \|v_i\|^2 \boxed{= n.} \] \item Supposons le contraire par l'absurde. On aurait alors : $\forall \omega \in \Omega,\ \|U(\omega)\|^2 >n$ et donc $\E\left(\|U(\omega)\|^2\right) >n$, ce qui est absurde. Il existe donc $\omega \in \Omega$ tel que $\|U(\omega)\|^2\leqslant n$, i.e.: \fbox{ il existe $(\varepsilon_1,\dots,\varepsilon_n) \in \{-1;1\}^n$ tel que $\|\varepsilon_1 v_1 + \dots + \varepsilon_n v_n\| \leqslant \sqrt{n}$.} \item Supposons la famille $\mathcal{F}$ orthogonale. Soit $(\varepsilon_1,\dots,\varepsilon_n) \in \{-1;1\}^n$. La famille $(\varepsilon_1 v_1,\dots,\varepsilon_n v_n)$ reste orthogonale, donc le théorème de Pythagore assure que : \[ \|\varepsilon_1 v_1 + \dots + \varepsilon_n v_n\|^2 = \sum_{k=1}^n |\varepsilon_k|^2 \|v_k\|^2 = n \] On a donc bien $\|\varepsilon_1 v_1 + \dots + \varepsilon_n v_n\|= \sqrt{n}$. Supposons réciproquement que, pour tout $(\varepsilon_1,\dots,\varepsilon_n)\in \{-1;1\}^n,\ \|\varepsilon_1 v_1 + \dots + \varepsilon_n v_n\|= \sqrt{n}$. La variable aléatoire $\|U\|^2$ est donc constante égale à $n$. Soient $i\in \llbracket 1, n\rrbracket$ et $(\varepsilon_j)_{j\neq i} \in \{-1;1\}^{n-1}$. D'après une égalité de polarisation on a : \[ \left \langle v_i , \sum_{j\neq i} \varepsilon_j v_j \right \rangle = \dfrac{1}{4}\left( \left\|v_i + \sum_{j\neq i} \varepsilon_j v_j \right\|^2 - \left\|v_i - \sum_{j\neq i} \varepsilon_j v_j \right\|^2 \right) = \dfrac{1}{4}(n-n) = 0. \] Pour $j\neq i$, on peut écrire : \[ v_j = \dfrac{1}{2}\left[ \left(v_j + \sum_{k\neq i,j} v_k\right) -\left(-v_j + \sum_{k\neq i,j} v_k\right)\right] \] Le vecteur $v_j$ s'écrit comme combinaison linéaire de vecteurs orthogonaux à $v_i$, il est donc lui-même orthogonal à $v_i$. La famille $\mathcal{F}$ est donc orthogonale. \fbox{ La famille $\mathcal{F}$ est donc orthogonale si et seulement si $\forall (\varepsilon_1,\dots,\varepsilon_n) \in \{-1;1\}^n,\ \|\varepsilon_1 v_1 + \dots + \varepsilon_n v_n\| = \sqrt{n}.$} \item Si $\mathcal{F}$ n'était pas orthogonale, il existerait $(\varepsilon_1,\dots,\varepsilon_n)\in \{-1;1\}^n$ tel que $\|\varepsilon_1 v_1 + \dots + \varepsilon_n v_n\|\neq \sqrt{n}$. Supposons par l'absurde que, pour tout $(\varepsilon_1,\dots,\varepsilon_n)\in \{-1;1\}^n$, $\|\varepsilon_1 v_1 + \dots + \varepsilon_n v_n\|< \sqrt{n}$. La variable aléatoire $\|U\|^2-n$ serait alors une variable aléatoire finie, négative et d'espérance nulle, donc presque-sûrement nulle, ce qui est absurde pas hypothèse. On en déduit donc que : \fbox{il existe $(\varepsilon_1,\dots,\varepsilon_n)\in \{-1;1\}^n$ tel que $\|\varepsilon_1 v_1 + \dots + \varepsilon_n v_n\| > \sqrt{n}$.} \end{enumerate} \end{comment} \newpage \Esps \noindent Soit $(u_n)$ la suite définie par~ : $u_0 \in \mathbb{R}$ et $\forall n \in \mathbb{N}, \ u_{n+1} = |u_n-n|$. \noindent À l'aide de Scilab, on calcule $(u_0,u_1,\ldots,u_{10})$ pour certaines valeurs de $u_0$. On obtient le tableau suivant~: \[ \begin{array}{|c||cccccccccccccccc|} \hline \multicolumn{1}{|c||}{ \quad u_0 \quad \qquad } & \multicolumn{16}{l|}{\text{Calcul de $(u_0,u_1,\ldots,u_{15})$ par Scilab}} \\ \hline \cellcolor[gray]{.9} \texttt{-3} & \texttt{-3.} & \cellcolor[gray]{.9} \texttt{3. } & \texttt{2.} & \cellcolor[gray]{.9} \texttt{0. } & \texttt{3. } & \cellcolor[gray]{.9} \texttt{1. } & \texttt{4. } & \cellcolor[gray]{.9} \texttt{2. } & \texttt{5. } & \cellcolor[gray]{.9} \texttt{3. } & \texttt{6. } & \cellcolor[gray]{.9} \texttt{4. } & \texttt{7. } & \cellcolor[gray]{.9} \texttt{5. } & \texttt{8. } & \cellcolor[gray]{.9} \texttt{6. } \\ \hline \cellcolor[gray]{.9} \texttt{4} & \texttt{4. } & \cellcolor[gray]{.9} \texttt{4. } & \texttt{3. } & \cellcolor[gray]{.9} \texttt{1. } & \texttt{2. } & \cellcolor[gray]{.9} \texttt{2. } & \texttt{3. } & \cellcolor[gray]{.9} \texttt{3. } & \texttt{4. } & \cellcolor[gray]{.9} \texttt{4. } & \texttt{5. } & \cellcolor[gray]{.9} \texttt{5. } & \texttt{6. } & \cellcolor[gray]{.9} \texttt{6. } & \texttt{7. } & \cellcolor[gray]{.9} \texttt{7. } \\ \hline \cellcolor[gray]{.9} \texttt{6} & \texttt{6. } & \cellcolor[gray]{.9} \texttt{6. } & \texttt{5. } & \cellcolor[gray]{.9} \texttt{3. } & \texttt{0. } & \cellcolor[gray]{.9} \texttt{4. } & \texttt{1. } & \cellcolor[gray]{.9} \texttt{5. } & \texttt{2. } & \cellcolor[gray]{.9} \texttt{6. } & \texttt{3. } & \cellcolor[gray]{.9} \texttt{7. } & \texttt{4. } & \cellcolor[gray]{.9} \texttt{8. } & \texttt{5. } & \cellcolor[gray]{.9} \texttt{9. } \\ \hline \cellcolor[gray]{.9} \texttt{9} & \texttt{9. } & \cellcolor[gray]{.9} \texttt{9. } & \texttt{8. } & \cellcolor[gray]{.9} \texttt{6. } & \texttt{3. } & \cellcolor[gray]{.9} \texttt{1. } & \texttt{4. } & \cellcolor[gray]{.9} \texttt{2. } & \texttt{5. } & \cellcolor[gray]{.9} \texttt{3. } & \texttt{6. } & \cellcolor[gray]{.9} \texttt{4. } & \texttt{7. } & \cellcolor[gray]{.9} \texttt{5. } & \texttt{8. } & \cellcolor[gray]{.9} \texttt{6. } \\ \hline \end{array} \] \noindent Déterminer un équivalent de $u_n$ lorsque $n \rightarrow+\infty$. \begin{comment} Ce qu'il faut conjecturer, c'est qu'à partir d'un certain rang, on aurait~: $\forall n \in \mathbb{N}, \ u_{n+2} = u_n+1$. \\ (Regarder une case sur deux, suivant les couleurs du tableau. La suite $(u_n)$ est clairement positive (à partir de $u_1$), et décroissante sur ses premiers termes (à partir de $u_1$ puisque~: \[ u_1=|u_0|, \quad u_2=u_1-1, \quad u_3=u_1-1-2, \quad \quad u_4 = u_1-1-2-3, \ldots \] Plus précisément tant que $u_n0$. La variable aléatoire $X$ suit une \emph{loi de Cauchy} de paramètre $\lambda$, noté $X\sim {\cal C}(\lambda)$ si $X$ admet pour densité la fonction définie sur $\R$ par : $$f_\lambda:x\mapsto \frac{1}{\pi}\frac{\lambda}{\lambda^2+x^2}$$ On pose $Z=\ln|X|$. Vérifier que $Z$ est définie presque sûrement et en déterminer une densité. \item On considère maintenant deux variables indépendantes $X$ et $Y$ suivant une loi de Cauchy de paramètre 1. Posons $U=\ln|XY|$. \be \item Vérifier que $U$ est définie presque sûrement et montrer qu'elle admet la fonction $g$ comme densité où : $$g:x\mapsto \frac{4}{\pi^2}\frac{x}{e^x-e^{-x}} \; \1{x\in \Rstar} $$ On pourra remarquer, au cours du calcul à effectuer, que: $$\mbox{Si }x\neq 0,\;\frac{1}{(y+1)(y+e^{2x})}=\frac{1}{e^{2x}-1}\pa{\frac{1}{y+1}-\frac{1}{y+e^{2x}}}$$ \item Etudier la continuité de $g$ sur $\R$. \ee \item \be \item En utilisant le fait que $g$ est une densité, montrer que $\int_0^{+\infty}\frac{\ln t}{t^2-1}\dt=\frac{\pi^2}{4}$. \item En déduire que $\int_0^1\frac{\ln t}{t^2-1}\dt=\frac{\pi^2}{8}$. \ee \item \be \item Montrer que, pour $t\in[0,1[$: $$\frac{\ln t}{t^2-1}=-\sum_{k=0}^nt^{2k}\ln t+r_n(t) \mbox{ où } r_n(t)=\frac{t\ln t}{t^2-1}t^{2n+1}.$$ \item Prouver que $\int_0^1r_n(t)\dt\to 0$ quand $n\to+\infty$. \item En déduire la valeur de $\sum_{k=0}{+\infty}\frac{1}{(2k+1)^2}$ puis celle de $\sum_{k=1}{+\infty}\frac{1}{k^2}$. \ee \ee %------------------------------------------------------------------------------------------------------------------ \begin{comment} \be \item Question de cours: programme ECS2 2014 p. 14. \item La probabilité $\P(X=0)$ est nulle donc $Z$ est définie presque sûrement. De plus, $Z$ est à valeurs dans $\R$ et pour $x\in\R$, en notant $F_Z$ la fonction de répartition de $Z$: \begin{eqnarray*} F_Z(x)&= & \P(Z\leq x)\\ & = & \P(\ln |X|\leq x)\\ & = & \P(|X|\leq e^x)\text{ par croissance de l'exponentielle}\\ & = & \P(-e^x\leq X\leq e^x)\\ & = & \int_{-e^x}^{e^x} \frac{1}{\pi}\frac{\lambda}{\lambda^2+t^2}\dt\\ & =& \frac{2}{\pi}\int_0^{e^x}\frac{\lambda}{\lambda^2+t^2}\dt\text{ par parité}\\ & = & \frac{2}{\pi}{\rm Arctan}\paf{e^x}{\lambda} \end{eqnarray*} En particulier $F_Z$ est ${\cal C}^1$ sur $\R$. On en déduit que $Z$ est une variable aléatoire à densité et une densité est la fonction: $$\boxed{f_\lambda:t\mapsto \frac{2\lambda}{\pi}\frac{e^t}{\lambda^2+e^{2t}}}$$ \item \be \item $U$ est définie dès que $XY\neq 0$, or: $$\P(XY=0)=\P(\acco{X=0}\cup\acco{Y=0})\leq P(X=0)+P(Y=0)=0$$ donc $U$ est définie presque sûrement. De plus, $U=\ln|X|+\ln|Y|$ et donc par indépendance de $X$ et de $Y$, si on note respectivement $f_1$ et $f_2$ des densités de $\ln|X|$ et de $\ln|Y|$, la variable $U$ admet comme densité la fonction $g$ définie par: $$g(x)=\int_{-\infty}^{+\infty}f_1(t)f_2(x-t)\dt$$ On va mener le calcul en utilisant le résultat de la question précédente. Soit $x\in\R^*$ \begin{eqnarray*} g(x) &= &\frac{4}{\pi^2}\int_{-\infty}^{+\infty}\frac{e^t}{1+e^{2t}}\frac{e^{x-t}}{1+e^{2(x-t)}}\dt\\ & =& \frac{4}{\pi^2}\int_{-\infty}^{+\infty}\frac{e^{x}}{(1+e^{2t})(e^{2t}+e^{2x})}e^{2t}\dt\\ &=& \frac{2}{\pi^2}\int_{0}^{+\infty}\frac{e^{x}}{(1+y)(y+e^{2x})}\dy\text{ en posant }y=e^{2t} \text{ (bijection croissante } {\cal C}^1)\\ &= & \frac{2e^x}{\pi^2}\int_{0}^{+\infty}\frac{1}{e^{2x}-1}\pa{\frac{1}{y+1}-\frac{1}{y+e^{2x}}}\dy\text{ car }e^{2x}\neq 1\text{ pour }x\neq 0\\ & = & \frac{2}{\pi^2}\frac{1}{e^x+e^{-x}}\crocint{\ln\frac{y+1}{y+e^{2x}}}_0^{+\infty}\\ &= & \boxed{\frac{4}{\pi^2}\frac{x}{e^x-e^{-x}}} \end{eqnarray*} \item Il suffit de calculer la limite quand $x$ tend vers $0$ de l'expression trouvée ci-dessus. Or: $$ \frac{4}{\pi^2}\frac{x}{e^x-e^{-x}}\eq{0} \frac{4}{\pi^2}\frac{x}{2x}=\frac{2}{\pi^2}$$ On en déduit donc que \fbox{ $g$ n'est pas continue en $0$.} \ee \item \be \item Comme $g$ est une densité définie sur $\R$: $$\int_{-\infty}^{+\infty}g(x)\dx=1\text{ donc }\int_{-\infty}^{+\infty}\frac{x}{e^x-e^{-x}}\dx=\frac{\pi^2}{4}$$ Or en effectuant le changement de variable $t=e^x$ -- valide car défini par une bijection croissante de classe ${\cal C}^1$ -- on obtient par ailleurs: $$\boxed{\frac{\pi^2}{4}=\int_{-\infty}^{+\infty}\frac{x}{e^x-e^{-x}}\dx=\int_0^{+\infty}\frac{\ln t}{t^2-1}\dt}$$ d'où le résultat demandé. \item Il suffit d'écrire par la relation de Chasles: $$\int_0^{+\infty}\frac{\ln t}{t^2-1}\dt=\int_0^{1}\frac{\ln t}{t^2-1}\dt+\int_1^{+\infty}\frac{\ln t}{t^2-1}\dt$$ et en effectuant le changement de variable $u=\frac{1}{t}$ dans la seconde intégrale -- changement de variable là encore valide car toujours défini par une bijection décroissante et ${\cal C}^1$ -- on obtient: $$\int_1^{+\infty}\frac{\ln t}{t^2-1}\dt=-\int_{1}^0\frac{\ln\paf{1}{u}}{\paf{1}{u}^2-1}\frac{\du}{u^2}=\int_0^{1}\frac{\ln u}{u^2-1}\du$$ d'où $2\int_0^{1}\frac{\ln t}{t^2-1}\dt=\frac{\pi^2}{4}$ et donc finalement: $\boxed{\int_0^{1}\frac{\ln t}{t^2-1}\dt=\frac{\pi^2}{8}}$. \ee \item \be \item Pour $t\neq 1$, par la formule pour les sommes géométriques: $$\sum_{k=0}^nt^{2k}=\frac{t^{2n+2}-1}{t^2-1}$$ d'où: $$\frac{1}{t^2-1}=-\sum_{k=0}^nt^{2k}+\frac{t^{2n+2}}{t^2-1}$$ puis finalement: $$\boxed{ \frac{\ln t}{t^2-1}=-\sum_{k=0}^nt^{2k}\ln t+\frac{t^{2n+2}\ln t}{t^2-1}=-\sum_{k=0}^nt^{2k}\ln t+r_n(t)}$$ avec $r_n(t)=\frac{t\ln t}{t^2-1}t^{2n+1}$. \item On pose $\varphi(t)=\frac{t\ln t}{t^2-1}$. Alors la fonction $\varphi$ est continue sur $]0,1[$. De plus, par croissance comparée: $$\lim\limits_{x\to 0}\varphi(t)=0$$ tandis qu'au voisinage de $1$: $$\varphi(t)\eq{1}\frac{t(t-1)}{t^2-1}=\frac{t}{t+1}$$ d'où: $\lim\limits_{x\to 1}\varphi(t)=\frac{1}{2}$ Ainsi la fonction $\varphi$ se prolonge par continuité sur $[0,1]$ en posant $\varphi(0)=0$ et $\varphi(1)=\frac{1}{2}$. On continue de noter $\varphi$ la fonction ainsi prolongée. Alors, la fonction $\varphi$ est continue sur le segment $[0,1]$ et elle y est donc bornée. Il existe donc $M>0$ tel que: $$\forall t \in[0,1]\quad |\varphi(t)|\leq M.$$ Remarquons que la continuité de $\varphi$ sur $[0,1]$ implique que la fonction $r_n$ est bien intégrable sur $[0,1]$ et on peut donc parler de son intégrale sur ce segment. Alors: \begin{eqnarray*} \abs{\int_0^1r_n(t)\dt }& \leq & \int_0^1\abs{r_n(t)}\dt\\ &\leq & M\int_0^1t^{2n+1}\dt\\ & \leq & \frac{M}{2n+2} \end{eqnarray*} Par théorème de comparaison, comme $\frac{M}{2n+2}\to 0$ quand $n\to+\infty$, on en déduit que: $$\boxed{\int_0^1r_n(t)\dt\to 0\text{ quand }n\to+\infty}$$ \item Remarquons que pour tout $k\in\N$, la fonction $t\mapsto t^{2k}\ln t$ est intégrable sur $[0,1]$. Cette fonction est en effet continue sur $]0,1]$ et, pour $k\geq 1$, se prolonge en continuité par $0$ car, par croissance comparée, $\lim\limits_{t\to 0}t^{2k}\ln t=0$. Dans le cas où $k=0$, $t\mapsto \ln t$ est bien intégrable au voisinage de $0$, par exemple par comparaison à une intégrale de Riemann convergente, ainsi $\abs{\ln t}=\peto{\frac{1}{\sqrt{t}}}$ au voisinage de $0$. On peut aussi calculer explicitement une primitive (c'est le calcul fait ci-dessous). Cela remarqué, on peut intégrer terme à terme sur $]0,1[$ l'égalité obtenue à la question 5.(a), ce qui donne: $$\int_0^1\frac{\ln t}{t^2-1}\dt=-\sum_{k=0}^n\int_0^1t^{2k}\ln t\dt +\int_0^1r_n(t)\dt $$ On pose $I_k=\int_0^1t^{2k}\ln t\dt$ pour $k\in\N$. En intégrant par parties, toutes les fonctions étant de classe ${\cal C}^1$ sur $]0,1]$, on obtient: \begin{eqnarray*} I_k &= & \crocint{\frac{t^{2k+1}}{2k+1}\ln t}_0^1-\int_0^1\frac{t^{2k}}{2k+1}\dt\\ & =& -\frac{1}{(2k+1)^2} \end{eqnarray*} Ainsi: $$\int_0^1\frac{\ln t}{t^2-1}\dt=\sum_{k=0}^n\frac{1}{(2k+1)^2} +\int_0^1r_n(t)\dt $$ d'où: $$\sum_{k=0}^n\frac{1}{(2k+1)^2}=\int_0^1\frac{\ln t}{t^2-1}\dt-\int_0^1r_n(t)\dt =\frac{\pi^2}{8}-\int_0^1r_n(t)\dt $$ Par la question précédente, on en déduit en passant à la limite que: $$\boxed{\sum_{k\geq 0}\frac{1}{(2k+1)^2}=\frac{\pi^2}{8}}$$ On remarque alors, en séparant les termes d'indice respectivement pair et impair, que: $$\sum_{k\geq 1}\frac{1}{k^2}=\sum_{k\geq 1}\frac{1}{(2k)^2}+\sum_{k\geq 0}\frac{1}{(2k+1)^2}= \frac{1}{4}\sum_{k\geq 1}\frac{1}{k^2}+\sum_{k\geq 0}\frac{1}{(2k+1)^2}$$ d'où: $$\sum_{k\geq 1}\frac{1}{k^2}=\frac{4}{3}\sum_{k\geq 0}\frac{1}{(2k+1)^2}\quad\text{soit}\quad \boxed{\sum_{k\geq 1}\frac{1}{k^2}=\frac{\pi^2}{6}}$$ \ee \ee \end{comment} \newpage \Esps Soit $f$ une fonction continue sur $\R$ telle que $$\forall x\in \R, \qquad f(\frac{x+1}{2})=f(x).$$ Montrer que $f$ est constante.\\ \begin{comment} \\ Soit $x\in \R$.\\ On considère la suite $(u_n)$ définie par $u_0=x$ et $\forall n\in \N $ $u_{n+1}=\frac{u_n+1}{2}$. Si $x\leq 1$ alors $(u_n)$ majorée par $1$ [récurrence] et croissante [signe de $f(x)-x$]. Si $x>1$ $(u_n)$, on montre de même que $(u_n)$ est minorée par $1$ et décroissante.\\ Ainsi dans tous les cas, $(u_n)$ converge. Elle converge donc vers l'unique point fixe $1$.\\ (on peut aussi calculer explicitement $u_n$ en remarquant que c'est une suite arithmético-géométrique : $\forall n\in\N$, $u_n=1+\paf12^n(x-1)$) Par récurrence, $\forall n\in \N$, $f(u_n)=f(x)$ par continuité et unicité de la limite $f(x)=f(1)$ et \fbox{$f$ est constante}. \end{comment} \newpage \exos Soit $n \geqslant 2$. On munit l'espace vectoriel $\mathcal{M}_{n,1}(\mathbb{R})$ de son produit scalaire canonique, défini par~: $\langle X,Y \rangle = \ ^t \!XY.$ \\ \begin{enumerate} \item Soit $Z = \left( \begin{array}{c} x_1 \\ x_2 \\ \vdots \\ x_n \end{array} \right)$ un vecteur non nul de $\mathcal{M}_{n,1}(\mathbb{R})$ et on note $H = Z \ ^t\!Z$. \begin{enumerate} \item Montrer que $H$ est diagonalisable et vérifier que $\text{Ker}(H) = (\text{Vect}(Z))^{\perp}$. \item Montrer que $Z$ est vecteur propre de $H$ et préciser la valeur propre associée. \end{enumerate} \end{enumerate} \noindent Dans la suite de l'exercice, on considère $X_1,X_2,\ldots,X_n$ des variables aléatoires mutuellement indépendantes, toutes définies sur un même espace probabilisé $(\Omega,\mathcal{A},\mathbb{P})$, suivant toutes une loi normale centrée réduite $\mathcal{N}(0,1)$. \\ On admet également la valeur suivante~: $\Gamma\left( \frac{1}{2} \right) = \sqrt{\pi}$. \\ \begin{enumerate} \setcounter{enumi}{1} \item {\bf Question de cours } Lois $\gamma$ : densité et stabilité par une opération à expliciter. \item Soit $i\in \llb 1,n \rrb$ \begin{enumerate} \item Donner la fonction de répartition de la variable $\displaystyle V_i = \frac{X_i^2}{2}$ en fonction de $\Phi$ la fonction de répartition de la loi normale centrée réduite.\\ Justifier que $V_i$ possède une densité et proposez-en une. \\ Reconnaître alors une loi $\gamma$ dont on précisera le paramètre. \item Donner la loi suivie par $\displaystyle \sum\limits_{i=1}^{n} V_i$. \end{enumerate} \item On note encore $Z = \left( \begin{array}{c} X_1 \\ X_2 \\ \vdots \\ X_n \end{array} \right)$ le vecteur aléatoire de $\mathcal{M}_{n,1}(\mathbb{R})$ obtenu et la matrice aléatoire $H = Z \ ^t \!Z$. \\ Soit $F$ un sous-espace vectoriel de $\mathcal{M}_{n,1}(\mathbb{R})$. On note $p_F$ la projection orthogonale sur $F$. \begin{enumerate} \item Calculer la probabilité que $Z$ soit le vecteur nul. \item Déterminer la loi suivie par la valeur propre aléatoire $\|Z\|^2$ de la matrice~$H$. \item Soit $u = \left( \begin{array}{c} u_1 \\ \vdots \\ u_n \end{array} \right)$ un vecteur normé de $F$. \\ Montrer que la variable aléatoire $\langle u,Z \rangle$ suit une loi normale dont on précisera les paramètres. \item On admet que pour tout $k \in \llbracket 1, n \rrbracket$, si $(v_1,\ldots,v_k)$ est une famille orthogonale de $\mathcal{M}_{n,1}(\mathbb{R})$, alors les variables aléatoires $\langle v_1,Z \rangle,$ $ \langle v_2, Z \rangle, \ldots, \langle v_k, Z \rangle$ sont mutuellement indépendantes. \\ Vérifier que les variables aléatoires $\| p_F(Z) \|^2$ et $\| Z-p_F(Z)\|^2$ sont indépendantes et déterminer leur loi. \end{enumerate} \end{enumerate} \begin{comment} \begin{enumerate} \item Programme officiel ECS2 pages 12 (définition) et 14 (stabilité par la somme) \item %Soit $Z = \left( \begin{array}{c} x_1 \\ x_2 \\ \vdots \\ x_n \end{array} \right)$ un vecteur non nul de $\mathcal{M}_{n,1}(\mathbb{R})$ et on note $H = Z \ ^t\!Z$. \begin{enumerate} \item %Montrer que $H$ est diagonalisable et vérifier que $\text{Ker}(H) = (Vect(Z))^{\perp}$. $ ^t\!H = ^t\!(Z \ ^t\!Z) = Z \ ^t\!Z = H$, donc $H$ est \fbox{ symétrique réelle donc diagonalisable.} \\ De plus, pour tout $T \in \mathcal{M}_{n,1}(\mathbb{R})$. \[ T \in \text{Ker}(H) \Longleftrightarrow HT=0 \Longleftrightarrow Z( \ ^t\!ZH)=0 \Longleftrightarrow \langle Z,H \rangle Z = 0 \overset{Z\neq 0}{\Longleftrightarrow} \langle Z,H \rangle = 0 \Longleftrightarrow\boxed{ T \in ({\text{Vect}}(Z))^\perp} \] \item %Montrer que $Z$ est vecteur propre de $H$ et préciser la valeur propre associée. Le vecteur $Z$ est bien non nul et~: $HZ = Z \ ^t\!Z Z = Z (\langle Z,Z \rangle) = \|Z\|^2 Z$. \\ Ainsi, \fbox{$Z$ est vecteur propre de $H$ pour la valeur propre $\|Z\|^2 \neq 0$.} \end{enumerate} %\end{enumerate} %Dans la suite de l'exercice, on considère $X_1,X_2,\ldots,X_n$ des variables aléatoires mutuellement indépendantes, toutes définies sur un même espace probabilisé $(\Omega,\mathcal{A},\mathbb{P})$, suivant toutes une loi normale centrée réduite $\mathcal{N}(0,1)$. \\ %On note alors encore $Z = \left( \begin{array}{c} X_1 \\ X_2 \\ \vdots \\ X_n \end{array} \right)$ le vecteur aléatoire de $\mathcal{M}_{n,1}(\mathbb{R})$ obtenu et la matrice aléatoire $H = Z \ ^t \!Z$. \\ %On note également $\overline{Z} = \displaystyle \frac{1}{n} \sum\limits_{i=1}^{n} X_i$ et $S^2 = \displaystyle \frac{1}{n-1} \sum\limits_{i=1}^{n} (X_i-\overline{Z})^2$ la moyenne empirique et la variance empirique de $Z$. \\ %On admet enfin que~: $\Gamma\left( \frac{1}{2} \right) = \sqrt{\pi}$. \\ %\begin{enumerate} \item \begin{enumerate} \item %Donner la fonction de répartition de la variable $\displaystyle V_i = \frac{X_i^2}{2}$ en fonction la fonction de répartition $\Phi$ de la loi normale centrée réduite. Justifier que $V_i$ possède une densité et proposez-en une. \\ Reconnaître alors une loi $\gamma$ dont on précisera le paramètre. $X_i(\Omega)=\mathbb{R}$, donc $V_i(\Omega)=\mathbb{R}^+$. \begin{itemize} \item[$\bullet$] pour $x < 0$, $F_{V_i}(x) = 0$; \item[$\bullet$] pour $x \geqslant 0$, \[ F_{V_i}(x) = \mathbb{P}(V_i \leqslant x) = \mathbb{P}(X_i^2 \leqslant 2x) = \mathbb{P}(-\sqrt{2x} \leqslant X_i \leqslant \sqrt{2x}) = 2 \Phi(\sqrt{2x})-1 \] \end{itemize} Ainsi~: \[ \forall x \in \mathbb{R}, \ F_{V_i}(x) = \left\{ \begin{array}{cl} 0 & \text{ si } x < 0 \\ 2 \Phi(\sqrt{2x})-1 & \text{ si } x \geqslant 0 \end{array} \right. \] La fonction $\Phi$ étant de classe $\mathcal{C}^1$ sur $\mathbb{R}$ et vérifiant $\Phi(0)=1/2$, on voit que $F_{V_i}$ est continue sur $\mathbb{R}$ et de classe $\mathcal{C}^1$ au moins sur $\mathbb{R} \setminus \{0\}$ ce qui suffit pour affirmer que $V_i$ est une variable à densité. \\ Comme densité de $V_i$, on peut proposer~: \[ f_{V_i}(x) = \left\{ \begin{array}{cl} 0 & \text{ si } x \leqslant 0 \\ 2 \cdot \frac{\sqrt{2}}{2 \sqrt{x}} \Phi'(\sqrt{2x}) & \text{ si } x > 0 \end{array} \right. = \left\{ \begin{array}{cl} 0 & \text{ si } x \leqslant 0 \\ \frac{1}{\sqrt{\pi x}} e^{-x} & \text{ si } x > 0 \end{array} \right. = \left\{ \begin{array}{cl} 0 & \text{ si } x \leqslant 0 \\ \frac{1}{\Gamma(1/2)} x^{1/2-1} e^{-x} & \text{ si } x > 0 \end{array} \right. \] \fbox{On reconnaît donc une loi $\gamma$ de paramètre $1/2$.} \item %Donner la loi suivie par $\displaystyle \sum\limits_{i=1}^{n} V_i$. En déduire la loi suivie par la valeur propre aléatoire $\|Z\|^2$ de la matrice~$H$. Les $V_i$ étant mutuellement indépendantes de loi $\gamma(1/2)$, leur somme $W=\displaystyle \sum\limits_{i=1}^{n} V_i$ suit donc une loi \fbox{$\gamma(n/2)$.} \end{enumerate} \item %Soit $F$ un sous-espace vectoriel de $\mathcal{M}_{n,1}(\mathbb{R})$. %On note $p_F$ la projection orthogonale sur $F$. \begin{enumerate} \item %Calculer la probabilité que $Z$ soit le vecteur nul. \[ [Z=0] = \bigcap\limits_{k=1}^{n} [X_k=0] \subset [X_1=0] \] Ainsi, $0 \leqslant \mathbb{P}(Z=0) \leqslant \mathbb{P}(X_1=0)=0$, donc \fbox{$\mathbb{P}(Z=0)=0$.} \item On a $\|Z\|^2 = \sum\limits_{i=1}^{n} X_i^2 = 2 W$, avec $W$ la variable aléatoire calculée à la question 2(b). On a donc~: \[ \forall x \in \mathbb{R}, \ \mathbb{P}(\|Z\|^2 \leqslant x) = \mathbb{P}\left(W \leqslant \frac{x}{2} \right)=F_W\left( \frac{x}{2} \right) \] Par composition, $\|Z\|^2$ est donc encore une variable à densité, de loi~: \[ \forall x \in \mathbb{R}, \boxed{ \ f_{\|Z\|^2}(x) = \frac{1}{2} f_W\left( \frac{x}{2} \right) = \frac{1}{2} \cdot \frac{1}{\Gamma\left( \frac{n}{2} \right)} e^{\frac{n}{2}-1} e^{-x} \textbf{1}_{]0,+\infty[}(x)} \] \item %Soit $u = \left( \begin{array}{c} u_1 \\ \vdots \\ u_n \end{array} \right)$ un vecteur orthonormé de $F$. \\ %Montrer que la variable $\langle u,Z \rangle$ suit une loi normale dont on précisera les paramètres. La variable aléatoire $\langle u,Z \rangle = \sum\limits_{k=1}^{n} u_k X_k$ est une combinaison linéaire de variables aléatoires indépendantes de loi normale, elle suit donc encore une loi normale. De plus, \[ \mathbb{E}[\langle u,Z \rangle] = \sum\limits_{k=1}^{n} u_k \mathbb{E}[X_k]= 0 \] et \[ \mathbb{V}[\langle u,Z \rangle] = \sum\limits_{k=1}^{n} u_k^2 \mathbb{V}[X_k] = \sum\limits_{k=1}^{n} u_k^2 = \|u\|^2 = 1 \] Ainsi, \fbox{$\langle u,Z \rangle$ suit une loi normale centrée réduite.} \item %Déterminer la loi suivie par les variables aléatoires $\| p_F(Z) \|^2$ et $\| Z-p_F(Z)\|^2$. Notons $r$ la dimension de $F$. \\ Notons $\mathcal{B}=(v_1,\ldots,v_r,v_{r+1},\ldots,v_n)$ une base orthonormée de $\mathcal{M}_{n,1}(\mathbb{R})$ telle que $(v_1,\ldots,v_r)$ soit une base orthonormée de $F$ (et $(v_{r+1},\ldots,v_n)$ une base orthonormée de $F^\perp$). Alors~: $p_F(Z) = \sum\limits_{k=1}^{r} \langle v_k,Z \rangle v_k$ et $Z-p_F(Z) = \sum\limits_{k=r+1}^{n} \langle v_k, Z \rangle v_k$. Par théorème de Pythagore, on a donc $\| p_F(Z) \|^2 = \sum\limits_{k=1}^{r} (\langle v_k, Z \rangle)^2$ et $\|Z-p_F(Z)\|^2 = \sum\limits_{k=r+1}^{n} (\langle v_k,Z \rangle)^2$ On sait d'après (a) que chaque $\langle v_k, Z \rangle$ suit une loi normale centrée réduite, et elles sont indépendantes. d'après le résultat qui a été admis. \\ Par le lemme des coalitions, on voit donc que $\|p_F(Z)\|^2$ et $\| Z-p_F(Z)\|^2$ sont indépendantes. \\ De plus, en adaptant la méthode du (2.b), on voit que \fbox{$\frac{\|p_F(Z)\|^2}{2}$ suit une loi gamma $\gamma(d/2)$} et que \fbox{$\frac{\|Z-p_F(Z)\|^2}{2}$ suit une loi gamma $\gamma((n-d)/2)$.} On en déduit ainsi la loi de $\|p_F(Z)\|^2$ et de $\|Z-p_F(Z)\|^2$ par transformation affine. \end{enumerate} %\item On suppose dans cette question que $F= Vect\left( \left( \begin{array}{c} 1 \\ \vdots \\ 1 \end{array} \right) \right) $. \\ % %\begin{enumerate} %\item Rappeler la loi de $\overline{Z}$. %\item Expliciter $p_F(Z)$ et $\|Z-p_F(Z)\|^2$. %En déduire la loi de la variable $S^2 = \displaystyle \displaystyle \frac{1}{n-1} \sum\limits_{i=1}^{n} (X_i-\overline{Z})^2$. %\end{enumerate} \end{enumerate} \end{comment} \newpage \Esps Soit une suite r\'eelle $(u_n)_{n\in \N}$ telle que $$ \lim_{n\to +\infty}(u_n-u_n^2)=0 $$ On suppose que $(u_n)_{n\in \N}$ ne tend pas vers $0$. \be \item A t-on ${\lim_{n\to +\infty}u_n=1 }$? \item La suite $(u_n)_{n\in \N}$ est-elle born\'ee? \ee \bigskip \begin{comment} \be \item \fbox{Non} pour la question. Prendre $u_n=0$ si $n$ pair et $1$ sinon \item \fbox{Oui} pour la question. Comme il existe $M>0$ tel que pour $n$ assez grand $-M \le u_n-u_n^2$, soit $u_n^2-u_n-M\le 0$, la suite $(u_n)_{n\in \N}$ se situe entre les $2$ racines du polynôme $X^2-X-M$ et est donc forc\'ement born\'ee. \ee \end{comment} \newpage \exos Soit $n\in \N^*$. Notons $P_n$ le polynôme $(X^2-1)^n$ et $L_n=P_n^{(n)}$, sa dérivé $n$-ième. \begin{enumerate} \item \textbf{Question de cours : } énoncer le théorème d'intégration par parties. \item À l'aide d'intégrations par parties successives, montrer que : \[ \forall Q \in \R_{n-1}[X],\ \int_{-1}^1 Q(x) L_n(x) \dx = 0. \] \item On note $x_1$,...,$x_p$ les (éventuelles) racines de $L_n$ d'ordre de multiplicité impaire. On définit alors le polynôme Q par : $$\left\{ \begin{array}{ll} Q=(X-x_1)\cdot \cdot \cdot (X-x_p) & \text{ si }P\text{ admet des racines d'ordre de multiplicité impair} \\ Q=1 & \text{ sinon } \end{array}\right.$$%} \`A l'aide du polynôme $Q$ et de la question précédente, montrer que $L_n$ admet $n$ racines simples appartenant à l'intervalle $]-1,1[$. \item Pour tout $k \in \llbracket 1, n\rrbracket$, on note $$g_k:\fonc{\R_{2n-1}[X]}{\R}{P}{P(x_k)} \mbox{ et } \; \; f:\fonc{\R_{2n-1}[X]}{\R}{P}{ \int_{-1}^1 P(x) \dx }$$ Montrer que $f$ est nulle sur $\bigcap_{k=1}^{n} \Ker g_k$. \item Montrer que la famille $(g_k)_{1\leqslant k \leqslant n}$ est libre dans ${\mathcal L} (\R_{2n-1}[X],\R)$. \item On {\bf admet } provisoirement que si $(\varphi_1,\dots,\varphi_m)$ est une famille libre de $m$ formes linéaires sur un $\R$-espace vectoriel $E$, il existe des vecteurs $u_1,\dots,u_m$ tels que pour tout $(i,j) \in \llbracket 1, m\rrbracket$, $\varphi_i(u_i) = 1$ et $\varphi_i(u_j) = 0$ si $i \neq j$. \medskip Déduire des questions précédentes qu'il existe $(\alpha_1,\dots, \alpha_n) \in \R^n$ tel que : \[ \forall Q \in \R_{2n-1}[X],\ \int_{-1}^1 Q(x) \dx = \sum_{i=1}^n \alpha_i Q(x_i). \] \item Montrer la propriété provisoirement admise par récurrence sur $m$. \end{enumerate} \begin{comment} \begin{enumerate} \item Programme officiel ECS1 page 12. \item Par intégration par parties (les polynômes sont $\mathcal{C}^\infty$ sur $[-1,1]$), on trouve : \[ \int_{-1}^1 Q(x)L_n(x) \dx = \Big[P_n^{(n-1)}(x)Q(x) \Big]_{-1}^1 - \int_{-1}^1 Q'(x) P_n^{(n-1)}(x) \dx. \] Remarquons que 1 et -1 sont racines $n$-èmes de $P_n$, donc racines simples de $P_n^{(n-1)}$. On trouve alors : \[ \int_{-1}^1 Q(x)L_n(x) \dx = - \int_{-1}^1 Q'(x) P_n^{(n-1)}(x) \dx. \] En intégrant $n-1$ fois de plus par parties, on trouve : \[ \int_{-1}^1 Q(x)L_n(x) \dx = - \int_{-1}^1 Q'(x) P_n^{(n-1)}(x) \dx = \dots = (-1)^n \int_{-1}^1 Q^{(n)}(x) P_n(x) \dx \boxed{=0}. \] la dernière égalité étant obtenue puisque $Q^{(n)} =0$. \item Si $p \leqslant n-1$, le résultat de la question précédente assure que $\int_{-1}^1 Q(x)L_n(x) \dx = 0$, ce qui est absurde puis le polynôme $QL_n$ est par construction de signe constant et ne s'annule que sur les racines de $L_n$, qui sont en nombre fini. On en déduit que $p=n$, i.e. \fbox{$L_n$ admet $n$ racines dans $]-1,1[$.} Puisque $\deg L_n = n$, toutes ces racines sont simples. \item Soit $R \in \bigcap_{k=1}^{n} \Ker g_k$. On a alors $R(x_1) = \dots = R(x_n) = 0$. Puisque les réels $x_1,\dots,x_n$ sont deux-à-deux distincts, le polynôme $(X-x_1)\dots(X-x_n)$ divise $R$, donc $L_n$ divise $R$. Il existe donc un polynôme $R \in \R_{n-1}[X]$ tel que $R = QL_n$. La première question assure alors que : \[ f(R) = \int_{-1}^1 R(x) \dx = \int_{-1}^1 Q(x)L_n(x) \dx = 0 \] \fbox{$f$ est nulle sur $\bigcap_{k=1}^{n} \Ker g_k$.} \item Soit $(\lambda_1,\dots,\lambda_n)\in \R^n$ tel que $\lambda_1g_1 + \dots + \lambda_n g_n = 0$, c'est-à-dire : \[ \forall Q \in \R_{2n-1}[X],\ \lambda_1Q(x_1) + \dots + \lambda_n Q(x_n) = 0. \] Pour tout $k\in \llbracket 1, n\rrbracket$, on pose $R_k = \dfrac{\displaystyle{\prod_{\underset{i\neq k}{i=1}}^n (X-x_i)}}{\displaystyle{\prod_{\underset{i\neq k}{i=1}}^n (x_k-x_i)}} \in \R_{2n-1}[X]$. On a $R_k(x_k) = 1$ et $R_k(x_i) = 0$ si $i\neq k$. L'évaluation en $Q = R_k$ assure alors que $\lambda_k = 0$ pour tout $k\in \llbracket 1, n\rrbracket$. \fbox{La famille $(g_k)_{1\leqslant k \leqslant n}$ est bien libre.} \item La question revient à montrer que $f \in \Vect(g_1,\dots, g_n)$. Supposons par l'absurde que $f \notin \Vect(g_1,\dots, g_k)$. La famille $(g_1,\dots,g_n,f)$ serait alors libre. Il existerait alors $Q \in \R_{2n-1}[X]$ tel que $g_1(Q) = \dots = g_n(Q) = 0$ et $f(Q) = 1$, ce qui est absurde puisque $Q \in \bigcap_{k=1}^n \Ker g_k$. \fbox{Il existe $(\alpha_1,\dots, \alpha_n) \in \R^n$ tel que : $\forall Q \in \R_{2n-1}[X],\ \int_{-1}^1 Q(x) \dx = \sum_{i=1}^n \alpha_i Q(x_i).$} \item Montrons le résultat par récurrence sur $m\in \N^*$. Pour $m = 1$ : $\varphi_1$ est une forme linéaire non nulle (car elle forme une famille libre), donc il existe $x_1\in E$ tel que $\varphi_1(x_1) = 1$. La propriété est donc initialisée. Soit $m>1$ et supposons une famille de vecteurs $(x_1,\dots,x_{m-1})$ déjà construite. Puisque la famille $(\varphi_1,\dots,\varphi_m)$ est libre, la forme linéaire $\psi = \varphi_m - \sum_{k=1}^{m-1} \varphi_m(u_k) \varphi_k$ est non nulle. Il existe donc $y \in E$ tel que $\psi(y) = 1$. * On pose $u'_m = y - \sum_{k=1}^{m-1} \varphi_k(y) u_k$. Comme $\psi(y)=1$, $\varphi_m(y)=\sum_{k=1}^{m-1} \varphi_k(u_k)\varphi_k(y)$ et $\varphi_m(u'_m)=1$ De plus, si $j0$} \\ \end{cases} $$ \end{comment} \newpage \exos Une puce se déplace sur un axe gradué d'origine $O$ par bonds successifs d'une ou de deux unités suivant la procédure suivante: \bi \item[$\bullet$] au départ la puce est en $O$; \item[$\bullet$] si, à un instant, la puce est au point d'abscisse $k$, à l'instant d'après elle sera soit au point d'abscisse $k+1$, avec la probabilité $\frac{1}{2}$, soit au point d'abscisse $k+2$, avec la probabilité $\frac{1}{2}$; \item[$\bullet$] les sauts sont indépendants. \ei \be \item {\bf Question de cours : }formule des probabilités totales. \item Soit $n\in\N$. On note $S_n$ la variable aléatoire égale au nombre de sauts à deux unités effectués par la puce au cours des $n$ premiers sauts. Déterminer la loi de $S_n$, son espérance et sa variance. \item Soit $n\in\N$. On note $X_n$ la variable aléatoire égale à l'abscisse de la puce après $n$ sauts. Exprimer $X_n$ en fonction de $S_n$. En déduire l'espérance et la variance de $X_n$. \item Soit $n\in\Nstar$. On note $Y_n$ la variable aléatoire égale au nombre minimum de sauts nécessaires pour atteindre ou dépasser (au cas où on ne s'y arrêterait pas) la case d'abscisse $n$. On définit $Y_0$ comme la variable aléatoire certaine égale à $0$. \be \item Déterminer les valeurs prises par $Y_n$. \item Montrer que pour tout entier $n\geq 2$ et pour tout entier $k\geq 1$: $$\P(Y_n=k)=\frac{1}{2}\P(Y_{n-1}=k-1)+\frac{1}{2}\P(Y_{n-2}=k-1)$$ \item En déduire que, pour tout entier $n\geq 2$: $$\E(Y_n)=\frac{1}{2}\E(Y_{n-1})+\frac{1}{2}\E(Y_{n-2})+1$$ \item Déterminer un réel $a$ tel que la suite $(u_n)_{n\in\N}$ définie par $u_n=\E(Y_n)-na$ vérifie une relation de récurrence linéaire d'ordre 2. Déterminer alors $u_n$ puis $\E(Y_n)$ en fonction de $n$. \ee \ee %------------------------------------------------------------------------------------------------------------------ \begin{comment} \be \item Question de cours: programme ECS1 2013 p. 14. \item On note $Z_i$, pour $i\in \N^*$ la variable de Bernoulli égale à $1$ si le $i$-ème saut est de deux unités et à $0$ s'il s'agit d'un saut d'une unité. On a donc $Z_i\sim {\cal B}(1/2)$. Alors $S_n=\sum_{i=1}^nZ_i$ et comme les $Z_i$ sont indépendantes, puisque par hypothèse les sauts le sont, on en déduit que: $$\boxed{S_n\sim {\cal B}(n,1/2)}$$ En particulier: $$\E(S_n)=\frac{n}{2}\text{ et }V(S_n)=\frac{n}{4}$$ \item De manière immédiate, comme $S_n$ est le nombre de sauts de deux unités, et que les $n-S_n$ sauts restants sont de une unité, et que la puce est situé au point d'abscisse nulle au départ: $$X_n=(n-S_n)+2S_n\text{ soit }\boxed{X_n=n+S_n}$$ On en déduit par linéarité de l'espérance que: $$\boxed{\E(X_n)=n+\E(S_n)=\frac{3n}{2}}$$ tandis que par les propriétés de la variance: $$\boxed{V(X_n)=V(S_n)=\frac{n}{4}}$$ \item \be \item La plus grande valeur prise par $Y_n$ est $n$, dans le cas où l'on effectue que des sauts de une unité. L'autre cas extrême correspond à des sauts qui valent tous deux unités et en ce cas, on atteint, ou dépasse la case d'abscisse $n$ en $\left\lfloor\frac{n+1}{2}\right\rfloor$ sauts (si $n=2k$, $\left\lfloor\frac{n+1}{2}\right\rfloor=k$ tandis que si $n=2k+1$, $\left\lfloor\frac{n+1}{2}\right\rfloor=k+1$). Ainsi: $Y_n(\Omega)\subset\left\llb \left\lfloor \frac{n+1}{2} \right\rfloor, n\right\rrb$. Comme tous les cas intermédiaires sont possibles, l'inclusion réciproque est également vérifiée et l'on en déduit finalement: $\boxed{Y_n(\Omega)=\left\llb \left\lfloor \frac{n+1}{2} \right\rfloor, n\right\rrb}$. \item On considère ce qui se passe au premier saut via la variable de Bernoulli $Z_1$ introduit à la question 1: $Z_1=1$ si le premier saut fait deux unités et $0$ sinon. Alors, par la formule des probabilités totales: \begin{eqnarray*} \P(Y_n=k)&=&\P(Y_n=k|Z_1=0)\P(Z_1=0)+\P(Y_n=k|Z_1=1)\P(Z_1=1)\\ &=&\frac{1}{2}\P(Y_n=k|Z_1=0)+\frac{1}{2}\P(Y_n=k|Z_1=1) \end{eqnarray*} De plus: $\P(Y_n=k|Z_1=0)=\P(Y_{n-1}=k-1)$ car une fois effectué un premier saut de une unité, il reste à parcourir une distance de $n-1$ à atteindre en $k-1$ sauts car au départ on voulait parcourir une distance de $n$ unités en $k$ sauts. C'est l'indépendance des sauts et l'invariance du problème par translation sur l'axe gradué qui assure la validité de ce raisonnement. De même: $\P(Y_n=k|Z_1=1)=\P(Y_{n-2}=k-1)$ puisque si le premier saut est de deux unités, il reste $n-2$ unités à parcourir en $k-1$ sauts. Ainsi: $$\boxed{\P(Y_n=k)=\frac{1}{2}\P(Y_{n-1}=k-1)+\frac{1}{2}\P(Y_{n-2}=k-1)}$$ \item Il suffit de calculer en revenant à la définition de l'espérance et en utilisant la relation établie à la question précédente: \begin{eqnarray*} \E(Y_n)&= & \sum_{k= \left\lfloor\frac{n+1}{2} \right\rfloor}^nk\P(Y_n=k)\\ &= & \sum_{k=1}^nk\P(Y_n=k)\text{ car }\P(Y_n=k)=0\text{ pour } k\in \left\llb 1, \left\lfloor\frac{n+1}{2} \right\rfloor-1\right\rrb\\ & = & \frac{1}{2}\sum_{k=1}^nk\left(\P(Y_{n-1}=k-1)+\P(Y_{n-2}=k-1)\right)\\ & = & \frac{1}{2}\sum_{k=1}^nk \P(Y_{n-1}=k-1)+\frac{1}{2}\sum_{k=1}^n k \P(Y_{n-2}=k-1) \end{eqnarray*} Or: \begin{eqnarray*} \frac{1}{2}\sum_{k=1}^nk \P(Y_{n-1}=k-1)&=&\frac{1}{2}\sum_{k=1}^n(k -1)\P(Y_{n-1}=k-1) +\frac{1}{2}\sum_{k=1}^n\P(Y_{n-1}=k-1)\\ & = & \frac{1}{2}\sum_{k=0}^{n-1}k \P(Y_{n-1}=k) +\frac{1}{2}\text{ car } Y_{n-1}(\Omega)\subset \llb 0,n-1\rrb\\ & = & \E(Y_{n-1})+\frac{1}{2} \end{eqnarray*} et de même: \begin{eqnarray*} \frac{1}{2}\sum_{k=1}^nk \P(Y_{n-2}=k-1)&=&\frac{1}{2}\sum_{k=1}^n (k-1 )\P(Y_{n-2}=k-1)+\frac{1}{2}\sum_{k=1}^n \P(Y_{n-2}=k-1)\\ & = & \frac{1}{2}\sum_{k=0}^{n-1}k \P(Y_{n-2}=k) +\frac{1}{2}\text{ car } Y_{n-2}(\Omega)\subset \llb 0,n-1\rrb\\ & = & \E(Y_{n-2})+\frac{1}{2} \end{eqnarray*} On en déduit donc finalement: $$\boxed{\E(Y_n)=\frac{1}{2}\E(Y_{n-1})+\frac{1}{2}\E(Y_{n-2})+1}$$ \item On cherche $a$ tel que: $$an=\frac{1}{2}a(n-1)+\frac{1}{2}a(n-2)+1$$ ce qui revient à $-\frac{3}{2}a+1=0$ soit $\boxed{a=\frac{2}{3}}$. Alors, en posant $u_n=\E(Y_n)-\frac{2}{3}a$, on obtient pour $n\geq 2$: $$u_n=\frac{1}{2}u_{n-1}+\frac{1}{2}u_{n-2}$$ Le trinôme caractéristique associé est $x^2-\frac{1}{2}x-\frac{1}{2}$ qui a deux racines distinctes, dont une évidente: $$r_1=1\quad\text{et}\quad r_2=-\frac{1}{2}$$ Ainsi, pour tout $n$: $$u_n=A+B\left(-\frac{1}{2}\right)^n$$ avec $A$ et $B$ des réels. De plus, de manière immédiate, $Y_0$ et $Y_1$ sont des variables aléatoires certaines avec $Y_0=0$ et $Y_1=1$, d'où $\E(Y_0)=0$ et $\E(Y_1)=1$ et $u_0=0$ et $u_1=\frac{1}{3}$ et donc $A$ et $B$ sont déterminées par le système d'équations: $$\begin{cases} A+B=0\\ A-\frac{1}{2}B=\frac{1}{3} \end{cases}\Longleftrightarrow\begin{cases} A=\frac{2}{9}\\ B=-\frac{2}{9} \end{cases}$$ Finalement: $$ u_n=\frac{2}{9}\left(1-\left(\frac{1}{2}\right)^n\right)$$ et donc: $$\boxed{\forall n\in\N\quad \E(Y_n)=\frac{2}{3}n+\frac{2}{9}\left(1-\left(\frac{1}{2}\right)^n\right)}$$ \ee \ee \end{comment} \newpage \Esps Soit $f$ une fonction continue sur $\R$.\\ On définit par récurrence la suite de fonctions $f^n$ par $f^1=f$ et pour tout $n$ de $\N^*$, $f^{n+1}=f\circ f^n$.\\ Ainsi, $f^n=f\circ f\circ\cdots \circ f$, où la fonction $f$ apparaît $n$ fois. \be\item On suppose qu'il existe $n\in\Nstar$ tel que $f^n$ admette un point fixe. Montrer que $f$ admet un point fixe. \item Proposer une fonction définie sur $\R$ qui n'admette aucun point fixe. \item Proposer une fonction définie sur $\R$ qui admette une infinité de points fixes. \ee \begin{comment} \\ \be\item Par l'absurde, on fait faire un dessin au candidat. (Comme $f$ est continue, c'est en fait une conséquence du TVI, si $f$ n'admet pas de point fixe, le graphe de $f$ est au dessus ou au dessous de la première bissectrice)\\ Supposons que le graphe de $f$ soit au dessus de la première bissectrice alors $$\forall x\in \R \qquad f(x)>x.$$ Mais alors $$ \forall x\in \R \qquad f^n(x)=f(f^{n-1}(x))>f^{n-1}(x)>f^{n-2}(x)>...>x$$ C'est absurde. \fbox{$f$ admet un point fixe.} \item \fbox{$x\mapsto x-1$} \item \fbox{$x\mapsto x\sin(x) $} \ee \includegraphics{Fig22.png} \end{comment} \newpage \exos Soit $\lambda$ un réel strictement positif. \\ On considère $X$ et $Y$ deux variables aléatoires indépendantes, définies sur un même espace probabilisé, suivant toutes les deux une loi exponentielle de paramètre $\lambda$. \begin{enumerate} \item Question de cours~: Rappeler la définition et les propriétés de la fonction de répartition d'une variable aléatoire à densité. \item Soit $\mu > 0$. \begin{enumerate} \item Montrer que la variable $Z = -\mu X$ est une variable aléatoire à densité, et en donner une densité. \item On note $S_\mu$ la variable définie par $S_\mu = Y-\mu X$. \\ Montrer que $S_\mu$ admet pour densité la fonction $f_\mu$ définie par~: \[ \forall x \in \mathbb{R}, \quad f_\mu(x) = \left\{ \begin{array}{ll} \displaystyle \frac{\lambda}{1+\mu} \exp\left( - \lambda x \right) & \text{ si } x \geq 0 \medskip \\ \displaystyle \frac{\lambda}{1+\mu} \exp\left( \frac{\lambda}{\mu} x \right) & \text{ si } x <0 \end{array} \right. \] \end{enumerate} \item On considère le polynôme $Q$ à coefficients aléatoires défini par : \[{\mbox Pour tout réel }t,\;\; Q(t) = t^2+tX-Y . \] \begin{enumerate} \item Vérifier que le polynôme $Q$ admet, avec probabilité $1$, deux racines réelles (aléatoires) distinctes, notées $S$ et $T$ telles que~: \[ S \leqslant 0 \leqslant T. \] \item Justifier que, pour tout réel $t$ positif, on a~: \[ [T \leqslant t] = [Y-tX \leqslant t^2] \] \item En déduire que la variable aléatoire $T$ admet une densité, et en donner une. %\item Déterminer la loi de $S$. \end{enumerate} \end{enumerate} \begin{comment} \begin{enumerate} \item Programme officiel ECS1 page 20. \item %Soit $\mu > 0$. \begin{enumerate} \item %Montrer que la variable $Z = -\mu X$ est une variable aléatoire à densité, et en donner une densité. On a $Z(\Omega)=]-\infty,0]$ et pour $x < 0$, $\displaystyle P(Z \leqslant x) = P\left( - \mu X \leqslant x\right) = P\left( X \geqslant - \frac{x}{\mu} \right) = 1 - F_X\left( \frac{-x}{\mu} \right)$. \\ %\newpage On obtient alors que, pour tout réel $x$, on a~: $\displaystyle F_Z(x) = \left\{ \begin{array}{ll} \exp\left( \lambda \frac{x}{\mu} \right) & \text{ si } x < 0 \\ 1 & \text{ si } x \geqslant 0 \end{array} \right.$ \\ $F_Z$ étant continue sur $\mathbb{R}$, $\mathcal{C}^1$ sur $\mathbb{R}^*$, $Z$ admet une densité $f_Z$ donnée par~: $\boxed{\displaystyle f_Z(x) = \left\{ \begin{array}{ll} \displaystyle \frac{\lambda}{\mu} e^{ \frac{\lambda}{\mu} x} & \text{ si } x < 0 \\ 0 & \text{ sinon} \end{array} \right.}$ \item %On note $S_\mu$ la variable définie par $S_\mu = Y-\mu X$. \\ %Montrer que $S_\mu$ admet pour densité la fonction $f_\mu$ définie par~: %\[ \forall x \in \mathbb{R}, \quad f_\mu(x) = \left\{ \begin{array}{ll} \displaystyle \frac{\lambda}{1+\mu} \exp\left( - \lambda x \right) & \text{ si } x \geqslant 0 \medskip \\ \displaystyle \frac{\lambda}{1+\mu} \exp\left( \frac{\lambda}{\mu} x \right) & \text{ si } x \leqslant 0 \end{array} \right. \] Les variables aléatoires $Y$ et $-\mu X$ étant indépendantes, à densité, la variable aléatoire $S_\mu$ admet bien une densité, donnée par le produit de convolution~: \[ f_\mu(x) = \int_{-\infty}^{+\infty} f_Y(t) f_Z(x-t) dt = \int_{\max(x,0)}^{+\infty} \lambda e^{-\lambda t} \left( \frac{\lambda}{\mu} \right) e^{ \frac{\lambda}{\mu}(x-t)} dt \] \begin{itemize} \item[$\bullet$] si $ x < 0$, on a~: $ \displaystyle f_\mu(x) = \lambda e^{ \frac{\lambda}{\mu} x} \int_{0}^{+\infty} \frac{\lambda}{\mu} e^{ - \frac{\lambda}{\mu}(1+\mu) t} dt = \lambda e^{ \frac{\lambda}{\mu} x} \frac{1}{1+\mu} = \boxed{\frac{\lambda}{1+\mu} e^{ \frac{\lambda}{\mu} x}}$ \item[$\bullet$] si $ x \geqslant 0$, on a~: $\displaystyle f_\mu(x) = \lambda e^{ \frac{\lambda}{\mu} x} \int_{x}^{+\infty} \frac{\lambda}{\mu} e^{ - \frac{\lambda}{\mu}(1+\mu) t} dt = \lambda e^{ \frac{\lambda}{\mu} x} \frac{1}{1+\mu} e^{ - \frac{\lambda}{\mu}(1+\mu) x} \boxed{ = \frac{\lambda}{1+\mu} e^{-\lambda x}}$. \medskip \end{itemize} \end{enumerate} \item %On considère, pour tout réel $t$, le polynôme $Q(t)$ à coefficients aléatoires~: %\[ Q(t) = t^2+tX-Y . \] \begin{enumerate} \item %Vérifier que le polynôme $Q$ admet, avec probabilité $1$, deux racines réelles (aléatoires) distinctes, notées $S$ et $T$ telles que~: %\[ S \leqslant 0 \leqslant T. \] Ici, on a $\Delta = X^2+4Y \geqslant 0$ \\ De plus, $ P(\Delta=0) = P(X^2=0,Y=0) = P(X=0)P(Y=0)=0$, donc presque-sûrement, $\Delta > 0$ et on a donc \fbox{deux racines réelles distinctes} $S$ et $T$. On note S la plus petite. \\ Puisque $ST=-Y \leqslant 0$, les deux racines sont de signes distincts, donc \fbox{$S \leqslant 0 \leqslant T$.} \item %Justifier que, pour tout réel $t$ positif, on a~: %\[ [T \leqslant t] = [Y-tX \leqslant t^2] \] On sait que le polynôme $Q$ est positif sur $]-\infty,S]$ et sur $[T,+\infty[$. Donc en particulier, pour $t$ positif, on a $Q(t) \leqslant 0$ si $t \in [0,T]$ et $Q(t) \geqslant 0$ si $t \in [T,+\infty[$. Ainsi~: \[ \forall t \in \mathbb{R}^+, \quad \boxed{[ T \leqslant t] = [Q(t) \geqslant 0] = [t^2+tX-Y \geqslant 0] = [Y-tX \leqslant t^2] }\] \item %En déduire que la variable aléatoire $T$ admet une densité, et en donner une. Remarquons déjà que $T$ étant positive, on a $T(\Omega) \subset \mathbb{R}^+$. \begin{itemize} \item[$\bullet$] si $t \leqslant 0$, $P(T \leqslant t)=0$. \item[$\bullet$] si $t > 0$, $P(T \leqslant t) = P(Y-tX \leqslant t^2) = \displaystyle \int_{-\infty}^{t^2} f_t(x) dx= \int_{-\infty}^{0} \frac{\lambda}{1+t} e^{ \frac{\lambda}{t} x} dx + \int_{0}^{t^2} \frac{\lambda}{1+t} e^{-\lambda x} dx $\\ $\displaystyle = \frac{\lambda}{1+t} \bigg[ \frac{t}{\lambda} e^{ \frac{\lambda}{t} x } \bigg]_{-\infty}^{0} + \frac{\lambda}{1+t} \bigg[ - \frac{1}{\lambda} e^{-\lambda x} \bigg]_0^{t^2} = \frac{t}{1+t} + \frac{1}{1+t} - \frac{1}{1+t} e^{-\lambda t^2} = 1 - \frac{1}{1+t} e^{-\lambda t^2}$. \end{itemize} Ainsi on a pour tout réel $t$~: $\displaystyle F_T(t) = \left\{ \begin{array}{ll} 0 & \text{ si } t < 0 \\ 1 - \displaystyle \frac{1}{1+t} e^{-\lambda t^2} & \text{ si } t \geqslant 0 \end{array} \right.$. $F_T$ étant continue sur $\mathbb{R}$ et $\mathcal{C}^1$ sur $\mathbb{R}^*$, $T$ admet bien une densité, par exemple~: \[ \boxed{t \mapsto \left\{ \begin{array}{ll} 0 & \text{ si } t < 0 \\\displaystyle \frac{1+2\lambda t + 2 \lambda t^2}{(1+t)^2} e^{-\lambda t^2} & \text{ si } t \geqslant 0 \end{array} \right.} \] %\item On sait déjà que $S(\Omega)\subset ]-\infty,0]$. Pour tout $t \geqslant 0$, $P(S \leqslant t)=1$. \\ %Soit $t < 0$. Avec la même démarche qu'en 3(b), %\[ [S \leqslant t] = [Q(t) \leqslant 0] = [t^2+tX-Y \leqslant 0] = [ Y-tX \geqslant t^2] \] %Donc~: %\[ \forall t < 0, \ P(S \leqslant t) = P(Y-tX \geqslant t^2) \] %Attention que la loi de $Y-tX$ n'a été étudiée que si $t > 0$ dans la question 2.Or ici~: $t < 0$. \\ %Par transformation linéaire, $Z=-t X$ suit encore une loi exponentielle de paramètre $\displaystyle \frac{-\lambda}{t}$. \\ %La variable aléatoire $Y-tX$ est donc positive, de densité donnée par le produit de convolution~: %\begin{align*} %\forall x \geqslant 0, \ g_t(x) & = \int_{-\infty}^{+\infty} f_Y(u) f_{Z}(x-u) du \\ %& = \int_{0}^{+\infty} \lambda e^{-\lambda u} f_Z(x-u) du \\ %& = \int_{0}^{x} \lambda e^{-\lambda (x-v)} f_Z(v) dv \\ %& = \int_{0}^{x} \lambda e^{-\lambda (x-v)} \frac{\lambda}{-t} e^{\frac{\lambda}{t}v} dv \\ %& = \frac{\lambda^2}{-t} e^{-\lambda x} \int_{0}^{x} e^{ \lambda\left( 1 + \frac{1}{t} \right) v} dv \end{align*} %Si $t=-1$, on a $g_t(x) = \displaystyle \lambda^2 x e^{-\lambda x}$. %Si $t \neq -1$, on a~: %\begin{align*} %g_t(x) & = \frac{\lambda^2}{-t} e^{-\lambda x} \cdot \frac{1}{\lambda\left( 1 + \frac{1}{t} \right)} \left( e^{\lambda \left( 1+ \frac{1}{t} \right) x} - 1 \right) \\ %& = \frac{\lambda}{t+1} e^{-\lambda x} \left( 1 - e^{\lambda x + \frac{\lambda}{t} x} \right) \\ %& = \frac{\lambda}{t+1} \left( e^{-\lambda x} - e^{ \frac{\lambda}{t} x} \right) \end{align*} %Ainsi~: %\begin{align*} %\forall t < 0, \ P(S \leqslant t) = P(Y-tX \geqslant t^2) & = \int_{t^2}^{+\infty} \frac{\lambda}{t+1} \left( e^{-\lambda x} - e^{ \frac{\lambda}{t} x} \right) dx\\ %& = \lim\limits_{A \rightarrow+\infty} \bigg[ \frac{-1}{(t+1)} e^{-\lambda x} - \frac{2t }{\lambda (t+1)} \end{align*} \end{enumerate} \end{enumerate} \end{comment} \newpage \Esps On rappelle que pour $i\in\N$, $f^{(i)}$ est la dérivée $i$-ème de $f$. On définit pour chaque $n$ entier naturel la propriété $P_n$ suivante : $$ P_n: "\forall f \in C^{\infty}(\R,\R) , \; \; \exists a \in \R \mbox{ tel que } \prod_{i=0}^n f^{(i)}(a) \geq 0."$$ \be \item Les propositions $P_0$ et $P_1$ sont-elles vraies ? \item Soit $f\in C^{\infty}(\R,\R) $ telle que pour tout $x\in\R$, $f(x)f'(x)f''(x)f^{(3)}(x)<0$.\\ Montrer que $f$ et $f''$ sont toutes les deux à valeurs dans $\mathbb{R}^{+*}$ ou toutes les deux à valeurs dans $\mathbb{R}^{-*}$. \item En déduire que la proposition $P_3$ est vraie. \ee \begin{comment} \be \item \fbox{$P_0$ et $P_1$ sont fausses} : il suffit de choisir $f_0: x\mapsto -e^x$ et $f_1: x\mapsto \exp(-x)$. \item Remarquons que $f, f', f''$ et $f'''$ qui sont continues sur $\mathbb{R}$, sont de signe constant car sinon elle s'annulerait ce qui est contraire à l'hypothèse $f f' f'' f''' < 0.$\\ Si $f''>0$ \: alors $f$\, est convexe,\; donc sa courbe représentative ${\cal{C}}_f$\; est au-dessus de ses tangentes.\vspace{.5mm}\\ En particulier au-dessus de la tangente en $0$ ,\, d'équation \: $y=f'(0)\,.\,x+f(0)$.\\ Comme $f'(0)$\, est non nul \:(\,$f'$\, ne s'annulant pas),\: $y$\, tend vers $+\infty$\, quand $x$\, tend vers $\pm\infty$, selon le signe de \,$f'(0)$.\\ Donc \,$f(x)$\; est positif quand $x$\, tend vers $\pm\infty$,\; donc partout ($f$\, est de signe constant) : \,\quad $f\!>\!0$.\\ $f''<0$ \: implique que $f$\, est concave,\, et un raisonnement analogue montre que : $f\!<\!0$.\\ Donc \fbox{$f$\, et $f''$\, ont donc toujours le même signe,\: et ne s'annulent pas sur \,$\R$.} \item Raisonnons par l'absurde: supposons que de même qu'en 2)~~que $f .f'. f''. f''' < 0$, alors les fonctions $f'$\, et $f'''$ sont de même signe ( même preuve avec $f'$ et $f'''$ au lieu de $f$ et $f''$.)\\ Il en résulte que :\: $f\hspace{.1mm}.\hspace{.2mm}f\,'\hspace{-.5mm}.\hspace{.2mm}f''\hspace{-.5mm}.\hspace{.3mm}f'''\!>0$ ; \: donc contradiction; par suite :\vspace{.5mm}\\ \fbox{ $\exists\; a\!\in\!\R$ \: tel que : \quad $f\!\left(a\right)\,f'\!\left(a\right)\,f''\!\left(a\right)\,f'''\!\left(a\right)\ge0$, $P_3$ est vraie.} \ee \end{comment} \newpage \exos \begin{enumerate} \item \textbf{Question de cours :} Énoncer le théorème de Rolle \item Montrer que, pour tout $a=(a_1,a_2,a_3,a_4)\in \R^4$, il existe un unique polynôme $P_a\in \R_3[X]$ tel que $P_a(0) =a_1,\ P_a'(0) = a_2,\ P_a(1) = a_3$ et $P_a'(1) = a_4$. \item On note $(e_1,e_2,e_3,e_4)$ la base canonique de $\R^4$. Déterminer $P_{e_1}$ On pourrait montrer que $P_{e_2}=X(X-1)^2$, $P_{e_3}= (-2X+3)X^2$ et $P_{e_4}=X^2(X-1)$. \ \\ \ \\ Soit $f$ une fonction réelle de classe $\mathcal{C}^4$ sur $[0,1]$. On note $a(f)=(f(0),f'(0),f(1),f'(1))$ On note alors $Q_f=P_{a(f)}$, l'unique polynôme de $\R_3[X]$ vérifiant $$Q_{f}(0)=f(0),\ Q_{f}'(0) = f'(0),\ Q_{f}(1) = f(1)\text{ et }Q_{f}'(1) = f'(1).$$ \item Montrer que $\int_0^1 Q_{f}(t) \dt= -\dfrac{1}{2}f(0) + \dfrac{1}{12}f'(0) + \dfrac{1}{2}f(1) - \dfrac{1}{12}f'(1)$. \item Soit $x\in]0;1[$ fixé.\\ Notons $C = 4!\dfrac{f(x)-Q_{f}(x)}{x^2(1-x)^2}$. En considérant la fonction $g : t \mapsto f(t) - Q_{f}(t) - \dfrac{C}{4!}t^2(1-t)^2$, montrer qu'il existe $\xi \in ]0,1[$ tel que $f(x) - Q_{f}(x) = \dfrac{x^2(1-x)^2}{4!}f^{(4)}(\xi)$. \item Montrer qu'il existe $M$ une constante que l'on exprimera en fonction des propriétés de $f$ telle que : $$\abs{\int_0^1 f(t)-\pa{-\dfrac{1}{2}f(0) + \dfrac{1}{12}f'(0) + \dfrac{1}{2}f(1) - \dfrac{1}{12}f'(1) \dt}}\leq \dfrac{M}{720}.$$ \end{enumerate} \begin{comment} \begin{enumerate} \item Programme officiel ECS1 page 12. \item Considérons l'application $\phi$, qui à tout $P\in \R_3[X]$ associe $(P(0),P'(0), P(1), P'(1))\in \R^4$. C'est une application linéaire. Étudions son injectivité : un polynôme $P\in \R_3[X]$ appartient au noyau de $\phi$ si, et seulement si 0 et 1 sont racines (au moins) double de $P$, i.e. si et seulement si $P$ divisible par $X^2(X-1)^2$ ; puisque $\deg P\leqslant 3$, cela revient à affirmer que $P=0$. L'application $\phi$ est injective. Puisque $\R_3[X]$ et $\R^4$ sont deux espaces vectoriels de même dimension, $\varphi$ est un isomorphisme. On en déduit donc que pour tout $a=(a_1,a_2,a_3,a_4)\in \R^4$ : \fbox{ il existe un unique polynôme $P\in \R_3[X]$ tel que $P(0) =a_1,\ P'(0) = a_2,\ P(1) = a_3$ et $P'(1) = a_4$.} \item On cherche $P_{1}\in \R_3[X]$ tel que $P_{1}(0)=1$ et $P_{1}'(0) = P_{1}(1) = P_{1}'(1) = 0$. Il est donc de la forme $(X-1)^2(aX+b)$. Puisque $P_1(0)=1$ et $P_1'(0)=0$, il vient que $b = 1$ et $a=2$, i.e. \fbox{$P_{1}=(2X+1)(X-1)^2$.} On trouve de la même manière \fbox{$P_{2}=X(X-1)^2$, $P_{3}= (-2X+3)X^2$ et $P_{4}=X^2(X-1)$.} \item $Q_{f}=\phi^{-1}(a(f))=f(0)\phi^{-1}(e_1)+f'(0)\phi^{-1}(e_2)+f(1)\phi^{-1}(e_3)+f'(1)\phi^{-1}(e_4)$ par linéarité de $\phi^{-1}$. Ainsi $Q_{f} = f(0)P_1+f'(0)P_2 + f(1)P_3 + f'(1)P_4$. On en déduit que, par linéarité de l'intégrale : \begin{align*} \int_0^1 Q_{f}(t) \dt &= f(0)\int_0^1P_{1}(t) \dt +f'(0)\int_0^1P_{2}(t)\dt + f(1)\int_0^1P_{3}(t)\dt + f'(1)\int_0^1P_{4}(t)\dt \\ &\boxed{= -\dfrac{1}{2}f(0) + \dfrac{1}{12}f'(0) + \dfrac{1}{2}f(1) - \dfrac{1}{12}f'(1).} \end{align*} \item Remarquons que $g$ est de classe $\mathcal{C}^4$ sur $[0,1]$. Puisque $g(0) = g(1) = g(x) = 0$, on peut appliquer le théorème de Rolle à $g$ sur les intervalles $]0,x[$ et $]x,1[$ : il existe $a \in ]0,x[$ et $b\in]x,1[$ tel que $g'(a) = g'(b) = 0$. On a de plus $g'(0) = g'(1)=0$. \fbox{$g'$ s'annule au moins 4 fois dans $[0;1]$} En appliquant successivement le théorème de Rolle à $g'$, $g''$ et $g^{(3)}$, on montre que $g''$ s'annule s'annule (au moins) trois fois sur $]0,1[$, puis $g^{(3)}$ s'annule au moins deux fois sur $]0,1[$, et enfin que $g^{(4)}=f^{(4)} - C$ s'annule au moins une fois sur $]0,1[$. \fbox{Il existe donc $\xi \in ]0,1[$ tel que $f^{(4)}(\xi) = C$, i.e. $f(x) - Q_{f}(x) = \dfrac{x^2(1-x)^2}{4!}f^{(4)}(\xi)$.} \item Notons $M = \underset{x \in [0,1]}{\max} |f^{4}(x)|$ ($M$ existe par continuité de $f^{(4)}$ sur $[0,1]$). D'après la question 5.(b), on a : \[ \forall x \in ]0,1[,\ |f(x) - Q_{f}(x)| \leqslant \dfrac{x^2(1-x)^2}{4!}M. \] Par continuité de $f$ et $Q_{f}$ en 0 et 1, cette majoration est toujours valable en 0 et en 1. L'inégalité triangulaire assure alors que : \[ \left|\int_0^1 f(x) \dx - \int_0^1 Q_{f}(x) \dx \right| \leqslant \int_0^1 \left|f(x) -Q_{f}(x)\right| \dx = M \int_0^1 \dfrac{x^2(1-x)^2}{4!} \dx=\frac{M}{30\times 24}= \boxed{\dfrac{M}{720}.} \] \end{enumerate} \end{comment} \newpage \Esps Donner la finalit\'e du progamme suivant: \begin{verbatim} N=100000;S=0; for i=1:N u=rand(); S=S+4/N*1/(1+u^2); end disp(S) \end{verbatim} \begin{comment} Le programme donne une moyenne empirique d'un échantillon de variables ayant la loi de ${Y={4\over{1+X^2}}}$ o\`u $X$ suit une loi uniforme sur [0,1]. Par la loi des grand nombre, ce programme approche $\E(Y)=4\int_0^1{1\over {1+x^2}}\dx$ Le programme permet donc \fbox{d'approcher $\pi$ en utilisant la loi des grands nombres. (méthode de Monté-Carlo)} {\it Question: subsidiaire: vaut-il mieux utiliser cette méthode ou une méthode des rectangles à 100 termes pour approcher $\pi$?} \end{comment} \newpage \exos \be \item {\bf Question de cours :} Somme de deux variables aléatoires à densité indépendantes. \item Soit $(X_k)_{k\geq 1}$ une famille de variables indépendantes suivant chacune une même loi exponentielles de paramètre $\lambda>0$.\\ On pose $S_n=\sum_{k=1}^nX_k$ pour $n\in\N^*$. Montrer qu'une densité de $S_n$ est la fonction $f_n$ définie par: $$f_n(x)=\begin{cases} \frac{\lambda^nx^{n-1}\e^{-\lambda x}}{(n-1)!}&{\rm si\ }x\geq 0 \cr 0&{\rm si\ }x<0\end{cases}$$ \item On note, pour $n\in\Nstar$, $F_n$ la fonction de répartition de $S_n$. Montrer que, pour $t\geq 0$: $$F_n(t)=1-\e^{-\lambda t}\pa{1+\frac{\lambda t}{1!}+\ldots+\frac{(\lambda t)^{n-1}}{(n-1)!}}$$ %\item En déduire $\P(S_n\leq t, S_{n+1}>t)$ \item Soit $t\geq 0$. On désigne par $N(t)$ le plus grand entier $n$ tel que $S_n\leq t$.\\ Si $S_1>t$, on pose $N(t)=0$. \\ Déterminer la loi de $N(t)$. \item On considère maintenant une variable aléatoire $R$ de loi géométrique de paramètre $p\in]0,1[$. On pose $q=1-p$. On suppose que $R$ est indépendante de chacune des variables $X_k$, $k\in\N^*$ et l'on pose $S=\sum_{k=1}^RX_k$. \be \item Déterminer la fonction de répartition de $S$. \item En déduire la loi de $S$ et montrer que $\E(S)=\E(X_1)\E(R)$. \ee \ee %------------------------------------------------------------------------------------------------------------------ \begin{comment} \be \item Question de cours: programme ECS2 2014 p. 14. \item On va prouver le résultat par récurrence sur $n$. Pour $n=1$, de manière immédiate: $$f_1(x)=\begin{cases}\lambda \e^{-\lambda x}&{\rm si\ }x\geq 0 \cr 0&{\rm si\ }x<0\end{cases}$$ et le résultat est vrai. Soit donc $n\geq 1$ tel que le résultat demandé soit vrai au rang $n$. Alors, par le lemme des coalitions $S_n$ et $X_{n+1}$ sont indépendantes, $S_{n+1}$ est à densité, et : $$f_{n+1}(x)=\int_{-\infty}^{+\infty}f_n(t)f_1(x-t)\dt$$ Compte-tenu du support des densités $f_n$ et $f$, $f_{n+1}(x)=0$ pour $x\leq 0$ et, pour $x>0$: \begin{eqnarray*} f_{n+1}(x)&=& \int_0^xf_n(t)f_1(x-t)\dt\\ & =& \int_0^x \frac{\lambda^nt^{n-1}\e^{-\lambda t}}{(n-1)!}\lambda \e^{-\lambda(x-t)}\dt\\ & =& \lambda^{n+1}e^{-\lambda x}\int_0^x\frac{t^{n-1}}{(n-1)!}\dt\\ &= & \lambda^{n+1}\e^{-\lambda x}\frac{x^{n}}{n!} \end{eqnarray*} et le résultat est vrai au rang $n+1$. \item On démontre là encore le résultat par récurrence sur $n$. Pour $n=1$ le résultat est vrai car $F_1(t)=1-e^{-\lambda t}$ est bien la fonction de répartition d'une loi exponentielle de paramètre $\lambda$. Soit donc $n\geq $ tel que le résultat soit vrai au rang $n$. Alors, pour $t\geq 0$: \begin{eqnarray*} F_{n+1}(t)&= & \P(S_{n+1}\leq t)\\ &=& \int_0^tf_{n+1}(x)\dx\\ & =& \int_0^t \frac{\lambda^{n+1}x^{n}\e^{-\lambda x}}{n!}\dx\\ & =& \crocint{-\lambda^{n}e^{-\lambda x}\frac{x^{n}}{n!}}_0^t+\int_0^t \frac{\lambda^{n}x^{n-1}\e^{-\lambda x}}{(n-1)!}\dx\text{ par intégration par parties}\\ &= & -\lambda^{n}e^{-\lambda t}\frac{t^{n}}{n!}+F_n(t)\\ &= & 1-\e^{-\lambda t}\pa{1+\frac{\lambda t}{1!}+\ldots+\frac{(\lambda t)^{n-1}}{(n-1)!}+\frac{(\lambda t)^{n}}{n!}} \end{eqnarray*} par hypothèse de récurrence. Le résultat est ainsi prouvé. On peut également dériver la fonction $F_n$ dont l'expression est donnée dans l'énoncé et vérifier que $F_n'=f_n$. \item Soit $n\in\N$. Alors : $\P(N(t)=n)=\P(S_n\leq t, S_{n+1}>t)$ car les variables $X_k$ sont à valeurs positives. De même toujours par positivité des $X_k$: $$\acco{S_{n+1}>t}=\acco{S_n>t}\cup\acco{S_n\leq t, S_{n+1}>t}$$ et l'union des deux événements dans le second membre est disjointe. On a donc: $$\P(S_n\leq t, S_{n+1}>t)=\P(S_{n+1}>t)-\P(S_n>t)=F_{n}(t)-F_{n+1}(t) $$ Finalement, compte-tenu de la question précédente: $$\boxed{\P(N(t)=n)=e^{-\lambda t}\frac{(\lambda t)^{n}}{n!}}$$ On reconnaît une loi de Poisson de paramètre $\lambda t$. En particulier, cela prouve que $N(t)$ est presque sûrement fini. \item \be \item Par la formule des probabilités totales: \begin{eqnarray*} \P(S\leq x)& =& \sum_{r\geq 1} \P_{R=r}(S\leq x)\P(R=r)\\ & = & \sum_{r\geq 1} \P(S_r\leq x)\P(R=r)\\ & =& \sum_{r\geq 1} \pa{1-e^{-\lambda x}\sum_{k=0}^{r-1}\frac{(\lambda x)^k}{k!}}q^{r-1}p\\ &= &1-pe^{-\lambda x}\sum_{r\geq 1}\sum_{k=0}^{r-1}\frac{(\lambda x)^k}{k!}q^{r-1}\\ & =& 1-pe^{-\lambda x}\sum_{k\geq 0}\sum_{r\geq k+1}\frac{(\lambda x)^k}{k!}q^{r-1}\text{ en intervertissant l'ordre de sommation}\\ & = & 1-pe^{-\lambda x}\sum_{k\geq 0}\frac{(\lambda x)^k}{k!}\frac{q^k}{1-q}\\ & = & 1-e^{-\lambda x}\sum_{k\geq 0}\frac{(q\lambda x)^k}{k!}\text{ car }1-q=p\\ & = & 1-e^{-\lambda x}e^{q\lambda x}\\ &=&\boxed{1-e^{-\lambda px}} \end{eqnarray*} \item On reconnaît la fonction de répartition \fbox{d'une loi exponentielle de paramètre $\lambda p$}. En particulier \fbox{$\E(S)=\frac{1}{\lambda p}=\E(X_1)\E(R)$.} \ee \ee \end{comment} \newpage \Esps Soient $n\in\Nstar$ et $A \in \M_n(\R)$ une matrice diagonalisable. On pose $B= A^3+A+I_n$.\\ Montrer qu'il existe un polynôme $Q\in\R[X]$ tel que $A=Q(B)$. \begin{comment} La fonction $f : x \mapsto x^3+x+1$ est strictement croissante sur $\R$ donc injective.\\ Notons $\lambda_1,\dots,\lambda_p$ les valeurs propres de $A$, de multiplicités respectives $m_1,\dots,m_p$. Il existe alors une matrice $P$ inversible telle que $A=PDP^{-1}$ où : \[ D = \begin{pmatrix} \lambda_1 I_{m_1} & & (0) \\ & \ddots & \\ (0) & & \lambda_p I_{m_p} \end{pmatrix} = \mathrm{diag}(\lambda_1 I_{m_1}, \dots, \lambda_p I_{m_p}). \] On a alors $B = f(A) = Pf(D)P^{-1} = P\Delta P^{-1}$ où : \[ \Delta = \mathrm{diag}(f(\lambda_1) I_{m_1}, \dots, f(\lambda_p) I_{m_p}). \] Pour tout $i\in \llbracket 1, p\rrbracket$, notons $\mu_i = f(\lambda_i)$.\\ Puisque $f$ est injective, les réels $\mu_1,\dots, \mu_p$ sont deux-à-deux distincts. Le polynôme $Q = \sum_{i=1}^p \lambda_i L_i$, où \[ L_i= \dfrac{\displaystyle{\prod_{\underset{k\neq i}{k=1}}^p (X-\mu_k)}}{\displaystyle{\prod_{\underset{k\neq i}{k=1}}^p (\mu_i-\mu_k)}} \] vérifie $Q(\mu_i) = \lambda_i$ pour tout $i\in \llbracket 1, p\rrbracket$. On a alors : $Q(B) = PQ(\Delta) P^{-1} = PDP^{-1} = A$. \fbox{il existe un polynôme $Q\in\R[X]$ tel que $A=Q(B)$.} \textit{On pourra demander aux plus rapides si la propriété est-elle toujours vraie lorsque $A \in \M_n(\C)$.}\\ Le résultat n'est plus vrai en toutes généralités si $A \in \M_n(\C)$ : la fonction $f : z \mapsto z^3+z+1$ n'est pas injective sur $\C$. Il existe donc deux complexes distincts $\alpha$ et $\beta$ tels que $f(\alpha) = f(\beta) \, (= \gamma)$. Si $A = \begin{pmatrix} \alpha & 0 \\ 0 & \beta \end{pmatrix}$, alors $B = \begin{pmatrix} f(\alpha) & 0 \\ 0 & f(\beta) \end{pmatrix} = \gamma I_2$. La matrice $B$ est donc scalaire alors que $A$ ne l'est pas. \fbox{La propriété est fausse lorsque l'on choisit $A\in\M_n(\C)$, sauf si $n=1$.} \end{comment} \newpage \exos \begin{enumerate} \item {\bf Question de cours~:} définition de la convergence absolue d'une série. Lien avec la convergence. \item Soit $x$ un réel. Montrer que la série $\displaystyle \sum\limits_{n \geqslant 0} \frac{x^n}{(n!)^2}$ converge. \\ On note alors~: \[ \forall x \in \mathbb{R}, \ f(x) = \displaystyle \sum\limits_{n=0}^{+\infty} \frac{x^n}{(n!)^2}.\] \item Soient $x$ et $y$ deux réels positifs. Notons $z=\max(x,y)$. Montrer que~: \[ \left| f(x)- f(y) \right| \leqslant e^z |x-y| \] En déduire que $f$ est continue sur $[0,+\infty[$. \item Montrer que pour tout $x > 0$, on a~: $f(x) > 1$ et que~ : $f(x)-1 \underset{x \rightarrow 0}{\sim} x$. \item On pose pour tout $x > 0$, \[ g(x) = \int_{1}^{x} \displaystyle \frac{1}{t (f(t))^2} \text{d} t. \] \begin{enumerate} \item Justifier que $g$ est bien définie sur $]0,+\infty[$. \item Montrer que~: \[ g(x) \underset{x \rightarrow 0}{\sim} \ln(x) \] \end{enumerate} \end{enumerate} \begin{comment} \begin{enumerate} \item Programme officiel ECS1 page 18. \item Soit $x$ un réel. Alors~: $\forall n \geqslant 0, \ 0 \leqslant \left| \frac{x^n}{(n!)^2} \right| = \frac{|x|^n}{(n!)^2} \leqslant \frac{|x|^n}{n!}$. \\ Comme la série exponentielle $\displaystyle \sum \frac{|x|^n}{n!}$ converge, on en déduit par critère de majoration de suites positives que \fbox{la série $\displaystyle \sum\limits_{n \geqslant 0} \frac{x^n}{(n!)^2}$ converge absolument, donc converge.} \item Soient $x$ et $y$ deux réels positifs. On a (toutes les séries étant bien convergentes)~: \begin{small} \begin{align*} \hspace{-1.5cm} \left| f(x)-f(y) \right| = \left| \sum\limits_{n=0}^{+\infty} \frac{x^n-y^n}{(n!)^2} \right| = \left| \sum\limits_{n=1}^{+\infty} \frac{(x-y)\sum\limits_{k=0}^{n-1} x^k y^{n-1-k}}{(n!)^2} \right| \leqslant |x-y| \sum\limits_{n=1}^{+\infty} \frac{ n z^{n-1}}{(n!)^2} \leqslant |x-y| \sum\limits_{n=0}^{+\infty} \frac{z^n}{n!} = \boxed{e^z |x-y|} \end{align*} \end{small} (on peut aussi appliquer l'IAF à $t\mapsto t^n$ dans l'intervalle $[x;y]$ ou $[y;x]$) Comme à $x$ fixé, $y\mapsto \max(x,y)$ est continue, on a bien : $\lim\limits_{y \rightarrow x}e^{\max(x,y)}\abs{x-y}=0$ Pour tout réel $x$ positif, on a donc ~: $\lim\limits_{y \rightarrow x} |f(x)-f(y)| = 0$ par encadrement. \\ Autrement dit, \fbox{$f$ est continue en tout réel $x$ de $[0,+\infty[$. } \item Soit $x > 0$. Remarquons que $f(x) = 1 + \displaystyle x + \sum\limits_{n=2}^{+\infty} \frac{x^n}{(n!)^2} \geqslant 1+ x > 1$. \\ De plus, pour $x$ proche de $0$, $x$ non nul, on a~: \[ \displaystyle \frac{f(x)-1}{x} = \frac{1}{x} \sum\limits_{n=1}^{+\infty} \frac{x^n}{(n!)^2} = \sum\limits_{n=1}^{+\infty} \frac{x^{n-1}}{(n!)^2} = \sum\limits_{k=0}^{+\infty} \frac{x^k}{((k+1)!)^2} = 1 + \sum\limits_{k=1}^{+\infty} \frac{x^k}{(k+1)!)^2}\] Or, \[ \left| \sum\limits_{k=1}^{+\infty} \frac{x^k}{(k+1)!)^2} \right| \leqslant \sum\limits_{k=1}^{+\infty} \frac{|x|^k}{k!} = e^{|x|}-1 \] Donc par encadrement, $\lim\limits_{x \rightarrow 0} \sum\limits_{k=1}^{+\infty} \frac{x^k}{(k+1)!)^2} =0$, et on a bien~: \[ \lim\limits_{x \rightarrow 0} \frac{f(x)-1}{x} = 1 \Longrightarrow\boxed{ f(x) -1 \underset{x \rightarrow 0}{\sim} x} \] \item \begin{enumerate} \item Soit $x > 0$. Alors la fonction $t \mapsto \displaystyle \frac{1}{t (f(t))^2}$ est continue sur $[1,x]$ (ou $[x,1]$) puisque $f$ est continue et ne s'annule pas sur $[1,x]$ (ou $[x,1]$). Ainsi, \fbox{ $g(x)$ est bien défini pour tout réel $x > 0$. } \item Remarquons alors que~: \[ g(x) - \ln(x) = \int_{1}^{x} \frac{1}{t (f(t))^2} \text{d} t - \int_{1}^{x} \frac{1}{t} \text{d} t = \int_{1}^{x} \frac{1-(f(t))^2}{t (f(t))^2} \text{d} t = \int_{x}^{1} \frac{f(t)-1}{t} \frac{f(t)+1}{(f(t))^2} \text{d} t \] Or, remarquons que~: $\frac{f(t)-1}{t} \underset{t \rightarrow 0^+}{\longrightarrow} 1$ d'après 4, et $\displaystyle \frac{f(t)+1}{(f(t))^2} \underset{t \rightarrow 0^+}{\longrightarrow} 2$. \\ Ainsi, la fonction $t \mapsto \displaystyle \frac{f(t)-1}{t} \frac{f(t)+1}{(f(t))^2}$ est prolongeable par continuité en $0$. \\ L'intégrale $\displaystyle \int_{0}^{1} \frac{f(t)-1}{t} \frac{f(t)+1}{(f(t))^2}$ est donc convergente car faussement impropre. En notant $I$ sa valeur, on a donc~: \[ g(x) - \ln(x) = I + \underset{x \rightarrow 0}{o}(1) \Longrightarrow g(x) = \ln(x) + I + \underset{x \rightarrow 0}{o}(1) \Longrightarrow \boxed{g(x) \underset{x \rightarrow 0}{\sim} \ln(x)} \] \end{enumerate} \end{enumerate} \end{comment} \newpage \Esps Soit $(\Omega, \A, \P)$ un espace probabilisé et $(X,Y)$ un vecteur aléatoire défini sur $\Omega$ tel que : \bi \item $(X,Y)(\Omega)=\{ (0 , 0), (0 , 1), (1 , 1)\} \cup \{(n , 0)| n \in \N\cap [2, +\infty[ \}$ \item $X$ suit une loi de Poisson de paramètre $\frac{1}{\sqrt 3}$ \item $Y$ suit une loi de Bernouilli de paramètre $\frac{1}{\sqrt 3}$ \ei \be \item Les deux variables aléatoires $X$ et $Y$ sont-elles indépendantes ? Déterminer la loi du vecteur $(X, Y)$.\\ \item Montrer que $ \P( X \neq Y) \leq \frac{2}{3}$.\\ \item Soit $A$ une partie de $\N$. Montrer que : $$ \mid \P(X \in A) - \P(Y \in A) \mid \leq \frac{2}{3} $$ \ee \begin{comment} \be \item Les variables $X$ et $Y$ \fbox{ne sont pas indépendantes} en effet $\mathbb{P(}( X = 1 \cap Y = 0) = 0 \neq \P( X = 1 ) \P( Y = 0 )$ .\\ Notons $p = \frac{1}{\sqrt{3}}$.\\ On a, par définition des lois marginales: $\boxed{\P \left ( (X,Y)=(1,1) \right )=P(X=1)= p e^{- p}}$\\ et $\P \left ( (X,Y)=(0,1) \right ) + \P \left ( (X,Y)=(1,1) \right ) = P(Y=1)=p$ d'où : $ \boxed{\P \left ( (X,Y)=(0,1) \right ) = p - p e^{- p}}$ $\P \left ( (X,Y)=(0,1) \right ) + \P \left ( (X,Y)=(0,0) \right )=P(X=0) = e^{- p}$ d'où $ \boxed{\P \left ( (X,Y)=(0,0) \right )= p e^{- p} + e^{- p} - p}$ Enfin pour tout $n$ supérieur ou égal à $2 $ on a: $\boxed{ \P \left ( (X,Y)=(n,0) \right )= p^n e^{ - p} / n!}.$ Mais attention: pour prouver que qu'une telle loi existe il faut vérifier que la somme des proba vaut $1$ : $$ pe^{-p} + p -pe^{-p} + p e^{-p} -p + e^{-p} + \sum_{n=2}^{+ \infty} p^n e^{-p} / n! = 1$$ \item On a: $$\P( X \neq Y) = 1 - \P(X = Y) = 1 -\P \left ( (X,Y)=(1,1) \right )-\P \left ( (X,Y)=(0,0) \right ) = 1 - (e^{-p} + p e^{-p} -p +p e^{-p}) $$ or, par convexité de la fonction $f(x) =e^{-x} $ on a: $e^{-p} \geq 1 - p$. D'où: $$\P( X \neq Y) = 1 + p - e^{-p} (2p + 1) \leq 1 + p - 2p (1 - p) = 2 p^2 = \boxed{\frac{2}{3}}$$ \item $\P(X \in A) = \P( [X \in A] \cap (Y \in A] ) + \P( [X \in A] \cap (Y \in \overline{A} ] ) $ et de même\\ $\P(Y \in A) = \P( [X \in A] \cap (Y \in A] ) + \P( [Y \in A] \cap (X \in \overline{A} ] ) $ d'où par soustraction $$\abs{\P(X \in A) - \P (Y \in A ) } =\abs{\P( [X \in A] \cap (Y \in \overline{A} ] ) - \P( [Y \in A] \cap (X \in \overline{A} ] )} \leq \P( [X \in A] \cap (Y \in \overline{A} ] ) + \P( [Y \in A] \cap (X \in \overline{A} ] ) $$ or l'évènement $[X \in A] \cap (Y \in \overline{A} ]\cap [Y \in A] \cap (X \in \overline{A} ] \subset [X\neq Y]$ et donc : $$\boxed{ \abs{\P(X \in A) - \P (Y \in A ) }\leq \P( X \neq Y)\leq \frac23}$$ \ee \end{comment} \newpage \par\advance\planchecount by1 \exos On considère l'équation: \begin{equation}\label{equat}\tag{$E$} 1-5x=2x^2\ln x \end{equation} \be \item {\bf Question de cours : }énoncer l'inégalité des accroissements finis. \item Soit $\varphi$ définie sur $]0,+\infty[$ par $\varphi(x)=\frac{1}{x^2}-\frac{5}{x}-2\ln x$. En étudiant $\varphi$, montrer que l'équation (\ref{equat}) admet une unique solution $\alpha$ et justifier que $\alpha\in\left]0,\frac{1}{2}\right[$. \item Soit $f$ définie pour $x\in]0,+\infty[$ par: $$f(x)=\frac{1-x-2x^2\ln x}{4}$$ \be \item Montrer que l'on peut prolonger $f$ par une fonction continue et dérivable sur $[0,+\infty[$. Préciser alors $f(0)$ et $f'(0)$. \item On rappelle que $0.69<\ln(2)<0.7$. \'Etudier les variations de $f'$ et de $f$ sur $[0,1]$. \ee \item Le but de cette question est de déterminer une valeur approchée de $\alpha$. Pour cela on définit la suite $(u_n)_{n\geq 0}$ par: $$u_0=\frac{1}{5}\quad\text{et}\quad u_{n+1}=f(u_n)\text{ pour }n\geq 0$$ \be \item Justifier que: $\forall n\in\N\quad |u_{n+1}-\alpha|\leq \frac{3}{4}|u_n-\alpha|$. \item En déduire que la suite $(u_n)_{n\geq 0}$ converge. \item Comment utiliser cette suite $(u_n)$ pour obtenir une valeur approchée de $\alpha$ à $10^{-5}$ près? \item \'Ecrire un programme Scilab qui permette d'obtenir une telle valeur approchée à $10^{-5}$ près. \ee \ee %------------------------------------------------------------------------------------------------------------------ \begin{comment} \be \item Question de cours: programme ECS1 2013 p. 12. \item La fonction $\varphi$ est dérivable sur son ensemble de définition et, pour $x>0$: $$\varphi'(x)=-\frac{2}{x^3}+\frac{5}{x^2}-\frac{2}{x}=-\frac{2x^2-5x+2}{x^3}=-\frac{(2x-1)(x-2)}{x^3}$$ On a donc $\varphi'(x)=0$ si et seulement $x=2$ ou $x=\frac{1}{2}$. De plus $\varphi'(x)>0$ si et seulement si $x\in\left]\frac{1}{2},2\right[$. La fonction est donc strictement décroissante sur $]0,1/2]$, strictement croissante sur $[1/2,2]$ puis à nouveau strictement décroissante sur $[2,+\infty[$. On remarque que $\varphi(2)=-\frac{9}{4}-2\ln 2$ d'où, par l'étude des variations, le fait que, pour tout $x\geq\frac{1}{2}$, $\varphi(x)<0$. En particulier, $\varphi$ ne s'annule pas sur $\left[\frac{1}{2},+\infty\right[$. Par ailleurs, $\varphi$ est continue strictement décroissante sur $\left]0,\frac{1}{2}\right[$, elle induit donc une bijection de $\left]0,\frac{1}{2}\right[$ sur $]\varphi\left(\frac{1}{2}\right),+\infty[$ en vertu du théorème de la bijection. De plus $0\in]\varphi\left(\frac{1}{2}\right),+\infty[$ car $\varphi\left(\frac{1}{2}\right)<0$. On en déduit donc qu'il existe un unique $\alpha\in\left]0,\frac{1}{2}\right[$ tel que $\varphi(\alpha)=0$. De plus c'est l'unique solution de l'équation $\varphi(x)=0$ sur $]0,+\infty[$ par ce qui précède. Enfin, l'équation (\ref{equat}) est équivalente à l'équation $\varphi(x)=0$ sur $]0,+\infty[$. On en déduit donc que \underline{l'équation (\ref{equat}) admet une unique solution $\alpha$ sur $]0,+\infty[$ et que $\alpha\in\left]0,\frac{1}{2}\right[$}. \item \be \item Par croissance comparée, $\lim\limits_{x\to 0}x^2\ln x=0$ d'où $\lim\limits_{x\to 0}f(x)=\frac{1}{4}$. On peut donc prolonger $f$ par continuité en $0$ en posant $\boxed{f(0)=\frac{1}{4}}$. Le taux d'accroissement en $0$ est alors: $$\frac{f(x)-\frac{1}{4}}{x}=-\frac{1+2x\ln x}{4}$$ Encore par croissance comparée, on obtient $\lim\limits_{x\to 0}\frac{f(x)-\frac{1}{4}}{x}=-\frac{1}{4}$. Ainsi la fonction est dérivable en $0$ avec $f'(0)=-\frac{1}{4}$. La dérivabilité sur $]0,+\infty[$ ne pose pas de problème et $f$ est donc bien dérivable sur $[0,+\infty[$. \item $\forall x\in ]0,+\infty[$, $f'(x)=\frac{-1}4-x\ln(x)-\frac{x}{2}$. La fonction $f'$ est dérivable sur $]0,+\infty[$ et pour $x>0$: $f''(x)=-\frac{3}{2}-\ln(x)$ Ainsi $f''(x)=0$ si et seulement si $x=e^{-\frac{3}{2}}\in]0,1]$. De plus $f''(x)<0$ pour $x\in \left]e^{-\frac{3}{2}},1\right]$ et $f''(x)>0$ si et seulement si $x\in\left]0,e^{-\frac{3}{2}}\right[$. De plus, $\lim\limits_{x\to 0}f'(x)=0$, donc $f'$ est continue en 0. Ainsi $f'$ est strictement croissante sur $\left[0,e^{-\frac{3}{2}}\right]$ puis strictement décroissante sur $\left[e^{-\frac{3}{2}},1\right]$. La fonction $f'$ admet donc un maximum en $x=e^{-\frac{3}{2}}$ et ce maximum est égal à $e^{-\frac{3}{2}}-\frac{1}{4}$. Ce nombre est négatif car $3>4\ln(2)$. On a donc $f'(x)<0$ pour tout $x\in[0,1]$. De plus, comme $f'(0)=-\frac{1}{4}$ et $f'(1)=-\frac{3}{4}$, on en déduit que: $$\boxed{\forall x\in[0,1]\quad |f'(x)|\leq \frac{3}{4}}$$ Comme $f'<0$ sur $[0,1]$, $f$ y est strictement décroissante et comme $f(0)=\frac{1}{4}$ tandis que $f(1)=0$, on en déduit que: $$\boxed{\forall x\in[0,1]\quad f(x)\in[0,1]}$$ \ee \item \be \item On commence par montrer par récurrence sur $n$ que $u_n$ est bien défini avec $u_n\in[0,1]$ pour tout $n$. Pour $n=0$ le résultat est évident. Soit $n\geq 0$ tel que $u_n$ est bien défini avec $u_n\in[0,1]$. Alors $u_{n+1}=f(u_n)$ est bien défini car $f$ est définie sur $[0,1]$, et d'après la question 2.c), $u_{n+1}=f(u_n)\in[0,1]$. Cela prouve le résultat au rang $n+1$. On en déduit donc le résultat annoncé par récurrence. En particulier: $\boxed{\forall n\in\N\quad u_n\in[0,1]}$. Alors d'après la question 2.c), $\forall x\in[0,1]\quad |f'(x)|\leq \frac{3}{4}$, donc par l'inégalité des accroissements finis: $$\forall n\in\N\quad |f(u_{n})-f(\alpha)|\leq \frac{3}{4}|u_n-\alpha|$$ soit $$\boxed{\forall n\in\N\quad |u_{n+1}-\alpha|\leq \frac{3}{4}|u_n-\alpha|}$$ \item On montrer par récurrence, que, pour $n\in\N$: $$|u_{n}-\alpha|\leq \left(\frac{3}{4}\right)^n$$ Le résultat est trivial pour $n=0$ car $u_0$ et $\alpha$ sont dans $[0,1]$. On suppose qu'il est vrai au rang $n\geq 0$. Alors, par la question précédente: $$|u_{n+1}-\alpha|\leq \frac{3}{4}|u_n-\alpha|\leq \frac{3}{4} \left(\frac{3}{4}\right)^n= \left(\frac{3}{4}\right)^{n+1}$$ en utilisant l'hypothèse de récurrence. On en déduit donc par récurrence: $$\forall n\in\N\quad |u_{n}-\alpha|\leq \left(\frac{3}{4}\right)^n$$ Alors, comme $0<\frac{3}{4}<1$, par théorème de comparaison, \underline{$(u_n)$ est convergente} et $\boxed{\lim\limits_{n\rightarrow +\infty} u_n=\alpha}$. \item Comme: $$\forall n\in\N\quad |u_{n}-\alpha|\leq \left(\frac{3}{4}\right)^n$$ il suffit de trouver $n$ tel que $\left(\frac{3}{4}\right)^n<10^{-5}$ pour obtenir une valeur approchée de $\alpha$ à $10^{-5}$ près. Cela revient à choisir $n$ tel que $\displaystyle n>-5\frac{\ln(10)}{\ln\left(\frac{3}{4}\right)}$. Pour une telle valeur de $n$, $u_n$ est une valeur approchée de $\alpha$ à $10^{-5}$ près. \item Une possibilité est le code suivant, qui en plus de calculer la valeur approchée de $\alpha$ renvoie également la valeur de l'indice $n$ du terme $u_n$ étant l'approximation recherchée: \\ \texttt{u=1/5} \texttt{n=0} \texttt{geo=1} \texttt{while geo>10\^\!(-5)} \quad\quad\texttt{u=(1-u-2*u\^\!\,2*log(u))/4} \quad\quad\texttt{geo=geo*3/4} \quad\quad\texttt{n=n+1} \texttt{end} \texttt{disp(n,u)} \ee \ee \end{comment} \newpage \Esps Un joueur lance simultanément $N$ dés équilibrés. Puis, il effectue un deuxième lancer en ne relançant que les dés qui n'ont pas donné 6. Il continue ainsi, en ne relançant à chaque tirage que les dés n'ayant jamais donné 6. On note, pour $n\in\N^*$, $S_n$ le nombre de 6 obtenus lors des $n$ premiers lancers. \be \item Déterminer la loi de $S_1$ puis celle $S_2$. \item Quelle est la loi de $S_n$? son espérance? \item Montrer que $\P(\bigcup_{n=1}^{+\infty} \left [ S_n=N \right ])=1$ \ee \begin{comment} \be \item \fbox{$S_1\hookrightarrow {\mathcal B}(N,\frac{1}{6})$, $S_2\hookrightarrow {\mathcal B}(N,1-(\frac{5}{6})^2)$} \item \fbox{$S_n\hookrightarrow {\mathcal B}(N,1-(\frac{5}{6})^n)$ } \item Théorème de limite monotone p21 $ECS1 $ \\ $( \left [ S_n=N \right ])_{n\in \N}$ est une suite croissante d'événements donc $$\P(\bigcup_{n=1}^{+\infty} \left [ S_n=N \right ])=\lim_{n\rightarrow +\infty} \P( \left[ S_n=N \right ])=\lim_{n\rightarrow +\infty}(1-(\paf{5}{6})^n)\boxed{=1}$$ Interprétation : avec une infinité de lancers, presque sûrement tous les dés donneront un 6. \ee \end{comment} \newpage \exos \begin{enumerate} \item {\bf Question de cours :} Théorème de d'Alembert-Gauss. \item Soit $n \geqslant 1$ et soient $a_0, a_1, \ldots, a_{n-1}$ des réels positifs non tous nuls. \\ On considère le polynôme $P \in \mathbb{R}_n[X]$ défini par : $P(X)=X^n-\displaystyle\sum_{k=0}^{n-1}a_kX^k$. \begin{enumerate} \item Montrer que la fonction $f : x \mapsto -\dfrac{P(x)}{x^n}$ est strictement décroissante sur $]0,+\infty[$. \item En déduire que le polynôme $P$ admet exactement une racine dans $\R_{+,*}$ \\ On note $\alpha$ cette racine par la suite. \item Soit $z$ une racine complexe de $P$. \begin{enumerate}\item Si $z\neq 0$, déterminer le signe de $f(|z|)$. \item En déduire que : $|z|\leqslant\alpha$. \end{enumerate} \end{enumerate} Soit $n \geqslant 1$, soient $b_0, b_1, \ldots, b_{n-1}$ des réels non tous nuls et soit $b_n$ un réel non nul. \\ On considère maintenant le polynôme $Q \in \mathbb{R}_n[X]$ défini par : $Q(X) = \displaystyle\sum_{k=0}^{n}b_kX^k$. \item \begin{enumerate} \item Montrer que l'équation $\displaystyle\sum_{k=0}^{n-1}|b_k|x^k=|b_n|x^n$ admet une unique solution réelle strictement positive. \\ On note cette solution $\beta$. \item Soit $z$ une racine complexe de $Q$. \\ Montrer que $|z|\leqslant\beta$. Notons maintenant $z_1,z_2, \ldots ,z_n$ les $n$ racines complexes (distinctes ou non) de $Q$ avec : $$|z_1|\leqslant|z_2|\leqslant \cdots \leqslant |z_n|\leqslant \beta$$\ee \item \begin{enumerate} \item On \textbf{admet} que pour tout entier $k\in [\![0,n]\!]$, on a : $\left|\dfrac{b_k}{b_n}\right|\leqslant \begin{pmatrix}n\\k\end{pmatrix}|z_n|^{n-k}$. \\ En déduire que~: ${\beta}^n\leqslant \displaystyle\sum_{k=0}^{n-1}\begin{pmatrix}n\\k\end{pmatrix}{\beta}^k|z_n|^{n-k}$. \item Montrer finalement que : $(\sqrt[n]{2}-1)\beta\leqslant |z_n|$. \end{enumerate} \item On va prouver la formule admise \be\item On note pour $k\in\llb0,n\rrb$, $E_k$ l'ensemble des parties à $k$ éléments de $\llb0,n\rrb$. Factoriser $Q$, et en déduire que $\forall k\in\llb0,n\rrb$, $\frac{b_k}{b_n}=\sum_{\acco{i_1,...,i_{n-k}}\in E_{n-k}}z_{i_1}z_{i_2}...z_{i_{n-k}}$ \item Prouver alors la formule admise. \ee \end{enumerate} \begin{comment} \begin{enumerate} \item Programme officiel ECS1 page 7 \item \begin{enumerate} \item Pour tout $x>0$, $f(x)=-1+\displaystyle\sum_{k=0}^{n-1}a_kx^{k-n}$.\\ La fonction $f$ est donc la somme de fonctions décroissantes sur $]0,+\infty[$, dont au moins une (les $a_k$ sont non tous nuls) est strictement décroissante.\\ \fbox{La fonction $f$ est donc strictement décroissante sur $]0,+\infty[$.} \item La fonction $f$ est continue et strictement décroissante sur $]0,+\infty[$, donc réalise une bijection de $]0,+\infty[$ vers $f(]0,+\infty[ = \displaystyle]\lim_{+\infty}f,\lim_{0^+}f[=]-1,+\infty[$ car il existe $k\in [\![0,n-1]\!]$ tel que $f(x)\underset{x \rightarrow 0}{\sim}a_kx^{k-n}$ avec $a_k>0$. \\ La fonction $f$ s'annule donc une et une seule fois sur $]0,+\infty[$, et donc $P$ aussi (car pour $x > 0$, on a l'équivalence $f(x)=0 \Longleftrightarrow P(x)=0$). $P$ admet donc \fbox{une unique racine réelle strictement positive. } \item Soit $z$ une racine complexe de $P$. \begin{enumerate}\item Supposons $z\neq 0$. On a alors~: $P(|z|)=|z|^n-\displaystyle\sum_{k=0}^{n-1} a_k |z|^k$. Or $P(z)=0$, donc : $z^n = \sum\limits_{k=0}^{n-1} a_k z^k$. \\ Donc par inégalité triangulaire, on a $|z|^n = |z^n| = \left| \sum\limits_{k=0}^{n-1} a_k z^k \right| \leqslant \sum\limits_{k=0}^{n-1} a_k |z^k|$. \\ Ainsi, on a $P(|z|) \leqslant 0$ et donc \fbox{$f(|z|) \geqslant 0 = f(\alpha)$.} \item Si $z \neq 0$, d'après la question précédente, on a $f(|z|) \geqslant f(\alpha)$, donc comme $f$ décroissante, on a \fbox{$|z| \leqslant \alpha$.} \\ Et si $z=0$, c'est bien vrai puisque $0 \leqslant \alpha$. \end{enumerate} \end{enumerate} \item \be\item On note $P_0(X)=X^n-\displaystyle\sum_{k=0}^{n-1}\Big|\dfrac{b_k}{b_n}\Big|X^k.$ Comme $b_n\neq 0$ : $\displaystyle\sum_{k=0}^{n-1}|b_k|x^k=|b_n|x^n\Longleftrightarrow \displaystyle\sum_{k=0}^{n-1}\Big|\dfrac{b_k}{b_n}\Big|x^k=x^n\Longleftrightarrow P_0(x)=0$.\\ $P_0$ vérifie les conditions de la question 2, donc $P_0$ admet une unique racine réelle strictement positive. \fbox{L'équation $\displaystyle\sum_{k=0}^{n-1}|b_k|x^k=|b_n|x^n$ admet donc une unique solution réelle strictement positive notée $\beta$.} \item En notant $z$ une racine complexe de $Q$, alors $|b_n z^n|\leq \sum_{k=1}^{n-1} |b_k| |z|^k $ et donc $P_0(|z|) \leq 0$.\\ Or $\lim_{x\rightarrow +\infty } P_0(x)=+\infty$ donc par le théorème des valeurs intermédiaires $\beta\in [|z|, +\infty [$ et donc $$\boxed{|z| \leqslant \beta}$$ \ee \item %Notons $z_1,z_2,...,z_n$ les $n$ racines complexes (distinctes ou non) de $Q(X)$ avec : $$|z_1|\leqslant|z_2|\leqslant \cdots \leqslant |z_n|\leqslant \beta$$ \begin{enumerate} \item On admet que pour tout entier $k\in [\![0,n]\!]$, on a : $\left|\dfrac{b_k}{b_n}\right|\leqslant \begin{pmatrix}n\\k\end{pmatrix}|z_n|^{n-k}$. \\ En multipliant par $\beta^k$, puis en sommant, il vient : $$ \displaystyle\sum_{k=0}^{n-1}\beta^k\left|\dfrac{b_k}{b_n}\right|\leqslant \displaystyle\sum_{k=0}^{n-1}\begin{pmatrix}n\\k\end{pmatrix}\beta^k|z_n|^{n-k}$$ Or par définition de $\beta$, on a $\displaystyle\sum_{k=0}^{n-1}|b_k|\beta^k=|b_n|\beta^n$, d'où : $\boxed{{\beta}^n\leqslant \displaystyle\sum_{k=0}^{n-1}\begin{pmatrix}n\\k\end{pmatrix}{\beta}^k|z_n|^{n-k}}$. \item On en déduit que $\beta^n\leqslant (\beta+|z_n|)^{n}-\beta^n \Longleftrightarrow 2\beta^n\leqslant (\beta+|z_n|)^{n}$. \\ On conclut par croissance de la fonction $x\mapsto \sqrt[n]{x}$ sur $\mathbb{R}_+$ que : $\boxed{(\sqrt[n]{2}-1)\beta\leqslant |z_n|}$. \end{enumerate} \item \be\item Les relations coefficients/racines ne sont pas au programme. On factorise : $Q(X)=b_n\prod_{j=1}^n(X-z_j)$ On développe ce produit, on a somme de produit de "X" et de $z_j$. Pour $k\in\llb0,n\rrb$, on aura un terme de degré $k$ quand le facteur $"X"$ sera pris $k$ fois et le facteur $z_j$ $n-k$ fois. Ainsi $b_kX^k=b_n\sum_{\acco{i_1,...,i_{n-k}}\in E_{n-k}}z_{i_1}z_{i_2}...z_{i_{n-k}}$ et \fbox{$\forall k\in\llb0,n\rrb$, $\frac{b_k}{b_n}=\sum_{\acco{i_1,...,i_{n-k}}\in E_{n-k}}z_{i_1}z_{i_2}...z_{i_{n-k}}$} \item Soit $k\in\llb0,n\rrb$. Par inégalité triangulaire $\abs{\frac{b_k}{b_n}}\leq \sum_{\acco{i_1,...,i_{n-k}}\in E_{k-1}} \abs{z_{i_1}z_{i_2}...z_{i_{n-k}}}\leq \sum_{\acco{i_1,...,i_{n-k}}\in E_{n-k}}\abs{z_n}^{n-k}$ Or, il y a $\binom{n}{n-k}=\binom{n}{k}$ éléments dans $E_{n-k}$, donc $\binom{n}{k}$ termes dans la somme. \fbox{$\forall k\in\llb0,n\rrb$, $\abs{\frac{b_k}{b_n}}\leq \binom{n}{k}\abs{z_n}^k$} \ee\ee % \end{enumerate} \end{comment} \newpage \Esps On consid\`ere un circuit \'electronique avec 3 composants $C_1$, $C_2$ et $C_3$.\\ Ce circuit ne fonctionne qui si $C_1$ fonctionne ainsi que $C_2$ ou $C_3$.\\ Sachant que les dur\'ees de vie de chaque composant, suppos\'ees mutuellement ind\'ependantes, suivent une loi exponentielle de param\`etre $\lambda>0$, d\'eterminer la loi de la dur\'ee de vie du circuit complet.\\ Proposer un programme Scilab permettant de vérifier le résultat obtenu. \begin{comment} En notant $V_i$ les dur\'ees de vie de chaque composant, on doit ainsi calculer la loi de $$ V=\min(V_1,V'_1) $$ avec $V'_1=\max(V_2,V_3)$. On a ainsi $$ \P(V'_1\le t)=\P(V_2\le t)\P(V_3\le t)=(1-\exp(-\lambda t))^2 $$ puis $$ \P(V>t)=\P(V_1>t)\P(V'_1>t)=\exp(-\lambda t)(1-(1-\exp(-\lambda t))^2)=\exp(-2\lambda t)(2-\exp(-\lambda t)) $$ et enfin (la fonction de répartition est bien continue sur $\R$ est de classe ${\mathcal C}^1$ sur $\R^*$) $$\boxed{f(t)=4\lambda\exp(-2\lambda t)-3\lambda\exp(-3\lambda t)=\lambda \exp(-2\lambda t)(4-3\exp(-\lambda t))} $$ On peut écrire le programme Scilab suivant pour vérifier ce résultat: \begin{verbatim} Ntest=100000;val=[];lambda=2; for i=1:10000; T=grand(3,1,'exp',1/lambda); val=[val,min(T(1),max(T(2),T(3)))]; end clf() histplot(100,val) x=0:0.01:(3*1/lambda) y=lambda*exp(-2*lambda*x).*(4-3*exp(-lambda*x)); plot2d(x,y) \end{verbatim} \includegraphics{Fig22.png} \end{comment} \newpage \exos Soit $n$ un entier supérieur ou égal à $2$. On note : $$\displaystyle \mathbb{A} = \acco{ M = (m_{i,j})~ \in M_n(\mathbb{R})~ / ~ \forall i \in \llb1,n\rrb, \;\abs{m_{i,n-i+1}} ~>~ \sum_{\begin{subarray}{c} j=1 \\ j \not=n+1-i \end{subarray}}^n |m_{i,j}| },$$ $$\displaystyle \mathbb{B} = \acco{ M = (m_{i,j})~\in M_n(\mathbb{R})~ / ~\forall ~i\in\llb1,n\rrb : \displaystyle m_{i,i} \neq 0 ~{\rm et } ~ \sum_{\begin{subarray}{c} (i,j)\in \llb 1,n \rrb ^2\\ i \not=j \end{subarray}}\frac{m_{i,j}^2}{m_{i,i}^2}<1 }.$$ \be\item {\bf Question de cours :} Formule du produit matriciel. \item Est-ce que $\mathbb{A} \subseteq \mathbb{B}$ ? Est-ce que $\mathbb{B} \subseteq \mathbb{A} $ ?\\ \item On munit $M_{n,1}(\mathbb{R})$ de sa norme euclidienne canonique : si $X=\begin{pmatrix}x_1\\\vdots\\x_n\end{pmatrix}\in M_{n,1}(\mathbb{R})$, on note $\norme{X}=\sqrt{\sum_{i=1}^n x_i^2}$. Soit une matrice $M$ de $M_n(\mathbb{R})$.\\ \be\item Montrer que $$\forall X\in M_{n,1}(\R),\quad \|M X\|^2 \leq Tr(M ^tM) \| X \|^2.$$ \item En déduire que si $Tr(M ^tM) < 1$ alors $I_n + M$ est inversible.\\ \item Soit $B=(b_{i,j})_{1\leq i,j\leq n}$ une matrice de ${\mathbb B}$. On note D la matrice diagonale dont les coefficients diagonaux sont égaux à ceux de B. Déterminer une matrice $C\in M_n(\R)$ telle que $B=DC$. \item En déduire que $\mathbb{B} \subseteq GL_n(\mathbb{R}).$\\ \ee \item \be\item Soient $k,l\in\llb1,n\rrb$ et $E_{k,l}$ le vecteur de la base canonique de $M_n(\R)$ dont tous les coefficients sont nuls, sauf celui situé sur la $k$-ième ligne et la $l$-ième colonne qui vaut $1$. Vérifier que $\forall a\in [0,1[$ $I_n+aE_{k,l}\in{\mathbb B}.$ \item Caractériser le sous-espace vectoriel $F$ engendré par l'ensemble des matrices de ${\mathbb B}$. \ee \ee \begin{comment} \be\item Programme officiel ECS1 page 8. \item $\displaystyle M =\begin{pmatrix}&&1\\&\iddots\\1\end{pmatrix}$ est dans $\mathbb{A}$ (le coefficient "antidiagonal" domine la ligne $i$) mais pas dans $\mathbb{B}$ (les coefficients diagonaux sont nuls).\\ $I_n$ est dans $\mathbb{B}$ (les coefficients diagonaux sont non nuls et $\sum_{\begin{subarray}{c} 1 \le i\le n\\ 1 \le j\le n \\ i \not=j \end{subarray}}\frac{m_{i,j}^2}{m_{i,i}^2}=0<1$) mais pas dans $\mathbb{A}$.\\ \fbox{ On n'a ni ${\mathbb A}\subset {\mathbb B}$ ni ${\mathbb B}\subset {\mathbb A}$} \item \be\item On a par la formule du produit matriciel : La $i$-ième coordonnée de $MX$ vaut $\sum_{j=1}^n m_{i,j}x_j$ et $M ^tM=(\sum_{k=1}^nm_{i,k}m_{i,j})_{1\leq i,j\leq n}$. Ainsi $\displaystyle Tr(M ^tM) = \sum_{(i,j)} m_{i,j}^2$ et $\displaystyle||M.X||^2=\sum_{i=1}^n(\sum_{j=1}^n m_{i,j}x_j)^2$. \\ Or,\, d'après l'inégalité de Cauchy-Schwarz : \: $\displaystyle (\sum_{j=1}^n m_{i,j}x_j)^2\le\sum_{j=1}^n m^2_{i,j}\,.\:\sum_{j=1}^n x^2_j=(\sum_{j=1}^n m_{i,j}^2).\: \|X\|^2$.\\ $\displaystyle \|M.X\|^2 \le \sum_{i=1}^n\left((\sum_{j=1}^n m_{i,j}^2 )\,.\: \|X\|^2\right)= \sum_{i=1}^n \sum_{j=1}^n m_{i,j}^2\!.\,\|X\|^2= Tr(M ^tM).\|X\|^2$ ,\quad d'où : \; $$\boxed{\|M.X\|^2 \le Tr(M ^tM).\|X\|^2}$$ \item On suppose que $ Tr(M ^tM)\!<\!1$. Montrons par l'absurde que $I_n+M$ inversible.\\ Sinon, il existe \, $X\!\in\!M_{n,1}(\R))\!-\!\acco{0}$ , tel que \: $(I\!+\!M)\hspace{.1mm}.\hspace{.5mm}X\!=\!0$\,,\; c'est-à-dire \: tel que : \,$-X\!=\!M.X$. Mais alors $ \|X\|\ =\!\|M.X\|$ d'où, d'après a), $ \|X\|^2 \!\le (Tr(M ^tM)).\|X\|^2 $ Comme $\norme{X}\neq 0$, on obtient $ 1 \le Tr(M ^tM) $ : ce qui contredit \;$Tr(M ^tM)\!<\!1$. Donc \fbox{la matrice $I + M$ est inversible.}\\ \item On a: $B = D C$ où \fbox{$ \displaystyle C = \begin{pmatrix} \frac{b_{1,1}}{b_{1,1}}&\frac{b_{1,2}}{b_{1,1}}&\cdots &\frac{b_{1,n}}{b_{1,1}}\\ \frac{b_{2,1}}{b_{2,2}}& \frac{b_{2,2}}{b_{2,2}} &\cdots &\frac{b_{2,n}}{b_{2,2}}\\ \vdots & & \ddots & \vdots\\ \frac{b_{n,1}}{b_{n,n}}&\frac{b_{n,2}}{b_{n,n}}&\cdots &\frac{b_{n,n}}{b_{n,n}} \end{pmatrix}$.} \item Montrons que $B$ est le produit de deux matrices inversibles. La matrice $D$ est inversible car ses coefficients non non nuls On note $T=C-I_n$. On calcule Tr$(T^tT)=\sum_{\begin{subarray}{c} 1 \le i\le n\\ 1 \le j\le n \\ i \not=j \end{subarray}}\frac{b_{i,j}^2}{b_{i,i}^2}<1$, puisque $B$ est dans $\mathbb{B}$. Donc $I_n+T=C$ est inversible \fbox{Donc la matrice $B$ est inversible.}\\ \ee \item \be \item si \,$k\!=\!l$, \: la matrice \,$aE_{k,l}+I_n$\, est diagonale \: et \: $\displaystyle \sum_{\begin{subarray}{c} i\not=j \end{subarray}}\frac{m_{i,j}^2}{m_{i,i}^2}=0<1$ \: est vérifié.\\ %==== rond si \,$k\!\not=\!l$,\, la matrice \,$E_{k,l}+I_n$\, a des $1$ sur la diagonale et un seul terme non nul hors de la diagonale,\, valant $a$. Alors~: \\ \hspace*{6.8cm} $\displaystyle \sum_{\begin{subarray}{c} i\not=j \end{subarray}}\frac{m_{i,j}^2}{m_{i,i}^2}=\frac{a^2}{1^2}<1$ \\ \fbox{Dans tous les cas $aE_{k,l}+I_n\in{\mathbb B}$} \item Soit $F$ le sous-espace vectoriel engendré par $\mathbb{B}$. D'après $(a)$, il contient $I_n+aE_{k,l}$ pour $a\in]0,1[$. Mais il contient aussi $I_n\in{\mathbb B}$, donc, il contient tous les vecteurs de la base canonique. Ainsi,\; l'ensemble F contient tous les vecteurs de la base canonique, et donc l'espace $M_n(\R)$ tout entier. \fbox{$F=M_n(\R)$} \ee\ee \end{comment} \newpage \Esps Soit $X$ une variable aléatoire à valeurs dans $\N^*$ admettant une espérance.\\ Montrer que $\dfrac{1}{X}$ admet une espérance puis montrer que $\E(X) \E \left(\dfrac{1}{X}\right) \geq 1$. Étudier le cas d'égalité. \begin{comment} Étudions la nature de la série $\sum_{n\in \N^*} \dfrac{1}{n} \P(X=n)$. Pour tout $n\in \N^*,\ 0 \leqslant \dfrac{1}{n}\P(X=n) \leqslant n \P(X=n)$ \fbox{La variable $\dfrac{1}{X}$ admet donc une espérance} en vertu du théorème de comparaison de séries à termes positifs. On peut aussi dire que la variable $\dfrac{1}{X}$ est bornée. Soit $t \in \R$. Considérons la variable aléatoire $Y_t = \left(t \dfrac{1}{\sqrt{X}}+ \sqrt{X}\right)^2 = \dfrac{t^2}{X} + 2t + X$. Puisque $X$ et $\dfrac{1}{X}$ admettent une espérance, $Y_t$ admet une espérance par linéarité. On en déduit que : \[ \forall t\in \R,\ \E\left(\dfrac{1}{X}\right)t^2 + 2t + \E(X) \geqslant 0. \] Puisque $\E\left(\dfrac{1}{X}\right) >0$ ($X \in \N^*$ presque-sûrement), on a trinôme du second degré de signe constant. Son discriminant est donc négatif ou nul, i.e. $4 - 4\E(X)\E\left(\dfrac{1}{X}\right)$, i.e. \fbox{$\E(X) \E \left(\dfrac{1}{X}\right) \geq 1$.} D'après le raisonnement précédent, \begin{align*} \E(X) \E \left(\dfrac{1}{X}\right) = 1 &\Leftrightarrow \exists t_0 \in \R, \ \E\left(\dfrac{1}{X}\right)t_0^2 + 2t_0 + \E(X) \\ &\Leftrightarrow \exists t_0 \in \R, \ \E\left[\left(t_0 \dfrac{1}{\sqrt{X}}+ \sqrt{X}\right)^2\right]=0\\ &\Leftrightarrow \exists t_0 \in \R, \ t_0 \dfrac{1}{\sqrt{X}}+ \sqrt{X}=0 \ \mathop{p.s.} \\ &\Leftrightarrow \boxed{X \text{ est constante presque-sûrement}.} \end{align*} \end{comment} \newpage \exos Soit $n\in\N$. On pose $E=\R_n[X]$ et, pour $P$ et $Q$ dans $E$, on pose: $$\prodscal{P}{Q}=\int_{-1}^1P(t)Q(t)\sqrt{\frac{1-t}{1+t}}\dt$$ \be \item {\bf Question de cours :} que peut-on dire des sous-espaces propres d'un endomorphisme symétrique d'un espace euclidien? \item Justifier que l'intégrale ci-dessus est bien définie puis prouver que $(P,Q)\mapsto \prodscal{P}{Q}$ est un produit scalaire sur $E$. \item Soit $\varphi$ définie sur $E$ par $\varphi(P)=(X^2-1)P''+(2X+1)P'$. Justifier que $\varphi$ est un endomorphisme de $E$. On admet pour l'instant qu'en outre, $\varphi$ est un endomorphisme \emph{symétrique} de $E$. Ce point sera prouvé à la dernière question de l'exercice. \item \be \item Déterminer les valeurs propres de $\varphi$. \item On note $\lambda_0, \lambda_1,\ldots, \lambda_n$ les valeurs propres de $\varphi$ ordonnées par ordre croissant. Soit $P_k$ un vecteur propre associé à $\lambda_k$ pour $k\in\llb 0,n\rrb$. Montrer que la famille $(P_k)_{0\leq k\leq n}$ est orthogonale et déterminer le degré de $P_k$ pour tout $k\in\llb 0,n\rrb$. \ee \item Soit $k\geq 1$. \be \item Montrer que $P_k$ possède au moins une racine d'ordre impair dans $]-1,1[$. \item On note $a_1,\ldots,a_r$ les racines d'ordre impair de $P_k$ sur $]-1,1[$ et soit $S=\prod_{i=1}^r(X-a_i)$. En considérant la quantité $\prodscal{S}{P_k}$, montrer que $P_k$ a $k$ racines distinctes dans $]-1,1[$. \ee \item On prouve dans cette question que l'endomorphisme $\varphi$ est symétrique. Soient $P$ et $Q$ dans $E$. Montrer, en intégrant par parties, que: $$\int_{-1}^1(1-t)^{3/2}(1+t)^{1/2}P''(t)Q(t)\dt=\int_{-1}^1(2t+1)P'(t)Q(t)\sqrt{\frac{1-t}{1+t}}\dt-\int_{-1}^1(1-t)^{3/2}(1+t)^{1/2}P'(t)Q'(t)\dt$$ et en déduire le résultat demandé. \ee %------------------------------------------------------------------------------------------------------------------ \begin{comment} \be \item Question de cours: programme ECS2 2013 p. 17. \item Le seul problème pour la définition de l'intégrale est en $-1$, or: $$P(t)Q(t)\sqrt{\frac{1-t}{1+t}}\eq{-1}P(-1)Q(-1)\frac{\sqrt{2}}{\sqrt{1+t}}$$ qui est intégrable par comparaison à une intégrale de Riemann convergente ($1/2<1$). L'application $(P,Q)\mapsto \prodscal{P}{Q}$ est clairement symétrique et bilinéaire par bilinéarité du produit et linéarité de l'intégrale. Pour $P\in E$: $$\prodscal{P}{P}=\int_{-1}^1P(t)^2\sqrt{\frac{1-t}{1+t}}\dt\geq 0$$ par positivité de l'intégrale et: \begin{eqnarray*} \int_{-1}^1P(t)^2\sqrt{\frac{1-t}{1+t}}\dt=0& \Longrightarrow & \forall t\in]-1,1[\quad P(t)^2\sqrt{\frac{1-t}{1+t}}=0\text{ par continuité et positivité}\\ & \Longrightarrow & \forall t\in]-1,1[\quad P(t)=0\\ & \Longrightarrow & P=0 \text{ car $P$ a une infinité de racines} \end{eqnarray*} L'application\fbox{ $(P,Q)\mapsto \prodscal{P}{Q}$ est donc bien un produit scalaire. } \item La linéarité de $\varphi$ est immédiate par linéarité de la dérivation et de la multiplication. De plus, $\varphi$ est clairement à valeurs dans $\R[X]$ et si $P\in E$, $\deg (X^2-1)P''=\deg (2X+1)P'=\deg P$ donc $\deg\varphi(P)\leq \deg P$ et donc $\varphi(P)\in E$. Ainsi, \fbox{$\varphi$ est un endomorphisme de $E$. } \item \be \item On remarque que $\varphi(1)=0$, $\varphi(X)=2X+1$ et pour $2\leq k\leq n$: $$\varphi(X^k)=k(k+1)X^k+kX^{k-1}-k(k-1)X^{k-2}$$ La matrice de $\varphi$ dans la base canonique est donc triangulaire supérieure et ses coefficients diagonaux sont les $n+1$ nombres \fbox{$k(k+1)$ avec $0\leq k\leq n$.} Ce sont donc les valeurs propres de $\varphi$. \item Avec les notations de l'énoncé on a donc $\lambda_k=k(k+1)$ pour $0\leq k\leq n$. Comme $\varphi$ a $n+1$ valeurs propres distinctes, $(P_0,\ldots,P_n)$ est une base de $E$. De plus comme $\varphi$ est symétrique, les sous-espaces propres de $E$ sont orthogonaux deux à deux et \fbox{la famille $(P_0,\ldots,P_n)$ est donc orthogonale. } Montrons que pour tout $k\in\llb 0,n\rrb$, $\deg P_k=k$. On le prouve par récurrence sur $n$. On sait déjà que le résultat est vrai pour $n=0$ car $\varphi(a)=0$ pour $a$ un polynôme constant. On suppose le résultat vrai au rang $n-1$ pour $n\geq 1$. Comme la matrice $\R_{n-1}[X]$ est triangulaire, $\R_{n-1}[X]$ est stable par $\varphi$ et la restriction de $\varphi$ à $\R_{n-1}[X]$ induit un endomorphisme de $\R_{n-1}[X]$ qui admet pour valeurs propres $\lambda_0,\ldots, \lambda_{n-1}$ comme on le voit en écrivant les matrices de $\varphi$ et de sa restriction dans la base canonique -- c'est le calcul effectué à la question précédente. En fait, en considérant la restriction de $\varphi$ à $\R_{n-1}[X]$ on se retrouve dans la même situation une dimension en dessous. Autrement dit, en notant $\varphi=\varphi_n$ pour $E=\R_n[X]$, $\varphi_{n|\R_{n-1}[X]}=\varphi_{n-1}$. Ainsi $(P_0,\ldots,P_{n-1})$ est une famille de vecteurs propres associée aux valeurs propres $(\lambda_0,\ldots,\lambda_{n-1})$ et par l'hypothèse de récurrence $\deg P_k=k$ pour $0\leq k\leq n-1$. Comme $(P_0,\ldots, P_n)$ est une base de $E$, on a nécessairement \fbox{$\deg P_n=n$} ce qui achève la récurrence. \ee \item \be \item Si $P_k$ n'a que des racines d'ordre pair sur $]-1,1[$, il ne change jamais de signe lorsqu'il s'annule sur cet intervalle et donc par le théorème des valeurs intermédiaires, il est de signe constant. Quitte à remplacer $P_k$ par $-P_k$, on peut supposer $P_k\geq 0$ sur $]-1,1[$. Alors, par continuité et positivité de l'intégrale: $$\prodscal{1}{P_k}=\int_{-1}^{1}P_k(t)\sqrt{\frac{1-t}{1+t}}\dt>0$$ Mais $1$ est dans le sous-espace propre associé à $\lambda_0=0$ d'où $\prodscal{1}{P_k}=0$. Contradiction. \item Par le même raisonnement, on suppose que le nombre $r$ de racines d'ordre impair de $P_k$ vérifie $r0$. C'est une contradiction donc $r=k$ et donc \fbox{$P_k$ admet $k$ racines distinctes} dans $]-1,1[$. Ces racines sont alors toutes simples. \ee \item On pose $f(t)=(1-t)^{3/2}(1+t)^{1/2}Q(t)$. Cette fonction est ${\cal C}^1$ sur $]0,1[$ et pour tout $t\in]0,1[$: \begin{eqnarray*} f'(t)&=&-\frac{3}{2}(1-t)^{1/2}(1+t)^{1/2}Q(t)+\frac{1}{2}(1-t)^{3/2}(1+t)^{-1/2}Q(t)+(1-t)^{3/2}(1+t)^{1/2}Q'(t)\\ & = &-\frac{3}{2}(1+t)Q(t)\sqrt{\frac{1-t}{1+t}}+\frac{1}{2}(1-t)Q(t)\sqrt{\frac{1-t}{1+t}}+(1-t)^{3/2}(1+t)^{1/2}Q'(t)\\ & = & -(2t+1)Q(t)\sqrt{\frac{1-t}{1+t}}+(1-t)^{3/2}(1+t)^{1/2}Q'(t) \end{eqnarray*} En intégrant par parties -- valide car $P'$ est également de classe ${\cal C}^1$ -- on obtient alors: \begin{eqnarray*} \int_{-1}^1(1-t)^{3/2}(1+t)^{1/2}P''(t)Q(t)\dt&= & \int_{-1}^1f(t)P''(t)\dt\\ & = & \crocint{f(t)P'(t)}_{-1}^1- \int_{-1}^1f'(t)P'(t)\dt\\ & = & - \int_{-1}^1f'(t)P'(t)\dt\\ &= &\int_{-1}^1(2t+1)P'(t)Q(t)\sqrt{\frac{1-t}{1+t}}\dt-\int_{-1}^1(1-t)^{3/2}(1+t)^{1/2}P'(t)Q'(t)\dt \end{eqnarray*} On trouve bien le résultat annoncé. En faisant passer la première intégrale du second membre dans le premier membre, l'égalité obtenue nous dit alors exactement que: $$\prodscal{\varphi(P)}{Q}=\int_{-1}^1(1-t)^{3/2}(1+t)^{1/2}P'(t)Q'(t)\dt$$ L'expression dans le membre de droite est symétrique en $P$ et $Q$ donc on a automatiquement \fbox{ $\prodscal{\varphi(P)}{Q}=\prodscal{P}{\varphi(Q)}$ et $\varphi$ est bien symétrique. } \ee \end{comment} \newpage \Esps \begin{verbatim} function Y=smul1(n,p) X=grand(1,n,'geom',p); disp(X) Z=max(X) T=zeros(1,Z) for k=1:n if T(X(k))==0 then T(X(k))=1 end end Y=sum(T) endfunction function m=smul2(n,p) X=grand(10000,n,'geom',p); Y=zeros(10000,1) for j=1:10000 Z=max(X(j,:)) T=zeros(1,Z) for k=1:n if T(X(j,k))==0 then T(X(j,k))=1 end end Y(j)=sum(T) end m=mean(Y) endfunction \end{verbatim} \be \item Commenter la fonction {\bf smul1}. Quelles sont les valeurs possibles de sortie ? Donner la loi de la VA {\bf smul1}$(2,0.75)$ \item Le résultat de {\bf smul2}$(2,0.75)$ est $1.3971.$ Commentez ce résultat. \ee \begin{comment} \be \item {\bf suml1} simule la VA $Y_n$ qui est égale au nombre de valeurs distinctes prises par $n$ variables $(X_i)_{1\leq i\leq n}$ indépendantes suivant des lois géométriques de paramètre $p$. $Y_n(\Omega)=\llb1,n\rrb$ $Y_2(\Omega)=\{1,2\}$ $$\P(Y_2=1)=P(X_1=X_2)=\sum_{k=1}^{+\infty} p^2((1-p)^2)^{k-1}=\frac{p^2}{1-(1-p)^2}=\boxed{\frac{p}{2-p}=\frac35}$$ $$P(Y_2=2)=1-P(Y_2=1)\boxed{=\frac{2(1-p)}{2-p}=\frac25}$$ \item {\bf smul2} donne la moyenne empirique d'un échantillon de 10000 variables de même loi que $Y_n$ Elle approche donc l'espérance de $Y_n$. $$\E(Y_2)=\frac{4-3p}{2-p} $$ pour $p=\frac{3}{4}$ on a \fbox{$\E(Y_2)=\frac{7}{5}=\frac{14}{10}$}. Le résultat de {\bf smul2} est bien cohérent \ee \end{comment} \end{document}